Jump to content

Wikipedia:Reference desk/Science: Difference between revisions

From Wikipedia, the free encyclopedia
Content deleted Content added
Line 656: Line 656:


How is artificial diamond made? -- [[User:Extra999|Extra999]] ([[User talk:Extra999|talk]]) 18:10, 8 March 2010 (UTC)
How is artificial diamond made? -- [[User:Extra999|Extra999]] ([[User talk:Extra999|talk]]) 18:10, 8 March 2010 (UTC)

== What is meant by decreasing the "vacuum"??? ==

What is meant by decreasing the vacuum?? Is it increasing the vacuum pressure hence decreasing the quality of it?? Or decreasing the emptiness???

Revision as of 13:10, 8 March 2010

Welcome to the science section
of the Wikipedia reference desk.
Select a section:
Want a faster answer?

Main page: Help searching Wikipedia

   

How can I get my question answered?

  • Select the section of the desk that best fits the general topic of your question (see the navigation column to the right).
  • Post your question to only one section, providing a short header that gives the topic of your question.
  • Type '~~~~' (that is, four tilde characters) at the end – this signs and dates your contribution so we know who wrote what and when.
  • Don't post personal contact information – it will be removed. Any answers will be provided here.
  • Please be as specific as possible, and include all relevant context – the usefulness of answers may depend on the context.
  • Note:
    • We don't answer (and may remove) questions that require medical diagnosis or legal advice.
    • We don't answer requests for opinions, predictions or debate.
    • We don't do your homework for you, though we'll help you past the stuck point.
    • We don't conduct original research or provide a free source of ideas, but we'll help you find information you need.



How do I answer a question?

Main page: Wikipedia:Reference desk/Guidelines

  • The best answers address the question directly, and back up facts with wikilinks and links to sources. Do not edit others' comments and do not give any medical or legal advice.
See also:


March 4

In-text and Reference Citation for websites and textbooks

I'm aware of how to do in-text and reference citations for papers. But what is the format for websites and textbooks? —Preceding unsigned comment added by 142.58.132.87 (talk) 00:14, 4 March 2010 (UTC)[reply]

Usually questions about how to use Wikipedia belong at Wikipedia:Help desk rather than the Reference Desk, but I'm going to backstab us by answering: See Template:Cite web and Template:Cite book. In general the Template:Citation template is going to be flexible enough for anything at all, if you choose to read through the bazillion options. Comet Tuttle (talk) 00:28, 4 March 2010 (UTC)[reply]
If you are asking about how to cite things in your own papers (not on Wikipedia), it depends on the citation style you are using. (And for some styles, there simply is no official way to do it.) --Mr.98 (talk) 01:10, 4 March 2010 (UTC)[reply]

It's the same format as on paper. Why would it be any different just because the pages are digitized. —Preceding unsigned comment added by Telijelly (talkcontribs) 13:55, 4 March 2010 (UTC)[reply]

There are LOTS of reasons why it would be different in a digital world - for one thing you can have active links. You don't really need all that stuff about authorship, publishers, dates and page numbers when a single mouse click on the reference will magically teleport you to the actual document being cited. SteveBaker (talk) 14:45, 4 March 2010 (UTC)[reply]
That's fine until the target document moves or is deleted, which happens a lot online: then detailed bibliographical data becomes essential. --Normansmithy (talk) 14:51, 4 March 2010 (UTC)[reply]
That's why the DOI system was introduced - theoretically, as long as the document exists somewhere the DOI should resolve to the URL. —Preceding unsigned comment added by 131.111.185.68 (talk) 16:42, 4 March 2010 (UTC)[reply]

euthanasia

  • Though several people have commented in good faith on this discussion, it appears that the original poster is more interested in either starting a debate or promoting a particular belief. Neither is appropriate for the reference desks. I am proactively closing this discussion down. If anyone is interested in the topic, we have articles on Euthanasia and assisted suicide for further reading. Other discussion should happen at some other website. Just not here. --Jayron32 21:21, 4 March 2010 (UTC)[reply]
This discussion has been closed. Please do not modify it.
The following discussion has been closed. Please do not modify it.


If it were an animal suffering from a severe illness people would say to put it to sleep because it was the humane thing to do. However, why must a human suffer because people are unwilling to do the “humane” thing and let them die with dignity so their suffering may end. —Preceding unsigned comment added by Thekiller35789 (talkcontribs) 00:42, 4 March 2010 (UTC)[reply]

See our articles euthanasia and voluntary euthanasia. The latter says, "As of 2009, some forms of voluntary euthanasia are legal in Belgium, Luxembourg, the Netherlands, Switzerland, and the U.S. states of Oregon and Washington." The article has links to more specific articles, like Euthanasia in the Netherlands, Euthanasia in the United States, etc. Comet Tuttle (talk) 00:58, 4 March 2010 (UTC)[reply]
The real meat-and-potatoes can be found in Voluntary euthanasia#Reasons given against voluntary euthanasia. Vranak (talk) 01:15, 4 March 2010 (UTC)[reply]
was it by any chance a video of a blind dog that made you ask this? I thought exactly the same thing. All the comments were "do the right thing" and it struk me too how if that was a person, no one would say that. Vespine (talk) 05:05, 4 March 2010 (UTC)[reply]
There are many excellent arguments in favour of voluntary euthanasia and assisted suicide, but I don't think the claim that it would place humans on the same eithical footing as animals is going to be one of the most persuasive. After all, we eat animals, but that is hardly an good argument for cannibalism, is it ? Gandalf61 (talk) 09:37, 4 March 2010 (UTC)[reply]
Well, we don't generally claim that we're eating the animals for their own good. We do make that claim for euthanasia. -- Coneslayer (talk) 13:54, 4 March 2010 (UTC)[reply]
We are not really killing the blind dog for his own good, we are doing it for our own convenience, and just deluding ourselves so we feel better. Googlemeister (talk) 14:06, 4 March 2010 (UTC)[reply]
I'm not familiar with the "blind dog" video referenced above, and the OP didn't ask about blindness specifically. I was thinking of illnesses that cause pain, inability to eat, etc. -- Coneslayer (talk) 14:13, 4 March 2010 (UTC)[reply]
We recently had to euthanise ("kill") our dog because at 12 years old, she developed a lump and started to be very slow and lethargic. She turned out to be riddled with cancer. With no way to complain, she'd probably been in considerable pain for six months or more - but as our vet said "Dogs are amazingly tough - they soldier on gracefully under the most crippling pain - but when they give up, they give up entirely." A serious effort to treat her would have cost $10,000 which we simply don't have and couldn't borrow - and would probably have only maybe a one in ten chance of working anyway. Unlike human hospitals, vetenarians don't work for free - even in emergency, life-threatening cases - and the government doesn't provide ultimate fallback health care for dogs. Because neither us nor our vet speak fluent dog - it would have been completely impossible to know whether "end of life care" (painkillers) were sufficient to provide a comfortable (albeit lingering) end.
So after some debate, we paid the vet extra to come to our house, we carried the dog to her favorite spot in the back yard (she really couldn't walk anymore), settled her on her comfy dog-bed, gave her some of her favorite tit-bits (which she didn't eat) and then the vet shot her full of a strong sedative. When she fell asleep, an overdose of muscle relaxant killed her within maybe 30 seconds by stopping her heart and breathing. After we'd shed some tears, the vet took the body away for disposal. Not a bad way to go, all things considered. After a few months, we grabbed a 7 week old puppy from the animal rescue center. The old dog died after a good life and one less puppy had to be euthanised at the rescue center - a net win for dog-kind in general. It's amazing how much the act of repeatedly cleaning dog poop off your living room carpet and finding your best leather shoes reduced to 3cm x 3cm pieces all over the bedroom floor helps you get over the loss of an old friend.
However, for humans who are suffering like that we have the financial support to at least make an effort to cure serious diseases and failing that, we can generally interact with the patient to give them at least some measure of relief from pain in a hospice-like setting. There is still a case for euthanasia ("killing") people who have had enough and are able to clearly say so - but there are certainly legal and ethical hurdles - as our articles indicate. It is unlikely that the loss of an adult due to some terminal disease will result in a family adopting a child as a replacement...and children that are not adopted are not routinely executed by the government.
So there is zero comparison between the two circumstances.
SteveBaker (talk) 14:42, 4 March 2010 (UTC)[reply]


there certainly is comparison, its not all about what the family wants either. if a person lives alone and has no family. if they are blind, paralyzed , in severe pain and allergic to opiates. they should have the right to be killed. —Preceding unsigned comment added by Thekiller35789 (talkcontribs) 15:01, 4 March 2010 (UTC)[reply]

Euthanasia is less the right to take your own-life than it is the right to allow someone else to take your life for you. Be that the state, a relative, a doctor or whoever. Allowing it introduces risks. Risks of systematic abuse, risks of misunderstandings, risks of coercion. On the face of it the idea of allowing euthanasia feels reasonable, feels like a good idea. Thinking through the logical and potential reality is where I (just my opinion) find it gets much more 'grey' and difficult. The key obstacle I believe is whether any party involved will be able to make a fair judgement. Relatives might want to put someone out of their misery because from the outside looking it that sort of a 'non existence' looks miserable, but what if that person doesn't want it but cannot say? What if the person said 10 years ago "if i'm in this position kill me" but once they're in the position they realise they want to live but cannot say? It's really not anything like as simple as you (thekiller35789) are seemingly making it. It's an incredibly complex issue with (for me) compelling arguments on both sides. ny156uk (talk) 17:11, 4 March 2010 (UTC)[reply]
The OP persists in using this Ref. Desk to argue their support for euthanasia viz. "a person..should have the right to be killed". The question "Why must a human suffer?" finds varying answers within various belief systems and is therefore unanswerable neutrally. The OP uses a rhetoric device to fill the vacuum of that unavailable answer with their loaded contention "because people are unwilling to kill them". The underlined action is what is meant but the OP does not admit that. Instead the OP substitutes the weasel worded advocacy "do the "humane" thing and let them die with dignity". That prevarication assumes meanings of humane and dignity that only advocates of euthanasia apply in this context. "Let them die" is a transparent euphemism for kill which by definition is to prevent a life running its natural course. Claiming a justification for the killing to be "so their suffering may end" is direct advocacy of euthanasia. The OP's tendentious question is already handled by the Wikipedia reference that Vranak provided. This section should not be held open for the on-going debate that the OP seeks. Cuddlyable3 (talk) 21:07, 4 March 2010 (UTC)[reply]

Vectors

It's my understanding that, in order to prove that some quantity with direction is a vector, one must show that it is commutative and rotationally invariant. First question: are these the only two criteria? Second question: I've been able to show that the angular velocity vector is commutative, but how would I be able to show that it's rotationally invariant. Finally, how would one show that torque is a vector. Since torque is just a cross product of two other vectors, I quess this would reduce to proving that the cross product of two vectors produces a third vector, but I haven't been able to find a proof of this online. Thanks! —Preceding unsigned comment added by 173.179.59.66 (talk) 04:32, 4 March 2010 (UTC)[reply]

I'm sorry, but nothing that you wrote there makes any sense. Vectors are neither commutative (only operations can be commutative) nor rotationally invariant. The fact that the cross product of two vectors produces a vector is obvious from the definition of the cross product. I'm afraid that you're so far from understanding what you're doing that you might have to go back and relearn the basics of what a vector is. Looie496 (talk) 08:07, 4 March 2010 (UTC)[reply]
1) Sorry, I meant commutative under addition. 2) How can a vector not be rotationally invariant? That would mean an equation like F = ma would depend on the choice of axis. 3) To my knowledge, the cross product is defined as a determinant of some sort. The result of this computation produces something of the form ai + bj + ck, but that doesn't mean it's a vector.173.179.59.66 (talk) 08:35, 4 March 2010 (UTC)[reply]
Looie is right. What you are talking about is usually refered to by physicists as covariant under rotations. The Vectors are covariant under rotations and the vectorial equations are invariant under rotations. Dauto (talk) 19:42, 4 March 2010 (UTC)[reply]
That's funny, because this entire time I was mixing covariant and invariant, really sorry about that! —Preceding unsigned comment added by 173.179.59.66 (talk) 20:25, 4 March 2010 (UTC)[reply]

Have a look at http://en.wikipedia.org/wiki/Vector_%28geometry%29#Vectors.2C_pseudovectors.2C_and_transformations . The components of vectors should transform like coordinates under rotations. It is my understanding that a cross product is actually a pseudovector but don't quote me on that. —Preceding unsigned comment added by 157.193.173.205 (talk) 08:31, 4 March 2010 (UTC)[reply]

I think you should close this question and ask it again on the math reference desk. Those guys will nail this one very quickly. SteveBaker (talk) 14:12, 4 March 2010 (UTC)[reply]
Something doesn't need to have direction to be a vector. The definition of a linear vector space is http://en.wikipedia.org/wiki/Vector_space#Definition Basically the space must have closure, it must be distributive in the scalars, scalar multiplication must be associative, Addition must be commutative and associative, there is a null vector and an inverse under addition for each vector. For instance operators can be treated as vectors.
On another point vectors can be commutative, in a space of Real vectors all vectors are commutative because < A | B > = < B | A > * = < B | A >

82.132.248.82 (talk) 23:13, 4 March 2010 (UTC)[reply]

Who can make the definition of "Space" understandable to users with a high-school education?

I have chased down every (blue) referenced term in both the Space page and across numerous pages in a vain attempt to gain an understanding of the true understanding of the nature of 'space'. I accept the mathematical assumptions made in much the same way as a theoretically dimensionless point is defined for the purposes of function and equation. However, the original "Space" page left me with no descernable improvement in my understanding of both the existential nature of 'space' nor the efficacy of the many clumsy analogies that are proffered in support of meaning. I have read much of the arguements regarding the nature of 'expansion' with reference to space and the balloon analogy is a case in point: To declare that the balloon represents the expanding Universe and then stick an ant on the outside to argue that its corporal structure is unaffected by the expansion of the balloon universe is not helpful. The balloon analogy only works when assuming that the Universe is represented by the balloon; to add an ant (on the outside) is to break the very conventions of the analogy itself by introducing an ant (presumably) outside the Universe!

What would help users like me (and there are many - I assure you) is a comprehensive AND comprehensible explanation of the fundamental nature and function of space in both its physical and theoretical applications. For instance:

  1. If space is a vacuum, or a volume in which nothing exists; how does gravity bend it?
  2. If space is truely a state of non-existence, a vacuum within which nothing exists; surely this definition is based upon the attribute of 'existence' being the sole property of matter, as theories abound of zero-point quantum energy fields and some empirical evidence to support the idea that the Universe is filled with energy in all manner of modes and quantities, etc.
  3. If space is merely a mathematical species of frames or matrixes that provide quantative relevence to various behaviors and transformations at macro, micro and quantum scales - what really causes space lensing??
  4. The elevated circles within which the aether was argued and eventually discarded were not successful in transmitting a truely understandable analogy of this conclusion to the satisfaction of those of us that work with the real Universe. Such ideas may be erroneous but they are also intuitive - to existing generations as well as to our venerable ancestors. Wikipedia is at its best when it conveys to the masses a wholesome and digestible explanation of these endemic misunderstandings.

Finally, quantum physics has introduced us to the concepts of granularity and "forbidden states" that lie between manifest energy levels at sub-atomic scale; however, (in my clumsy attempts to consolidate their analogies and explanations) space appears to be devoid of any such constriction. We are left to ponder such (intuitive) conundrums as:

  1. Is space confined to a resolution that must obey quantum rules?
  2. Can string-theorists simply magnify the dimensions of particle physics by a billion and claim to have discovered a true and final base for the smallness of things?
  3. Does matter really move; or does it simply emerge, first in a quantum spacepoint (A), then sink back into the quantum sea only to manifest again in Spacepoint (B) - that just happens to be the adjacent location closest to (A) - thereby giving the illusion of motion yet simply being a series of manifest existences bounded by the granularity of space; time; and energy/matter?

Scientists have done a wonderful job of improving upon the once clumsy definitions provided for the concept of "Energy" in this Wikipedia; I hope that a comparable attempt will be made to de-mystify the nature of "Space".

GPCViriya (talk) 09:46, 4 March 2010 (UTC)[reply]

Wow, that's a lot to take on. Let me just answer one small part of it by saying that space is not a pure vacuum. It certainly contains small densities of ordinary matter, and quite possibly other forms of matter such as dark matter. StuRat (talk) 13:53, 4 March 2010 (UTC)[reply]
OK - well, let me try to answer some of these:
  1. If space is a vacuum, or a volume in which nothing exists; how does gravity bend it? - In general relativity, matter bends space and bent space produces the illusion of gravity. That's only one view, but it's the prevailing one. Why matter bends space is probably an unanswerable question. It just does.
  2. If space is truely a state of non-existence, a vacuum within which nothing exists; surely this definition is based upon the attribute of 'existence' being the sole property of matter, as theories abound of zero-point quantum energy fields and some empirical evidence to support the idea that the Universe is filled with energy in all manner of modes and quantities, etc. - Space (as described by physicists and cosmologists isn't the same thing as 'vacuum'. It's kinda like the sheet of paper on which a diagram is drawn. The diagram is matter and energy - the paper is "space". It's the underpinning of what we perceive as "distance" or "position". The physicists concept of "space" is present even inside solid bodies. Now, if we're talking about "space" as (say) an astronaut might talk about it (the gaps between solid things that's mostly vacuum) then that's really something different. The "vacuum" of deep space isn't really empty - there are a few atoms per cubic meter, also lots of photons and all sorts of other exotic things like virtual particles.
  3. If space is merely a mathematical species of frames or matrixes that provide quantative relevence to various behaviors and transformations at macro, micro and quantum scales - what really causes space lensing?? - Gravitational lensing can be thought of as the consequences of bent space (see question (1), above). Space is bent - light takes the shortest path through it and the shortest path across a bent surface isn't a straight line (kinda like aircraft flying long distances across the Earth travel along "great circle" routes because they are less distance than "straight lines"). To our senses - which can't directly comprehend the curvature of space, the light appears to bend.
  4. The elevated circles within which the aether was argued and eventually discarded were not successful in transmitting a truely understandable analogy of this conclusion to the satisfaction of those of us that work with the real Universe. Such ideas may be erroneous but they are also intuitive - to existing generations as well as to our venerable ancestors. Wikipedia is at its best when it conveys to the masses a wholesome and digestible explanation of these endemic misunderstandings. - The aether theory was just wrong. It's a discarded hypothesis because it simply doesn't fit the facts. It may be intuitive - but it's a useless explanation because it doesn't explain the behavior of light.
Quantum physics questions:
  1. Is space confined to a resolution that must obey quantum rules? - There is a minimum unit of distance which (I suppose) you could think of as the "resolution" of space.
  2. Can string-theorists simply magnify the dimensions of particle physics by a billion and claim to have discovered a true and final base for the smallness of things? - No. String theory is much more complicated and radically different from the 'normal' view of particle physics.
  3. Does matter really move; or does it simply emerge, first in a quantum spacepoint (A), then sink back into the quantum sea only to manifest again in Spacepoint (B) - that just happens to be the adjacent location closest to (A) - thereby giving the illusion of motion yet simply being a series of manifest existences bounded by the granularity of space; time; and energy/matter? - No, matter moves.
SteveBaker (talk) 14:08, 4 March 2010 (UTC)[reply]
Regarding the balloon analogy: the surface of the balloon is analogous to space. As the balloon inflates, the surface expands. The ant represents a point (or a set of points near each other) on the surface of the balloon. Zain Ebrahim (talk) 14:17, 4 March 2010 (UTC)[reply]
It may be easier if you don't think of matter as something that's "in" a vacuum, but rather think of it as part of the nature of the vacuum. If you look at general relativity by itself, without any matter, it's still a highly nontrivial theory—you have gravitational waves that propagate around and interact with each other, just like any other kind of wave might do, even though there's nothing "in" the space. In the Standard Model of particle physics, which is the best current theory of everything other than gravity, everything other than gravity is like this too. The "particles" are various kinds of oscillations of the vacuum. This is enshrined in the Standard Model in a fairly deep way, through the concept of spontaneous symmetry breaking. The nicely symmetric fundamental vacuum state of the Standard Model is unstable, so, at low enough energies, the vacuum ends up in a different, stable state at some "distance" (in the landscape of possibilities) from the center of symmetry. The particles of everyday life are small oscillations around that state, and they're only indirectly related to oscillations around the unstable center. When particle physicists say "vacuum", they mean a small neighborhood of an equilibrium state, or, in other words, the vacuum proper and all of the low-energy particles and their interactions. For example, when they talk about a "landscape of string vacua" and trying to find a vacuum that corresponds to our world, that's what they mean.
At least this means we don't have to understand space as a container for matter; we only have to understand it as a thing unto itself. Unfortunately, that's the best you're going to get right now. Space behaves like a multidimensional continuum that can be deformed in various ways, and nobody knows anything more than that. There have been many extremely speculative attempts to give it another structure, which go by the name of pregeometry. Probably the best developed of those ideas is spin foam, about which I know nothing beyond what's in the Wikipedia article. -- BenRG (talk) 02:37, 5 March 2010 (UTC)[reply]


Thank you,
(StuRat): You are saying that space is seldom (or ever) found in a pure state of vacuum; however, I understand that matter - ordinary, dark or exotic is not included in the definition of the nature of space, it simply exists at a certain point in space, manifesting its own family of characteristics and behaviors. Therefore, you are describing vacuum as a condition relating to the ratio of matter to the volume of space - not describing the fundamental nature of space (in itself).

Thank you, (SteveBaker):

  1. The unanswerable question, (as how matter bends space) does not eliminate the underlying arguement of my question. My premise was; presuming that space (in its own nature) has no attributes, how can any thing bend it? When one refers to the change in shape (from flat to bent), under the influence of gravity; it is assumed that space (itself) has been influenced by the proximity of matter, which possesses an attribute of mass, which manifests a force called gravity, and has deformed space to reflect the influence of this force upon the very fabric of space.
  2. Your comments (in 2) address my question in point 3. I assume that by paper, you mean physicists rely upon the paper to illustrate the positions and/or co-ordinates occupied by particles within a certain arrangement in space? This accords with my own understanding - that space is used as a foundation for the deliniation of such properties as; position, motion, etc. These properties are not ascribed to space, but to the particles, etc,. that are included in the diagram. Your extra comments regarding the condition of inter-stellar space returns to your earlier comments regarding the nature of a vacuum and do not directly address the nature of space.
  3. In 3, you return to the point, that space is bent and light takes "the shortest path through it" thereby attributing some characteristic to space that is both bent and exerts an influence upon light to respond to this condition by following a bent path. However, this characteristic of space is left undefined albeit your contention that, whatever it is - it is bendable.
  4. I presume that you are referring to the particle/wave characteristics of light when you mention that the aether hypothesis has been proved wrong. I have no arguement with this; however, it does help to illustrate my point regarding the unintended confusions created by using analogies that do not eliminate the underlying inference for just such a medium. By this, I mean; any and all mention of bent-space has the immediate effect of creating the idea of space as a thing, or medium within which the behavior of particles, under certain conditions, can only be explained by pertubations or deformations in the fabric of that space, medium or thing. Creating a vacumm to demonstrate the passage of light without the support of a medium does not eliminate the possibility that space, itself, has the characteristics of a medium. Simply removing all other particles from a region in space does not change the nature of the space itself. A vacuum is not devoid of space any more than the volume contained within the body of a sub-atomic particle is devoid of space, regardless of its mass.
  5. To my quantum physics questions, you peaked my interest when you mentioned the existence of a theoretical minimum distance; then you simply discounted any significance to this assertion when you stated that "matter moves". Clearly both cannot be right. If matter was required to move only a fraction of the "minimum distance", it could not - then we would have to deal with all the potential and/or kinetic energies that may still require transition. If the matter was to move only 1 unit of the minimum distance; it could only achieve this (not by an analogue re-positioning) but by a cessation of existence in one location and a subsequent manifestation at the next, or nearest location (as any smaller movement would be prohibited).
  6. String theory may appear to be more elaborate and complex than particle theory, but it still must address the fundamental characteristics that have been thus far observed. In essence, (to the ear of a layman like myself); string theory has replaced the elementary particle with a field of strings, at a greatly reduced magnitude in both scale, momentum and inertia, and used the more numerous unique characteristics of a string (as opposed to a particle) in their analogies - to ascribe the various configurations and resonant conditions in the stings to represent the observed momentums and behaviors of higher species of elementary units.

I am in no position to question the verasity of the science, experimentation, and theoretical conjecture that is contained in either the Standard Model nor the fledgling String Hypotheses; however, neither side has provided the general population with any working analogies that withstand even the most precursory analysis. Although such examples are not meant to explain the complete picture; they actually pollute the mind of the thinker when these analagies are pushed too far.

Zain Ebrahim's opening comment is a case in point: Given that the analogy of the balloon is representative of the fabric of space and its inflation indicative of the effects of expansion: the ant may represent either a point, or several points in the aforementioned space. Assumed to be a point - the ant experiences no change; as a point is considered to be dimensionless and therefore immune to any change in scale happening around it. Assumed to be a collection of points where the feet touch the surface of the balloon - the ant will find its feet becoming ever more widely spread the longer the expansion of the balloon continues. These are necessary considerations that require the layman to first accept that an ant, outside the balloon, is still in the Universe that has been defined as the surface of the balloon. In fact, we need to discount both the volume of air within the balloon AND the volume of air outside the balloon to focus upon the real efficacy of using the balloon surface as an analogy for an expanding Universe. Introducing the ant invalidates the analogy completely and all subsequent arguements regarding the state of the ant serve only to obfuscate the intended meaning of the exercise. I think the original arguement was concerned with applying a fixed, scalar reference inside an expanding universe. What changes are observed to the space, matter and the arrangement of matter within that space and the influence that the expansion of the universe was having upon both the nature of the matter AND the measurements of their locations and dimensions relative to each other. I presume that the balloon analogy will not be helpful in resolving that one.

Finally, BenRG brings in a veritable plethora of ideas that both illustrate and clarify the nature of my arguement and afford me the chance to zoom in on the specific point that bothers me and is keeping me from appreciating the work that theoretical physics is doing:

  1. The dicotomy between vacuum and matter is not the crux of my questions regarding the nature of space. Any and all descriptions of the interaction between particles (both mass-carrying and force-carrying) may be conceived of as occurring in space without the need to interact with the space in any way. For example; the number of particles, the ratio between their energy levels and their freedom to move within a defined space can be used to calculate their density and even their pressure within a confined space and this can be simply illustrated by analogy without referring to the nature of space - apart from the volume included in the observation. Such a description treats space as a mathematical matrix or yard-stick by which to measure and subsequently observe inherent characteristics of both quantity and quality or behavior. Such observations and calculation do not ascribe any qualitative characteristics to space, only volume is used to provide scale and quantity.
  2. BenRG goes on to provide a whole list of potential waves and particles with which to describe phenomena that do not rely upon the presence of space to contribute to their behavior at all. Regardless of our choice to refer to gravity as a wave, a particle, or a wavefunction; any choice is valid and is all that is required to describe the effect it will have upon another particle. The trajectory of a photon is effected by its interaction with a graviton, gravity wave, or the mass of an attendant atomic or sub-atomic particle, which is included in the observation, and can be predicted, tested and described without the use of space as anything other than a location in which to observe and measure the interation. The Standard Model contains sufficient species of particles to carry any and all of the fundamental material and force-characteristics that exist in the Universe bounded by space, without having to assign any of these behaviors to space itself.
  3. In essence, we do have to think of space as a "container"; with no unique characteristics nor interactions with matter or force, and useful only as a field of observation, manifestation and quantification of all other species of behavior and existence. Any distortions to this "multidimensional continuum" are by inference alone, they can be explained by interations between particles and/or wavefunctions that describe the inherent characteristics and behaviors of specific modes of energy, manifesting on any level, from the quantum sea of virtual-quarks up to the distribution of galaxies within the Universe.

In conclusion; I see no need to include space as a discreet entity of existence, replete with unique characteristics that are essential to the orderly interaction of all other entities contained in the Standard Model. It appears simply to act as an inert background for manifest existence and only makes an appearance (as the 4 dimensions) when observations and measurements are required. So far, science has failed to convince me (an uneducated but rational humanbeing) that space is a term that describes anything other than the mind's ability to differentiate both scale and distance in reference to an observation of discreet objects.

Respectfully,

GPCViriya (talk) 09:33, 5 March 2010 (UTC)[reply]

GPCViriya, you have not sufficiently educated yourself to apply "common sense" arguments to the definition of space. Much of what you say is easily contradicted by referring to any rudimentary cosmology text. For example, you seem to think that space has no attributes; while many would argue that TIME has no attributes (and is, as you said about space, merely a form of measurement), space has attributes, as demonstrated e.g. in the moments between the Big Bang and the completion of the inflationary epoch. Space expanded by something like 10 to the 50th power in a fraction of a second, even though it was a vaccuum at the time. So, at least according to all reputable cosmologists, space indeed has attributes regardless of what it "contains." Seriously: read a book rather than applying "common sense." 63.17.82.123 (talk) 03:25, 6 March 2010 (UTC)[reply]
I agree with '63. We aren't here to debate your "reasoning" - you asked a question, you got good answers - as best we know, they are true. Please don't argue with the respondants - if you think you know better then you didn't need to ask in the first place and you are not allowed to start debates here. However, you certainly don't know better. The problem is that "Common sense" is the worst possible tool in our mental arsenal for dealing with the very large and the very small. This 'common sense' that we seem to be born with is simply an evolutionary instinct and we're not evolved to deal with the universe at the quantum-scale and at cosmological-scales. The first thing you need to do in understanding this stuff is to dump common sense. It doesn't "make sense" that space is curved - but it doesn't have to make sense to be true. The results of physics experiments and mathematics are the truth - and the tools of the scientific method and mathematics are the only way to get any kind of useful understanding of this stuff. Common sense is completely misleading. SteveBaker (talk) 18:35, 6 March 2010 (UTC)[reply]

True: I asked a question, I received answers and I questioned the answers, using the rudimentary education I have and attempting to highlight where my (obviously incorrect) interpretations were leading me. I did not set out to argue; I set out to illustrate my lack of understanding and my difficulties with the analogies, which is all I can address, as it has been stated from the beginning, that I do not know better, do not possess the tertiary education to do the math and hoped to find out more from you learned gentlemen through dialogue. I apologize if my clumsy attempt to analyze these answers has caused any offense - I meant none and made no attempt to hide my ignorance on these matters. I would like to know something about these fundamental concepts, yet when I ask on these kinds of forums; I am either flamed with abuse for being ignorant or ignored completely. I appreciate that these concepts are complicated and do not lend themselves to simple analogy. I appreciate your efforts in trying to enlighten me; however, I seem to have insulted you by highlighting my continueing confusion. My 'reasoning' was an attempt to illustrate how your answers raised ever more conflicts within the confines of the analogies that have been put forward as working analogues of what the true meanings are. Devoid of high mathematics, I am left only with what I think of as reasoning and which I applied to your elaborations in an attempt to illustrate my suspicion; that some analogies give rise to more confusion than they resolve. I'm sorry my approach has insulted you, I will refrain from seeking answers on this talk-page; however, before I go, could someone be so kind as to redirect me where I can go when the knowledge expounded on Wikipedia is too elevated to be understood by people with nothing but common sense to quide them? I am a keen user of Wikipedia; however, I often lack the depth of understanding to appreciate the explanations given. I use the blue links that often highlight the terms I don't understand and I spend many happy hours chasing my ignorances down to the n'th degree. This time; I arrived at "Space" - a word that I presumed I had a reasonable handle upon - but no. In my enthusiasm; I searched the community portal for a suitable place to make my enquiry. I knew I had no right to take part in the discussions that are directly attached to the "space" page, and I was led to this forum, thinking I was on the right track. Once again, I apologize for any irritation I have caused. Sincerely, GPCViriya (talk) 08:26, 7 March 2010 (UTC)[reply]

Inverted crater on Mars

I came across this image from the Mars Reconnaissance Orbiter. The image description calls it an inverted crater. What is an inverted crater? Is it just a round-shaped plateau that just happens to look like a crater? According to inverted relief, natural processes on Mars can cause features like river beds that were once depressions to end up above the surface. Is this another example of that? —Preceding unsigned comment added by 72.94.164.21 (talk) 12:33, 4 March 2010 (UTC)[reply]

[1] (the official NASA version of the Yahoo story with actual facts and stuff) says that the crater was originally a regular crater that filled up with sediment which compacted and became harder rock than the landscape from which the crater was formed - then erosion eroded away the softer crater walls and the surrounding land to leave the compacted sediment as this "inverted crater". A similar kind of thing happens on earth when a volcano erupts in soft material - then the lava hardens into a really solid rock like basalt - and then the soft ground around it erodes leaving a column of harder basalt. The name for those things escapes me for the moment - but I'm sure someone here will tell us. SteveBaker (talk) 13:33, 4 March 2010 (UTC)[reply]
Were you possibly seeking Intrusion or Volcanic plug? -- 124.157.247.225 (talk) 14:53, 4 March 2010 (UTC)[reply]

Inverted relief —Preceding unsigned comment added by Telijelly (talkcontribs) 13:52, 4 March 2010 (UTC)[reply]

American Cat Scans

Because of the sterotype that americans are fat, there was always this joke going around that amerians would get stuck on cat scan machines. Is it true than quite the contrary, the manufacturers made the machines large to accommodate fat people, but in doing so really thin and small people were unable to use the machines because it was scanning air around them and giving false resulte? —Preceding unsigned comment added by Gordanginton (talkcontribs) 13:26, 4 March 2010 (UTC)[reply]

No. First, I doubt the premise of your question. Second, the machines can scan whatever is within their circumference irrespective of whether that happens to be air or flesh. --Tagishsimon (talk) 13:32, 4 March 2010 (UTC)[reply]
My wife used to work as a nurse in the radiography department of a large hospital - she said that they had on occasion cat-scanned premature new-born babies - which are about the smallest thing you'd ever consider trying to scan - and that worked perfectly. So no - this is flat out not true. SteveBaker (talk) 13:36, 4 March 2010 (UTC)[reply]
does she still work as a nurse, just not in radiography anymore? What department is she working in now? I'm asking because I know someone in a similar situation and wonder what a good switch is from the radiography department. Thanks. —Preceding unsigned comment added by 84.153.250.71 (talk) 14:49, 4 March 2010 (UTC)[reply]
I work for a medical device company whose name doubles for a type of screwdriver, and in recent years our marketing materials have been completely written to emphasize the usefulness of our new products in handling "difficult patients" -- this is marketing-speak for fat/overweight/obese however you choose to describe it. Simply put, for something like an ultrasound it just takes more juice to get a good picture when there's more "material" between the sensor and the object you wish to image. 61.189.63.188 (talk) 14:37, 4 March 2010 (UTC)[reply]
You work for Flathead?? ike9898 (talk) 14:45, 4 March 2010 (UTC)[reply]
Or maybe Stanley ? StuRat (talk) 15:58, 4 March 2010 (UTC)[reply]
But there must be limitations on that method. Specifically, doesn't the near flesh get an increased exposure level ? StuRat (talk) 15:55, 4 March 2010 (UTC)[reply]

Patient size is a bigger factor for an MRI machine than for a CAT scanner, because the device benefits a lot more from having the sources and sensors close to the subject. MRI machines are notorious for causing claustrophobia. Looie496 (talk) 17:09, 4 March 2010 (UTC)[reply]

I was going to ask about that. When I was in grad school, the neuroscience/cognition folks had 3 sizes of MRI machines. I think they had one for humans, one for smaller primates, and one that was smaller yet (dogs? rodents?). I was wondering if the smaller machines worked better on the smaller animals, or if they were just less expensive. -- Coneslayer (talk) 17:13, 4 March 2010 (UTC)[reply]
I would think that 1 human sized MRI device would be cheaper then 3, regardless of the size of the others, but if there was a low % of machine downtime, it makes more sense. Googlemeister (talk) 17:30, 4 March 2010 (UTC)[reply]
A rodent-scale MRI can get resolution down to a tenth of a millimeter, as I recall, which is 10 times better than you get with an ordinary human MRI, so there are definite advantages to having a separate machine if there is money to buy it. Looie496 (talk) 17:52, 4 March 2010 (UTC)[reply]
The logical conclusion, then, is that to do a first rate job of analyzing a foot problem, they should saw it off, run it through a rodent MRI machine, then sew it back on. Among other benefits, this approach would likely reduce the number of people requesting scans for brain tumors. :-) StuRat (talk) 19:01, 4 March 2010 (UTC)[reply]

Side issue: Was that a correct use of the word steriotype? Americans have afterall really become on average quite overweight. Dauto (talk) 20:59, 4 March 2010 (UTC)[reply]

I've heard stereotype used to mean "any characteristic ascribed to a group of people, whether true or not", so it sounds correct here, to me. StuRat (talk) 22:40, 4 March 2010 (UTC)[reply]

The only limits are upper limits. Throughout the 1980s and 90s, most US CAT scan machines had an upper limit capacity of 300 lbs: many radiology depts or imaging centers would refuse to image someone who was heavier. Partly it was fitting in the cylinder and partly it was the warranty limit on the moving parts of the machine. There are no lower limits to size except resolution limits. alteripse (talk) 21:11, 4 March 2010 (UTC)[reply]

Compressing Water

Can water be compressed? How much of a space saving will be achieved from compressing water to it's maximum —Preceding unsigned comment added by Harodeuam (talkcontribs) 14:05, 4 March 2010 (UTC)[reply]

Yes, but not to any significant degree. See our article on the properties of water for details, but note that at a pressure of 400 atmospheres, water retains over 98% of its original volume. — Lomn 14:13, 4 March 2010 (UTC)[reply]
The article mentions oceans at 4 KM. I wonder if there is any info about the compression at around 11 KM, the approximate depth of the Mariana Trench? As I recall from physics class, the reason objects can be crushed at great depths is not so much that the water is "denser", but that there's a column of water several miles high crushing it. But I would think that weight would also impact the density of the water itself - even if only to a small extent? ←Baseball Bugs What's up, Doc? carrots14:40, 4 March 2010 (UTC)[reply]
The article also says that the compressibility decreases as the pressure increases. So if you get 1.8% compression at 4 km depth, the compression at 11 km must be more than that, but less than 5%. --Anonymous, 01:21 UTC, March 5, 2010.
Assuming that doesn't push you past a phase transition to one of water's many allotropes, I think you mean. Not that I've checked whether any of those are much denser, or whether the pressure at 11 km is even close to enough. --Trovatore (talk) 01:26, 5 March 2010 (UTC)[reply]
I don't think water has more than one liquid phase; if it did at oceanic pressures I'm sure I would've heard about it somewhere. --Anonymous, 05:15 UTC, March 6, 2010.
I think BBugs may have also been referring to the fact that the pressure at 11 km will be more that 11/4 of the pressure at 4 km because the density of the water that is responsible for the pressure is increasing with depth. You need to solve a differential equation to determine the density as a function of depth give the compressibility (or density) as a function of pressure. 124.157.249.168 (talk) 23:58, 7 March 2010 (UTC)[reply]

I got a related quesiton guys. If you don't store water, but only the hydrogen and the oxygen that you could combust together to produce it, then can you compress THOSE gasses in a way that is equivalent to compressing water? e.g. if you have an x cubic meter room, can you get x + y cubic meters of water to flow out of it, for an appreciable amount of y, not because you had tanks of compressed water in it (which would be very difficult, and the y would be tiny) but because you had tanks of compressed hydrogen and oxygen in it, and combined them to produce your water flow? 84.153.250.71 (talk) 14:44, 4 March 2010 (UTC)[reply]

Water has 2 grams of hydrogen per 18 grams of water, which is 18 millilitres, (See molecular formula and atomic weight). By Avogadro's Law, 2 grams of hydrogen under standard pressure and temperature would occupy 22.4 litres, which is 1,244 times the volume of the water it would form when burnt with atmospheric oxygen. The oxygen required would take up 11.2 litres, if it is not available locally. The burning of hydrogen will also produce very high temperature steam, which would then need to be cooled down. So in answer, no, it is not practicable to carry just the hydrogen. CS Miller (talk) 15:15, 4 March 2010 (UTC)[reply]
Let's see... at standard temp and pressure, it would take 33.6 liters to store 18 grams of water-equivalent. Assuming that we didn't vary temperature, then, you've got to crunch 33600 milliliters to 18 milliliters to get to water volume equivalence. That would be storing the gases at over 1860 atmospheres. Clearly, this is less practical than just storing water. — Lomn 15:04, 4 March 2010 (UTC)[reply]
I'm sorry, I don't get the "upshot". If I have an x cubic meter room underwater (no ambient oxygen source) and I want to store MORE than x cubic liters of water in it, is it better to try to compress water (maybe also cooling it as much as possible short having it freeze and expand) or to try to store it separated, as compressed, cooled gases hydrogen and oxygen? Assume you can just take the heat from combustion and boil the outside water with it... 84.153.250.71 (talk) 15:23, 4 March 2010 (UTC)[reply]
p.s. obviously there is no need to store water in a room underwater. I'm just askin'.. 84.153.250.71 (talk) 15:23, 4 March 2010 (UTC)[reply]
I doubt it. Reacting 29.5 litres (2kg) of liquid hydrogen with 14 litres (16kg) of liquid oxygen will produce 18 litres of water. I don't have the density of frozen hydrogen or oxygen handy. CS Miller (talk) 16:05, 4 March 2010 (UTC)[reply]
There could be a reason. If you want drinking water and you're under the ocean, you'd need to desalinate it and sterilize it before drinking. Whether the apparatus to do this takes up more space than the stored water depends on the duration of the stay as well as the technology used. This is a similar trade-off to space ships either carrying water or recycling water from breath, urine, and poo (when the water comes out brown, it's time to change the filter). StuRat (talk) 15:51, 4 March 2010 (UTC)[reply]
(ec)If you read Lomn's response, he is saying that in order to get the hydrogen content of water down to the same volume as the water (before even thinking about the oxygen), you'd need to produce an ungodly amount of pressure. So yes, if you can produce, say 3000-some atmospheres of pressure, you might be able to store it in a smaller volume as independent gasses (though the water itself would likely have compressed a bit more at that point). At a certain point, hydrogen would enter its metallic phase which is basically as a compressible as it gets without triggering fusion. Since you're not going to produce that amount of pressure just to store water, we can safely say you're better off just storing the water. —ShadowRanger (talk|stalk) 15:52, 4 March 2010 (UTC)[reply]
Oxygen condenses at room temperature under sufficient pressure, doesn't it? It will become essentially incompressible at that point too. I have no idea what that point is, though. --Tango (talk) 04:45, 5 March 2010 (UTC)[reply]
No. According to our article, the critical temperature of oxygen is about −118 Celsius. Above that temperature, no amount of pressure can liquefy it. Still, sure, you can compress it to the point where the atoms are more or less touching, and even though it's not a liquid, it will still be almost incompressible at that point. --Trovatore (talk) 06:25, 5 March 2010 (UTC)[reply]
If you put oxygen under great pressure, it would behave like a liquid, but you could not get a surface on it. So the resistance to pressure would climb. If you just compress water enough you will get a form of ice, at about 2000 atmospheres. Forming metal by compressing hydrogen or oxygen is much more difficult needing millions of atmospheres of pressure. Ice III has a density of 1.16. So 15% above water. Graeme Bartlett (talk) 09:02, 5 March 2010 (UTC)[reply]

What type of medical specialist does this?

This might seem like a request for medical advice, but really it isn't...

The type of medical insurance I have allows me go directly to a specialist without first going to a primary care physician to get a referral. I have an injury which I suspect is something along the lines of a injured upper back muscle (either a small tear or separation from the bone). So, what sort of physician specializes in this sort of problem?

ike9898 (talk) 14:43, 4 March 2010 (UTC)[reply]

Orthopedic surgery --Normansmithy (talk) 15:08, 4 March 2010 (UTC)[reply]
It's really impossible to decide yourself what the actual cause of a given sensation is. I had a similar shoulder problem, which felt like a joint or tendon malfunction. My doctor diagnosed it, however, as muscular adhesions, and sent me to the physiotherapist. The physio cleared it right up. Figuring out which specialist is appropriate is exactly the job of a primary care physician. -- Finlay McWalterTalk 21:00, 4 March 2010 (UTC)[reply]
No argument, but I'm sure you can see the appeal of possibly reducing the number of doctor appointments you have to go to. Of course, if I initially go to the wrong specialist, that won't save anybody any trouble! ike9898 (talk) 21:46, 4 March 2010 (UTC)[reply]
Well, that's the point of visiting a primary care physician. -RobertMel (talk) 19:45, 5 March 2010 (UTC)[reply]

Plasma

How is Plasma made? —Preceding unsigned comment added by Harodeuam (talkcontribs) 14:45, 4 March 2010 (UTC)[reply]

Are you referring to blood plasma or Plasma (physics), a state of matter comprising partly ionized gas? Blood plasma is made up of lots of different components from different sources: proteins, hormones, glucose, water, etc. Plasma (physics)#Common artificial plasma has information on making the other sort. --Normansmithy (talk) 15:14, 4 March 2010 (UTC)[reply]
(EC)Plasma (gas) or Blood plasma (the non-cell part of blood)? (there are other plasmas on the diambiguation page, but these two are the most likely). I suggest looking at those articles. -- Flyguy649 talk 15:16, 4 March 2010 (UTC)[reply]

Sun size

Daft question i'm sure but...is the sun noticeably 'larger' with the naked eye when you're closer to the equator? The moon obviously varies in how far away it is and it's very noticeable when it's larger, but sitting here in sunny (for today) England fully aware that those near the equator are 'closer' to the sun I was wondering whether that effect is noticeable. I'd recommend some people try but staring at the sun isn't that great for your eyes. ny156uk (talk) 17:22, 4 March 2010 (UTC)[reply]

Well, let's think about the scales involved. The sun is somewhere between 90 and 100 million miles away. How much larger do you think it would look if you were 4,000 miles closer (.004%)? This would not be a large enough difference to be noticeable to your average human. Also, the distance from the sun varies by much more then that amount throughout the year because the orbit is not a perfect circle. The moon on the other hand is only about 250,000, so 4,000 miles is around 2%, and might be noticeable. Googlemeister (talk) 17:26, 4 March 2010 (UTC)[reply]
(EC) There would be no difference detectable. The earth's mean radius is 6371 km. The mean distance of the earth to the sun is 149.6 million kilometres. Ignoring the 23° tilt of the earth's axis, someone at the pole is one earth radius farther from the sun than someone at the equator. That's 0.004 percent. I doubt there is a noticeable difference in the size of the sun from perihelion to aphelion. -- Flyguy649 talk 17:32, 4 March 2010 (UTC)[reply]
Also, the width of the Earth may vary by 4000 miles from the poles to the equator, but would vary far less between the equator and England. I'd guess maybe 1500 miles (or 0.0015%), but invite others to do the math. Note that the variation isn't linear, meaning the width of the Earth varies far less from the equator to 45 degrees than from 45 degrees to the poles. StuRat (talk) 17:37, 4 March 2010 (UTC)[reply]

File:Lunar libration with phase2.gif Image removed to reduce page load size. Franamax (talk) 21:21, 5 March 2010 (UTC)[reply]

Note also that the eccentricity of the Moon's orbit (some 42 000 km variation) dwarfs the equator-to-pole variation and the horizon-to-overhead variation (6 300 km). Between that and the Moon illusion (note that said illusion also holds for the sun, which has no appreciable difference in observable size, so there really is a psychological component), I'm not sure the naked eye would actually observe the difference in an equatorial versus England-based viewing. — Lomn 17:47, 4 March 2010 (UTC)[reply]
I posted an gif file that shows the aparent movement of the moon throughout a month (libration) that shows clearly how large is the change in aparent size of the moon due to the eccenticity of its orbit. Dauto (talk) 18:23, 4 March 2010 (UTC)[reply]
Interesting animation. It appears to show the Moon rolling slightly upward over the course of the month, which confuses me since one side of the Moon always faces the Earth. Is this just part of a little wobble, or is this an illusion, too ? StuRat (talk) 18:57, 4 March 2010 (UTC)[reply]
Libration should have all the details, but it's quite real. The Moon rotates on its axis at an even rate, but it orbits the Earth at a varying rate (due to the orbital eccentricity). The net effect is that the Moon alternately lags and leads, showing nearly 60% of its surface over the course of a sidereal month, even though only the same 50% is visible at any given point in a sidereal month. — Lomn 19:03, 4 March 2010 (UTC)[reply]
Is that additional 10% entirely due to libration, or is a small portion also due to having a slightly different view of the Moon from different locations on Earth ? StuRat (talk) 19:26, 4 March 2010 (UTC)[reply]
Some very small fraction is gained by moving around the Earth (or by waiting for the Earth to revolve; same effect), but the vast majority is due to the libration. I'd guess it's a difference of three orders of magnitude, though I haven't tried to work the math. — Lomn 21:53, 4 March 2010 (UTC)[reply]

Great pace - 3 answers in less than 15 minutes! Yeah figured it wasn't likely but wasn't sure of the numbers. Thanks everyone for their input - just came to me on the way home and figured what the hell i'll ask. ny156uk (talk) 17:44, 4 March 2010 (UTC)[reply]

Follow-up question for any who might know: My intuition would be that the size wouldn't change at all due to relative distance (as noted, the delta in the distance is minuscule compared to the total distance), but that it might vary a bit based on changes in the atmospheric lensing. I'm not a physics guy, so I'm probably wrong, but wouldn't the appearance be distorted the further you are from the equator, depending on the season? Perhaps slightly larger in the Northern hemisphere's winter and with a slight disturbance to its shape? Probably not all that observable on a human scale (particularly given the problems with looking directly at the sun), but more measurable than the effect already addressed? Similarly, the shape would change somewhat over the course of the day, since at sunrise and sunset it would be passing through a greater volume of the atmosphere than it does at solar noon? Anyone know if that has an effect greater than, say, a couple millimeters in observed size? —ShadowRanger (talk|stalk) 18:14, 4 March 2010 (UTC)[reply]
Atmospheric effects ca definately deform the observed shape of the sun. Look at that picture from the green flash article. Dauto (talk) 19:37, 4 March 2010 (UTC)[reply]
So presumably for some seasons and latitudes you'll get a visible effect even at midday. After all, above the Arctic circle the sun spends some time effectively permanently rising (or setting, however you want to view it). Still curious if places like London or Manhattan see any such effects (outside of sunrise and sunset), but that at least partially satisfies my curiosity. —ShadowRanger (talk|stalk) 19:52, 4 March 2010 (UTC)[reply]


I've taken the liberty of thumbnailing the images posted in this thread; the rest of the reference desk should be have to be subjected to such large and distracting images. User:Curious Cactus 20:43, 4 March 2010 (UTC)[reply]

Would you believe "should not"? --Anonymous, 01:30 UTC, March 5, 2010.

One further point. Do you notice that the Sun looks smaller in July than in January when it is 3,000,000 miles (5,000,000 km) closer? Then why would you expect to notice when it is 4,000 miles closer due to being seen from a different place on the Earth? --Anonymous, 01:32 UTC, March 3, 2010.

Highway-side notches

There's these weird notches in the pavement that alert a driver to if they are drifting too far to the left or right side of the highway. I want to know how they work, and if we have an article about them. Mac Davis (talk) 17:47, 4 March 2010 (UTC)[reply]

Rumble strips. -- Coneslayer (talk) 17:48, 4 March 2010 (UTC)[reply]
Thanks. I've noticed that if you drive backwards on them no noise is made, while if you drive forward on them, noise is loud. How could they be designed that way? Mac Davis (talk) 17:57, 4 March 2010 (UTC)[reply]
I believe (not 100%) that this is due to the way in which some strips are cut at an angle. If you drive forward, the wheel slowly (relatively speaking) descends the slope, then hits the sharp cut at the far side with the full force of your forward momentum, which jolts both the wheel itself and the attached suspension, creating a louder sound. In reverse, the only impact is the fall off that same ledge onto the slope, so the primary impact is only as strong as the momentum produced by gravity in a fall of an inch or so; your reverse momentum continues with minimal impact.
Of course, when you're going forward, you're usually going faster, so you're going to get more noise in any event, since you rarely go fast in reverse. But for some rumble strips, the slope-and-wall design would account for your experience. —ShadowRanger (talk|stalk) 18:04, 4 March 2010 (UTC)[reply]
Ah, that makes perfect sense. I should have thought of that. It seems like the notches are not very wide compared to the tire circumference though. I don't know if the tire would really go "up" or "down" a slope. Mac Davis (talk) 18:29, 4 March 2010 (UTC)[reply]
Parts of the tire would. They're flexible. So no, usually the tire never reaches the bottom (unless you're really low on air), but it still descends enough to trigger the effect mentioned. —ShadowRanger (talk|stalk) 18:30, 4 March 2010 (UTC)[reply]

Popcorn

My bag of Boston's Lite Popcorn advertised that it was "whole grain" -- what does that even mean? That they don't remove that brown chaff, because it doesn't seem like anyone removed it (other than corn pops, but that's not popcorn). DRosenbach (Talk | Contribs) 18:27, 4 March 2010 (UTC)[reply]

Whole grain means the bran, endosperm and germ of a plant are found in the product. Mac Davis (talk) 18:31, 4 March 2010 (UTC)[reply]
I wasn't asking for the definition of "whole grain" in general, but for its application to corn. DRosenbach (Talk | Contribs) 18:34, 4 March 2010 (UTC)[reply]
It probably means someone in the marketing department had a clever idea. You need the kernel intact in order to trap the moisture to enable popping in the first place. Whole grain is big, so they decided to cash in by mentioning it (despite the fact that it was already the case). At best a "non-whole grain" popcorn might be able to trim a tiny part off the "point" of each kernel (where it connected to the cob) without compromising structural integrity, but I doubt any effort is made to keep or remove it, they just do whatever is most efficient to strip the kernels. —ShadowRanger (talk|stalk) 18:34, 4 March 2010 (UTC)[reply]
So it's a scam -- just as I thought. Thanx! DRosenbach (Talk | Contribs) 18:36, 4 March 2010 (UTC)[reply]
Well, it's a scam if they are trying to imply that their product is any healthier than other popcorn. On the other hand, it might be useful to remind people that popcorn is a whole grain snack, when comparing with other snacks, like pretzels, where some may be whole grain and others may not be. Of course, the good the extra fiber does you probably pales in comparison with the harm all that salt and grease does. StuRat (talk) 18:49, 4 March 2010 (UTC)[reply]
As a good friend of mine says: popcorn is merely a vehicle for butter and salt. Vespine (talk) 23:26, 4 March 2010 (UTC)[reply]
You can get hot-air popcorn poppers that add no fat, and you can refrain from adding salt, and then you have a healthy snack that's as tasty as Styrofoam packing peanuts. +Angr 23:49, 4 March 2010 (UTC)[reply]
Plus, it's cariostatic (and that was a nice take on it, StuRat...never considered that). DRosenbach (Talk | Contribs) 04:54, 5 March 2010 (UTC)[reply]

How intelligent are beavers

Moved from Talk -- Coneslayer (talk) 18:26, 4 March 2010 (UTC)[reply]

How intelligent are beavers? Has anyone done comparative tests? The article on Beaver dams suggests that their dam building is hardwired in but I guess that still makes them the second most intelligent species on the plnet when it comes to building dams, but how do they do in more general things? KTo288 (talk) 18:25, 4 March 2010 (UTC)[reply]

I think your premise is based on a skewed view of the definition of intelligence. Bees and other wasps are able to construct virtually perfect three dimensional hexagonal hive structures, but I don't think that makes them smart. Weaver birds construct ridiculously cool nests...but really, all bird nests are cool. It would probably take you a long time to make one, and if likely wouldn't be as neat and sturdy, and you are using two hands with 5 fingers each, while birds generally do it with a beak. Intelligence has nothing to do with this. DRosenbach (Talk | Contribs) 18:30, 4 March 2010 (UTC)[reply]
Intelligence has more to do with pattern recognition and creation based on learned patterns than following instinctive behavior. Mac Davis (talk) 18:32, 4 March 2010 (UTC)[reply]


It's not hard to see that a certain amount of very simple but 'logical' thought has to go into the logistics of harvesting trees and building a dam. But,what makes you think that other rodents don't have similar cognitive abilities? Just because squirrels don't have an instinctive ability and need to build damns doesn't mean that they couldn't solve very simple problems. Squirrels often come up with clever ways of defeating anti-squirrel bird feeders. Chipmunks store a surprising amount food for the winter.
A lot of that is trial and error, and not planning things out ahead of time, but it's not like beavers don't make mistakes.
Sometimes beavers fell trees and then later realize they can't get them to the dam site. (I wonder if they're capable of feeling frustration.) Sometimes they start a dam in one place and then abandon the site and build in a slightly different place on the same stream. Sometimes they use a rock or tree as a major structural piece that isn't really tied down well at all.
What makes beavers seem so outstandingly intelligent is their determination and hard work. They make lots of screw-ups, but they keep at it until they get it right. Then you come along in the morning and see their completed dam and imagine that they built it right the first time. APL (talk) 19:37, 4 March 2010 (UTC)[reply]
Our article North American Beaver unfortunately does not refer to the intelligence of the species, but does note that in one experiment, researchers placed a cassette tape player, playing the sound of running water, in a field, and the beavers ran over and covered up the tape player with mud and wood to try to make a dam. I believe the editor who introduced this information may have held the view that beavers are idiots. Comet Tuttle (talk) 18:34, 4 March 2010 (UTC)[reply]
It is sometimes difficult for us to distinguish between intelligence and instinct. Spider webs also seem amazing, but they are apparently made via a complex set of instinctual actions, as opposed to higher thought and problem solving ability. A more obvious case is birds that "talk". At first it might seem that they are intelligent and holding a conversation, but in short order you realize they just repeat sounds they hear without any knowledge of what they mean. StuRat (talk) 18:43, 4 March 2010 (UTC)[reply]

Intelligence is task-specific, so your inquiry cannot really produce any useful results. Suffice to say that beavers can respond to varying conditions fairly well. My dad works as a civil engineer, and there are often issues with beavers causing problems in his municipality. They can make do in a semi-urban landscape, until wildlife control steps in. Vranak (talk) 18:51, 4 March 2010 (UTC)[reply]

Thanks for the responses (sorry for asking in the wrong place). From the responses above I guess I'm concentrating too much on the results of what they do rather than how they do it.KTo288 (talk) 21:46, 4 March 2010 (UTC)[reply]
Not very. I reckon, in a game of poker, you could easily lick a beaver. —Preceding unsigned comment added by 79.76.171.183 (talk) 22:27, 6 March 2010 (UTC)[reply]
But in a game of Monty Hall, pigeons beat humans. (ref: news) Imagine Reason (talk) 22:54, 6 March 2010 (UTC)[reply]

PAD vs PADI (peptidylarginine deiminase)

Reading some papers on peptidylarginine deimmanses has gotten me into a fluster! The enzyme citrullinates arginine to citrulline with some interesting consequences, in most the literature the enzymes are abbreviated to PAD, a few refer to them as PADI. However a few papers imply that the PADI is the gene sequence that encodes for PAD.

Some clarity on the matter would be appreciated as I don't want to be refering to genes as if they were proteins...

129.31.206.253 (talk) 19:22, 4 March 2010 (UTC)[reply]

I should add the wiki pages aren't helpful, titles refer them as PADIs whereas references refer to them as PAD MedicRoo (talk) 19:26, 4 March 2010 (UTC)[reply]

Usually gene names and protein names are the same, with the gene name in italics and the protein Capitalized. However, sometimes there are nomenclature differences between species, e.g. human and rodent. Searching at the Entrez database, shows Padi3 in rats and PADI3 in humans. Pad3/PAD3 are listed as synonyms. I suggest that the PADI vs PAD is probably due to either some minor disagreement amongst researchers or different names in different species. Also, HUGO is the human gene nomenclature database. Similar results can be found there. -- Flyguy649 talk 19:38, 4 March 2010 (UTC)[reply]
PAD doesn't seem like a good abbrev, to me, since it's also an abbreviation for so many other things, including at least one item in the medical field: Peripheral artery disease. So, if a medical journal has a title "New research on PAD", which do they mean ? StuRat (talk) 19:57, 4 March 2010 (UTC)[reply]
If you think that's bad you should check out APC, specifically the four distinct meanings in molecular biology/biochemistry.131.111.185.68 (talk) 23:32, 4 March 2010 (UTC)[reply]
StuRat, it is a minor problem since identifying which protein is the subject of the article is usually apparent from other words in the title. If that doesn't make it clear, the abstract will. Clicking on an article found in Pubmed goes to a page with the abstract and a link to the article. Also using an advanced search can help limit results. For example, I used to read papers discussing AP2. The one I was interested in is involved in vesicular trafficking; the other is a transcription factor. I could usually tell by the title which it was. (Both those articles are terrible, by the way). -- Flyguy649 talk 16:30, 5 March 2010 (UTC)[reply]
But isn't the cost of that extra character worth not having to add more search parameters or read the abstract to tell if you have the right one ? StuRat (talk) 17:57, 5 March 2010 (UTC)[reply]
I know what you mean, but it's hard to do in practice. Which one retains the old shortform? I think these problems are less likely to happen with new genes, since every gene now has a unique nomenclature and journals generally require the use of official nomenclature. But genes and proteins that predate genome sequencing are somewhat "grandfathered". Scientists are very resistant to change... traditionally, naming rights go to the earliest publication. But if two labs come up with different names at the roughly the same time, it can become a bit of a pissing contest and it may take years for one form to become dominant. Also, it happens on occasion that scientists don't realize that they are studying the same protein or gene, so there are multiple names for that reason. Or independent nomenclature develops in different species. Sometimes unification works -- e.g. smad proteins is an amalgamation of mad (Drosophila melanogaster) and Sma (Caenorhabitis elegans). Then there are no real rules preventing a gene name from being the same as a short form for a disease or other acronym/short form. Reviewers will likely discourage authors from coining an acronym/shortform if there's an existing well-known use of the same. I guess the short answer is that logic doesn't really prevail in reality! -- Flyguy649 talk 21:25, 5 March 2010 (UTC)[reply]
I take it that biochemistry lacks something equivalent to the International Astronomical Union, which resolves such naming and classification issues in astronomy. I'm glad astronomers are better organized, or we might still call the planet Uranus by it's original name, George. StuRat (talk) 22:12, 5 March 2010 (UTC)[reply]

Unknown beetle that looks like a large ladybug

I found a beetle on my wall today, and am trying to determine what it is. Unfortunately I don't have a picture because, as I was watching it, it attempted to fly away but had a fail and fell down behind something. Now I can't get to it. It looked just like a ladybug, however, except larger. It appeared to be about 1 cm in diameter, shaped like a half-sphere(ie, not a flat insect). It had the second pair of flight wings like ladybugs and many other beetles do. I didn't get a good look at what its head looked like due to the angle it was at and how large the round "shell" wings were, and when I tried to walk closer it fell down, but I believe there may have been long antennae present. Does anybody know of any beetles that resemble this? —Preceding unsigned comment added by 69.243.51.81 (talk) 20:24, 4 March 2010 (UTC)[reply]

You'd have to give your physical location to get a useful answer. There are millions of insect species, most of which are confined to extremely limited ranges. —ShadowRanger (talk|stalk) 20:37, 4 March 2010 (UTC)[reply]
If it looked like a large ladybug, it probably was. It's size just means it's engorged on aphids, a normal part of the ladybugs life cycle at this time of year. User:Curious Cactus 20:40, 4 March 2010 (UTC)[reply]
Ladybugs -- Coccinellidae -- are beetles. 1 cm in size is at the upper end of their size range, but there are species that reach 1 cm size. There are also other beetles with approximately "half sphere" body shape; for example, some Chrysomelidae and a number of water beetle species/genera from several families. --Dr Dima (talk) 21:27, 4 March 2010 (UTC)[reply]
The OP never denied that ladybugs are beetles. In fact, he confirmed it when he wrote "ladybugs and many other beetles". +Angr 21:32, 4 March 2010 (UTC)[reply]
I'm from maryland, so about halfway up the eastern coast of the usa. When I looked up ladybugs, most of the sources I found said that they didn't usually grow past 5mm, so finding one twice that size was what made me think it must be a different species. 69.243.51.81 (talk) 22:31, 4 March 2010 (UTC)[reply]
There are very many different species of 'ladybugs' or Coccinellidae beetles (5,000+ worldwide, 450+ in North America), and some are indeed up to 10mm in diameter. It's probably impossible for us to be sure what this one was without a good picture, but Harmonia axyridis (aka the Asian lady beetle, Japanese ladybug or Harlequin Ladybird) seems like a good candidate. 87.81.230.195 (talk) 14:19, 5 March 2010 (UTC)[reply]
This is the conclusion my friend and I came to last night. She has those kind in her house, and she routinely sees them as large as I saw the one yesterday, so we think it may have come home in my bag from her house. Thanks for your help. 69.243.51.81 (talk) 16:55, 5 March 2010 (UTC)[reply]

Is there any life on the planet Earth?

If we were looking at Earth orbiting a star, is there anything about it which would indicate that it had life? How close would you have to get to detect life? 92.29.76.62 (talk) 20:52, 4 March 2010 (UTC)[reply]

You could detect radio signals from earth that would show there was people with that technology. User:Curious Cactus 20:58, 4 March 2010 (UTC)[reply]
There's also an argument that the mere presence of abundant atmospheric oxygen would indicate life. The lack of oxygen wouldn't make life impossible, but according to Neil deGrasse Tyson on the Colbert Report a few days ago, oxygen is too prone to reacting with other elements to remain stable without living things to produce it (e.g. cyanobacteria). I doubt this theory is universally accepted, but it's another opinion on the matter.
Your question does require some parameters though. How close are the observers? Martians could see the light from cities on our night side and deduce it was artificial. How technologically advanced are they? We're improving our ability to examine distant stars all the time; as recently as a few years ago we couldn't actually detect planets outside our solar system, now we can detect particularly large ones and even divine some of their properties (mass, distance from their star, certain facts about their atmosphere, etc.). —ShadowRanger (talk|stalk) 21:13, 4 March 2010 (UTC)[reply]
It's tough to guess how the Martians would think. If the Martians don't know about aerobic life, and instead regard oxygen as a deadly poison, it might not immediately occur to them that high oxygen levels were an indicator for life. Their scientists might spend years debating what sort of unusual geologic process is causing our high oxygen levels. APL (talk) 22:07, 4 March 2010 (UTC)[reply]
Martians could also see the greening of each hemisphere's temperate region when it's spring occurs. They might not immediately think of photosynthesis and chlorophyll, but maybe they would. StuRat (talk) 22:32, 4 March 2010 (UTC)[reply]
I won't say it is impossible, but I think it is unlikely one could see terrestrial city lights from Mars. The total luminosity of civilization, as viewed from Mars, would be less than 1/1000th of a bright star, and you'd have separate that from reflected sunlight and moonlight. That would be a technically challenging thing to resolve directly. Dragons flight (talk) 06:28, 5 March 2010 (UTC)[reply]
1/1000 corresponds to about 7.5 magnitudes. I'm not sure what you mean by "a bright star" exactly, but you must mean at least magnitude 1, I sure, so we're talking no dimmer than magnitude 8.5. That's easily visible with a human eye looking through binoculars. If you look when Mars is at opposition and it's a new moon, the Earth's disc should be completely dark (except near the limbs where some light will be refracted by the atmosphere) and seeing down to mag 8.5 through binoculars shouldn't be at all hard. With a decent telescope, you would have no difficulty at all. --Tango (talk) 06:39, 5 March 2010 (UTC)[reply]
In response to Cactus: Intelligent life is different from "life". Radio signals might give the game away, but the question of how to detect low complexity life is quite different. —ShadowRanger (talk|stalk) 21:24, 4 March 2010 (UTC)[reply]
Of course detection of intelligent life is one indication of "life". My understanding of the state of SETI technology is that it is unlikely that even the most advanced receivers we use today could pick up our own transmissions, with the possible exception of our highest powered military radars, were they being emitted from as far away as the next to nearest star. This was mentioned in a Google Tech Talk on SETI, but I would like to find a better reference and add it to the SETI article. The article currently says: Furthermore, the earth emits considerable radio radiation as a byproduct of communications such as TV and radio, and these radiations would be easy to recognize as artificial due to their repetitive nature and narrow bandwidths. If this is typical, one way of discovering an extraterrestrial civilization might be to detect non-natural radio emissions from a location outside our solar system. I think most people believe it would easy to pick up Earth's television emissions from anywhere within a 73.5 light year radius. 124.157.247.225 (talk) 00:42, 5 March 2010 (UTC)[reply]
With regards to people asking about the observers; the OP is specifying if we could detect that life. So, I think we can assume the observes are oxygen-breathing, and at a technology level equal to ours. The question of distance is still unanswered, though. Vimescarrot (talk) 22:29, 4 March 2010 (UTC)[reply]
If we were looking at a duplicate Earth, the basic answer is that we could identify intelligent transmitting life from within a radius of a few dozen light years(maybe even a few hundred, depending on the power and directionality of the broadcast) if we were looking in the right direction, but we probably couldn't detect the planet itself if it wasn't in our solar system. If the life isn't using electricity, we'd be blind to anything outside the solar system.
In the next few years (once our interstellar planet detection technology improves a bit so we can detect Earth-sized planets and determine the composition of their atmosphere accurately), we could probably identify the presence of an Earth-sized world with an oxygen atmosphere in a our immediate stellar neighborhood (a few dozen light years in any direction) which would indicate, but not prove the existence of life. —ShadowRanger (talk|stalk) 23:00, 4 March 2010 (UTC)[reply]
Life was detected on earth with the Galileo and LCROSS probes, but of course these probes were quite close (well within our own solar system). -- Coneslayer (talk) 12:40, 5 March 2010 (UTC)[reply]
It is worth mentioning that the most sensitive radio telescope in the world is not sensitive enough to detect a broadcast transmission from the most powerful radio transmitter in the world at the distance of the nearest star (4 light years). Unless a radio signal is being beamed in a very tight beam, it's unlikely to be detectable across interstellar distances assuming the people doing the listening and transmitting are at roughly the same level of technological development as humans. SteveBaker (talk) 20:11, 5 March 2010 (UTC)[reply]
Well, but here you're talking about a single transmission, rather than the whole sea of radio emissions from the whole planet, right? I don't know whether he was correct, but I recall a claim by Larry Niven that the Earth emits as much (artificial, I think) radio flux as a small star, and I think we do see small stars in the radio sky. So on the (admittedly shaky) basis that both these statements are correct, we might hope to detect that there is intelligent life via radio observation, even if we can't resolve a particular message. --Trovatore (talk) 20:26, 5 March 2010 (UTC)[reply]
See our article Extraterrestrial_life. --Kvasir (talk) 20:36, 5 March 2010 (UTC)[reply]

Council house with air conditioning

A council house near me has what looks like air-conditioning. The house is still owned by the council. The climate in the UK does not require air-conditioning. What could explain this? Thanks 92.29.76.62 (talk) 20:56, 4 March 2010 (UTC

Isn't it obvious? The previous residents must have had it installed. Maybe they just had spare money and decided they wanted to try it. Also, the UK does have a climate that would warrant air conditioning in the summer. Especially since global warming it getting worse and worse. User:Curious Cactus 21:02, 4 March 2010 (UTC)[reply]

Methinks you do not know what a council house is. It was installed quite recently. 92.29.76.62 (talk) 22:04, 4 March 2010 (UTC)[reply]

What kind of air conditioning? Window units or central air? If window units, are they present in every single flat? If they're only present in some, then I'd guess the tenants had them installed at their own expense. I agree it's unlikely a city government would pay for air conditioning for all council house tenants in a climate like the UK's. On the other hand, you only said it looks like air conditioning. Is there anything else it could be? +Angr 22:15, 4 March 2010 (UTC)[reply]
A/C doesn't only lower temperatures, it also lowers humidity, and you do sometimes get high humidity there, as on a foggy day. (There are also dehumidifiers, which only lower humidity.) While I doubt if anyone in the UK would die from a lack of A/C, they could certainly be more comfortable at certain times of year with it. Now the question is whether they should pay, at taxpayer expense, to make poor people more comfortable. Some would say yes, others would say no. I would tend to answer differently depending on why they are poor. If they are a drug addict who can't hold a job, then hell no. If they are a paralyzed veteran who was wounded in combat, then yes. StuRat (talk) 22:21, 4 March 2010 (UTC)[reply]
Re people dying from lack of A/C: Newspapers do report old people dying of heat in un-air-conditioned homes during hot summers, actually. I don't know if they specified exactly what the cause of death was. Vimescarrot (talk) 22:26, 4 March 2010 (UTC)[reply]
Vimescarrot: Heatstroke. Ks0stm (TCG) 22:37, 4 March 2010 (UTC)[reply]
You only get fog on about two or three days in winter - it is clear that you are uniformed about the matter that you are taking about. 92.29.76.62 (talk) 23:21, 4 March 2010 (UTC)[reply]
Well, if you can't specify the location any more precisely than "UK", you can't expect us to know the weather very precisely, either, now can you ? StuRat (talk) 03:09, 5 March 2010 (UTC)[reply]
The above applies to all of England and Wales, and probably most or all of Scotland. 78.151.93.38 (talk) 14:26, 5 March 2010 (UTC)[reply]
So, no place in the UK gets fog more than 3 days a year, and always in winter ? I don't believe that. StuRat (talk) 15:01, 5 March 2010 (UTC)[reply]
It would be unusual to have more than three foggy days a year, or to have a foggy non-winter day, exccept perhaps in late autumn. I know from other North Americans that people who've never been here believe the UK is full of swirling fogs - an idea I suppose they've got as a legacy of old Sherlock Holmes films, and that we're an island. And no, we don't get fogs in summer, LOL! You may be using San Fransisco as an incorrect analogy - I understand they gets lots of fogs there due to that climate and topography, which we do not share. 78.146.208.26 (talk) 20:18, 8 March 2010 (UTC)[reply]

Great! Now my question is completely ruined and I will never get a sensible answer but several paragraphs of inevitable bickering and worse-than-useless uninformed speculation! 92.29.76.62 (talk) 22:47, 4 March 2010 (UTC)[reply]

Welcome to every day on the Wikipedia Reference Desk. Enjoy your stay. User:Curious Cactus 22:52, 4 March 2010 (UTC)[reply]
In your question, you basically asked us to speculate on something we have no experience of. We haven't seen the house you're talking about, nor are we in charge of what appliances are installed in it. Unless you provide more information (like answering the questions I asked above, or taking a photograph of the building and posting it here), uninformed speculation is all you can hope for, because quite frankly, it's all your question in its current state can possibly be answered by. +Angr 23:27, 4 March 2010 (UTC)[reply]
It is customary and normal etiquette to only answer questions you have knowledge of, rather than just guessing from several thousand miles away. 84.13.22.99 (talk) 00:57, 5 March 2010 (UTC)[reply]
Yes but clearly most of the respondents did feel they have sufficient knowledge to be able to offer informed speculation. There's no requirement you actually live in the UK to answer a question concerning the UK. As it stands, the OP still hasn't provided any information to ascertain whether it's possible the tenant installed these airconditioning units which is surely a key point. It's customary and normal etiquette to accept it when people are trying to help regardless of whether you agree with the answer particularly when the question calls for extensive speculation because of the nature of the question and the failure of the questioner to provide any real information even when asked. I would add that I believe VimeCarrot lives somewhere in the UK & Angr lives in Germany, so not 'several thousand miles away'; and I have no idea where User:Curious Cactus lives, do you? Also for all the complaints of pointless bickering, part of the reason this question has gotten offtrack is because of the OP's complaint. Nil Einne (talk) 01:57, 5 March 2010 (UTC)[reply]
Are you talking about a window A/C, or a central A/C?. I'm not from the UK, but if it's a window unit, I don't see what's wrong with Cactus's answer. They're cheap and easily installed, someone could have done so on a whim, or because they're fussy or medically sensitive to temperatures or air quality, or whatever.
Central A/C might be more of a mystery, if you're sure that's what it is. In a commercial building I would just assume that there was some technology in the building, like a server-room, that needed A/C, but that doesn't work in a residence. APL (talk) 02:24, 5 March 2010 (UTC)[reply]
I was reluctant to say this given the OP's complaints but screw it. It's perhaps worth remembering that many A/Cs can function as heatpumps and would probably be the most efficient form of electrical heating. Here in NZ, they're extensively promoted as such given that many of our houses have extremely shit insulation and many places lack gas heating it probably makes sense. Of course many may use them in the summer as well and this is useful in quite a few parts of the country like Auckland. I appreciate things are different in the UK, but it seems easily possible that the same thing would be an added incentive for some people to have them installed. This ref [2] says they're starting to become standard in new urban developments for example. Nil Einne (talk) 02:41, 5 March 2010 (UTC)[reply]
How sure are you that it is a council house and not an ex-council house? The right to buy scheme has resulted in a very large number of council houses now being in private hands. --Tango (talk) 04:49, 5 March 2010 (UTC)[reply]

Its absurd that people from the other side of the Atlantic who may never even have set foot in the UK in their lives or even know what a council house is should believe that they can and should answer a question about a UK council house. 89.242.242.77 (talk) 10:04, 5 March 2010 (UTC)[reply]

I don't understand why you are so bemused by this. Occupants of council houses have great flexibility in making improvements to them, although the council can insist that it they are kept in good order, and that changes are reversed when the occupants leave. If the current occupants plan on staying for the long-term, they might well decide it is worth installing air conditioning (depending on the system, it may also be possible to take it with them if they move). While most occupants of council houses are on low incomes, it isn't essential; they can stay on if their income increases, and in the few areas where there is a surplus of council housing, they are readily available to anyone. Warofdreams talk 11:50, 5 March 2010 (UTC)[reply]
Oh no! Government-owned, subsidized housing is completely unknown out here in the Rest Of The World, How will we ever answer your question about why a residential unit might need a piece of equipment that looks like HVAC unit??!?!?!?
Ok, I think I've figured out the answer you're fishing for, Here you go : "Because the [Political Party] is pandering to the lazy and shiftless poor, and needlessly wasting your tax dollars providing unnecessary luxuries to the [ethnic group] poor!" Does that help? APL (talk) 15:25, 5 March 2010 (UTC)[reply]
Nearly all the people I've met who live in a council house have been foul mouthed ignorant thugs with chips on their shoulders who hate middle-class people. Although that does not so much apply to expensive areas like London, where middle-class people like Vivienne Westwood often exploit the cheap accomm. 78.146.208.26 (talk) 20:44, 8 March 2010 (UTC)[reply]
A tangental query, but what areas have a surplus of council houses please? I thought there were long waiting lists for them. Thanks 78.151.93.38 (talk) 14:07, 5 March 2010 (UTC)[reply]
Certain deprived areas in the north and Scotland, which have a lack of demand for housing in general, have surplus council housing. Here, for instance, is a recent document from West Dunbartonshire Council noting that they have around 350 council properties with no demand, while here is something similar from Salford. Of course, it may be that these houses are run down, and with a little investment would be in demand - describing them as "surplus" might well be a way to justify selling them or demolishing them and selling the land, to make the council some quick cash. Warofdreams talk 15:00, 5 March 2010 (UTC)[reply]
If I was in charge I would demolish half of them and use the land to create parks and mini-nature reserves, and thus turn them into attractive areas people want to live. 78.146.208.26 (talk) 20:44, 8 March 2010 (UTC)[reply]

The what-looks-like air conditioning is a large metal box on the ground within a few inches of the wall of the house containing a large fan, with pipes from it going into the house. It is painted exactly the same colour as the walls of the house, suggesting it was installed at the expense of the local government who own the house. I wonder if it might be required for something like a kidney dialysis machine, but in the past they required an extra room to be built on to the house to contain the machine, and there is no extension on that small house. Perhaps those machines have got smaller in recent years. Council houses are lived in by people who are poor, particularly in that part of the country, and I am sure they would not buy air conditioning on a whim as they are not needed here. The house is still definately owned by the local government. I think its practically impossible that its some sort of cooling equipement for computer servers, as the local council would not give permission, and the people are very likely to be poor, uneducated, and irresponsible. 78.151.93.38 (talk) 14:24, 5 March 2010 (UTC)[reply]

Wow, lots of stereotypes there. There are many legitimate reasons why responsible, educated people can find themselves to be poor, such as divorce (and thus needing to support two households on an income that formerly only supported one), illness (making work impossible), etc. And perhaps the people in that house have slightly improved their financial situation, enough to afford A/C, but perhaps not enough to afford better housing. If they needed to get permission from the council to install the A/C, one of the requirements might well have been that they make it blend in as well as possible, including matching the paint color.
Modern dialysis machines are completely self contained and require no external cooling. There could also be a medical necessity for A/C, such as if the person is morbidly obese and thus more prone to heatstroke. Or, they could have severe allergies which prevent them from cooling the house by opening windows. I suggest you contact the council directly, and ask them for the details. StuRat (talk) 15:18, 5 March 2010 (UTC)[reply]
Air_source_heat_pumps would be a possibility - these are being promoted as a more efficient replacement for existing electric storage heater or oil heating systems (typically when mains gas is not available). As such the local council could easily have installed the system (grants are also available from central government under energy efficiency programmes). From a UK resident. 94.197.138.254 (talk) 18:28, 5 March 2010 (UTC)[reply]

energy from earth's rotation

i'm sure it must be unfeasable, but can't see exactly: why can't we derive power from the angular momentum of the earth? 109.246.247.147 (talk) 20:58, 4 March 2010 (UTC)[reply]

See? Do you mean Sea? User:Curious Cactus 20:59, 4 March 2010 (UTC)[reply]
corrected to we 109.246.247.147 (talk) 21:17, 4 March 2010 (UTC)[reply]
We can derive power from the rotation of the Earth, that's what tidal energy is. Tides are produced by the rotation of the Earth relative to the Moon and, to a lesser extent, the Sun. StuRat (talk) 21:01, 4 March 2010 (UTC)[reply]

this is correct, but doesn't quite answer the question i had in mind, which is why is it not possible to transfer momentum from the earth's rotation directly into torque or some other means of driving a earth-based engine? 109.246.247.147 (talk) 21:17, 4 March 2010 (UTC)[reply]

Virtually all power generation is based on energy level differentials. In order to derive power *directly* from the angular momentum of the Earth (as opposed to indirectly through the tides), you'd need to find a way to connect it to something not experiencing that momentum. And anything you attached it to would be giving up its kinetic energy to provide the power; deriving energy from the Moon for instance would cause a gradual orbital decay (and/or reduce the rotational speed of the Earth). The tides (and tidal power) are doing basically the same thing, it's just on a scale so low that it has minimal effect on our own rotation and the Moon's orbit. Of course, I'm not a physicist, so I'm sure I made some minor errors in my explanation, but I believe the basic point is sound. —ShadowRanger (talk|stalk) 21:20, 4 March 2010 (UTC)[reply]
The real reason is angular momentum conservation. BTW the moon's orbit is actually rising, not decaying. Dauto (talk) 21:30, 4 March 2010 (UTC)[reply]
I always thought that an orbit can "decay" either upwards, until the object leaves orbit, or downward, until it crashes. StuRat (talk) 22:24, 4 March 2010 (UTC)[reply]
Obviously, since the Earth is slowing down. The moon takes up much of that energy (another part is dissipated as heat, and yet another part should actually lift the Earth into a higher orbit around the sun...). --Stephan Schulz (talk) 21:34, 4 March 2010 (UTC)[reply]
I would say true, but not obvious.Dauto (talk) 21:38, 4 March 2010 (UTC)[reply]
Hmm. If the moon's orbit decays and the Earth slows down, you need a really big energy sink somewhere. --Stephan Schulz (talk) 21:42, 4 March 2010 (UTC)[reply]
Heat?Dauto (talk) 21:45, 4 March 2010 (UTC)[reply]
Yep. The tidal energy pretty much all turns into heat eventually, either in the water or in the Earth's core, mantle, and crust. StuRat (talk) 22:05, 4 March 2010 (UTC)[reply]
It was a rhetoric question but thanks for answering anyways.Dauto (talk) 03:10, 5 March 2010 (UTC)[reply]
We had pretty much the same question recently about attaching a gear to the earth Earth GearVespine (talk) 21:43, 4 March 2010 (UTC)[reply]
What you want is a 'Gyrogenerator' as seen here in the Museum Of Unworkable Devices. (Second one down on that page.)
Perhaps someone better at crunching the numbers could tell us if this would actually work. (Assuming sufficiently frictionless equipment, of course.) APL (talk) 22:28, 4 March 2010 (UTC)[reply]
I think you are confusing energy and momentum. Steve will be along soon to explain it. --Tango (talk) 05:04, 5 March 2010 (UTC)[reply]

acidosis

what is the treatment for urinary acidosis other than iv bicarbonate which is just a temporary fix. no other problems can be found to cause this. the person is not diabetic. (this is a hypothetical question.. im not asking for the cause but the treatment. —Preceding unsigned comment added by Thekiller35789 (talkcontribs) 21:36, 4 March 2010 (UTC)[reply]

Unfortunately, we cannot recommend treatment options, even in the hypothetical. Please read the Wikipedia:Medical disclaimer. Instead, you should seek, or advise your friend to seek, the advice of a qualified medical professional who can see and treat your friend in person. --Jayron32 22:18, 4 March 2010 (UTC)[reply]

contribs) 22:39, 4 March 2010 (UTC)[reply]

Also see the reference desk guidelines on the matter, which explains what we do and don't answer here specifically. Acidosis is a particularily good example of why we have these guidelines, because there are different possible causes, and the best treatment is often to eliminate the underlying cause. We cannot hope to diagnose the underlying cause of a specific instance in a specific person, and then proscribe the best form of treatment. If someone you know does have acidosis, they should discuss it with a medical professional who has the access and training to evaluate and treat the condition, rather than asking a random group of people on the internet.Buddy431 (talk) 22:28, 4 March 2010 (UTC)[reply]


no one i know has acidosis and neither do i . im just interested in it. it sounds to me like their is no real treatment if no underlying cause can be found ?

This looks like a homework question to me. The answer will be in your text book —Preceding unsigned comment added by 79.68.242.68 (talk) 00:46, 5 March 2010 (UTC)[reply]

There is no disease called urinary acidosis. You may be mixing up renal tubular acidosis, which comes in several varieties caused by many different underlying conditions. Treatments also vary according to the underlying condition. alteripse (talk) 02:03, 5 March 2010 (UTC)[reply]


im 45 years old and not in school, lol. the acidosis im taking about is when your urine has a low ph. —Preceding unsigned comment added by Thekiller35789 (talkcontribs) 02:39, 5 March 2010 (UTC)[reply]

There are uncountable reasons why a person's urine would have an abnormally low pH, and without a proper medical examination to determine the underlying cause, it would be absolutely impossible to recommend a course of treatment for it. That's why we don't answer questions like this; if its idle curiosity Urine#pH contains a very brief discussion of the issue, and articles like Acid-base homeostasis discusses the physiological pH levels in general. You could follow links from there to see what you find. If you have specific concerns about your own urine or the urine of people whom you know, seek a doctor. --Jayron32 06:27, 5 March 2010 (UTC)[reply]
I thought you were a 19 year old who was worried about what would happen to you if your mum died [3]? Incidentally if you don't know anyone with acidosis, why the sudden interest [4]? Nil Einne (talk) 20:02, 6 March 2010 (UTC)[reply]

[1]

zoonoses from dogs

Besides rabies, are there other zoonoses that humans can get from dogs and v v ? Googlemeister (talk) 21:58, 4 March 2010 (UTC)[reply]

Kennel cough User:Curious Cactus 22:53, 4 March 2010 (UTC)[reply]
The kennel cough article says nothing about transmission between dogs and people. Deor (talk) 23:05, 4 March 2010 (UTC)[reply]
Who said that Wikipedia articles are necessarily comprehensive? DRosenbach (Talk | Contribs) 05:01, 5 March 2010 (UTC)[reply]
Wikipedia did! It doesn't say anything about Wikipedia being accurate, though... --Tango (talk) 05:39, 5 March 2010 (UTC)[reply]
What about a barking cough? —Preceding unsigned comment added by 79.68.242.68 (talk) 00:48, 5 March 2010 (UTC)[reply]
A few come to mind:
I'm sure there are others. -- Scray (talk) 06:13, 5 March 2010 (UTC)[reply]
There is at least one other, mange. Dismas|(talk) 16:23, 5 March 2010 (UTC)[reply]

Queen bee naughty bits

Resolved

I can't seem to find a good diagram of queen honey bee anatomy to answer this question for myself. Apparently the stinger in most stinging insects is a modified ovipositor. A queen bee has a stinger, and she posits ova (I don't know if she has an ovipositor as such), so how does that work? Thanks. --Sean 22:12, 4 March 2010 (UTC)[reply]

It does seem odd and I can't find an explanation. Our article, Bee sting says: "In worker bees, the sting is a modified ovipositor. The queen bee has a smooth sting and can, if need be, sting skin-bearing creatures multiple times, but the queen does not leave the hive under normal conditions." This seems to confirm your statement, but does not really explain it. --Tango (talk) 05:12, 5 March 2010 (UTC)[reply]
Why can't it do double duty ? That is, deliver poison, when needed, and eggs when needed. The urethra in mammal males does double duty and the cloaca of many birds, reptiles, etc., does triple duty or more. StuRat (talk) 12:41, 5 March 2010 (UTC)[reply]
Perhaps it does, but the sizes seem wrong. A honeybee egg looks like a 1/2 scale grain of rice, and all the stingers I've ever seen are tiny in diameter, like a stout dog hair. I'm going to try a more specialized forum and will report back here. --Sean 13:57, 5 March 2010 (UTC)[reply]
I bet a queen bee's stinger is significantly larger, and probably also stretches dramatically when producing young, as a vagina does. StuRat (talk) 14:57, 5 March 2010 (UTC)[reply]
Self-answered by following up a ref in Hymenoptera (p. 49 of Introduction to Insect Biology and Diversity): "In the stinging Hymenoptera, the ovipositor no longer functions to deposit eggs; instead, eggs are released from the vaginal opening on the eighth segment." --Sean 21:33, 5 March 2010 (UTC)[reply]
Cool. I've put the resolved tag on this Q. StuRat (talk) 13:25, 6 March 2010 (UTC)[reply]


March 5

Water for Hydrogen Fuel in Deserts (cont.)

I am continuing from my last question.

Who has proposed using an energy store in deserts? I have recently read a book called Solar Hydrogen Energy: The POWER to save the Earth which proposed that hydrogen be used to transport solar energy from deserts to other places. It also proposed that most production of solar energy be in deserts so they won't take up farmland.

The water in oceans is saltwater. Could we extract the hydrogen used to transport energy from saltwater?

Most combustion of hydrogen as fuel would be in places outside deserts, places all over the world. They would be very far away from deserts. Alot of the combustion of hydrogen would be in transports and not in producing electricity so they wouldn't be transported back to deserts.

An Unknown Person (talk) 04:44, 5 March 2010 (UTC)[reply]

There have been lots of proposals to use hydrogen as a storehouse for excess energy production from solar, wind, hydroelectric, etc. It will likely eventually become a common fuel source, however there is an utter lack of hydrogen-fuel infrastructure (basically, no fuel stations exist to sell the stuff; no distribution network exists to get it to the stations, etc.) the way there is with gasoline. So you get a catch-22 with hydrogen as a fuel: No one owns hydrogen-fueled cars because you can't get hydrogen anywhere to fill the tank with, and you can't find hydrogen stations anywhere because there are no customers with hydrogen cars to buy them from. This barrier is what has kept fuel cell vehicles off the road, despite being a feasible means of fueling a car for, oh, 50 years or so. --Jayron32 04:56, 5 March 2010 (UTC)[reply]
That doesn't mean it's impossible. I imagine you could start with solar hydrogen production facilities near desert cities, like Las Vegas and Phoenix, and provide refueling stations in those cities and maybe a fleet of rental cars that use hydrogen. You could eventually expand the network with hydrogen pipelines to cities near the desert, like Los Angeles. It might never be feasible to ship hydrogen all the way across the country to places like New York, however, depending on the prices of other fuels. StuRat (talk) 12:32, 5 March 2010 (UTC)[reply]
The idea isn't to "ship" hydrogen - it's to use it as a storage medium. The cheapest way to "ship" hydrogen is to burn it to make electricity, send the electricity over wires to the destination and use that electricity to make hydrogen at the other end. Sure, it's inefficient - but it's a LOT easier and safer than building hydrogen pipelines or having trucks or trains carry the stuff. SteveBaker (talk) 20:08, 5 March 2010 (UTC)[reply]
You may be right about the advantages of each approach, but the Q was clearly: could "hydrogen be used to transport solar energy from deserts to other places" ? So, that's what I answered. StuRat (talk) 21:24, 5 March 2010 (UTC)[reply]

What does the integral of the position function represent?

That is, suppose we have velocity v'. If we integrate (with respect to time), we get position v + c. What do we get if we integrate again?--70.122.117.52 (talk) 05:06, 5 March 2010 (UTC)[reply]

The integral of the velocity is displacement, not position. That is, if we integrate a velocity function between any two arbitrary points in time, you get the distance traveled over that time. I'm not sure that integrating the displacement function gets you any meaningful physical quality. Mathematically (especially if you have a complicated velocity function) you could repeatedly integrate any function until you get only a constant left, but that doesn't mean that the operation produces a meaningful physical result. --Jayron32 05:18, 5 March 2010 (UTC)[reply]
(ec)The definite integral of velocity is displacement. The OP is talking about the indefinite integral, which would be position if you take the constant as being the starting position. Also, if you repeatedly integrate you don't end up with a constant - that's repeated differentiation. If you repeatedly integrate you end up with some function plus a polynomial (of ever increasing degree). --Tango (talk) 05:32, 5 March 2010 (UTC)[reply]
I don't think the integral of position wrt time has any physical significance. It would have units of length times time (eg. metre seconds), which doesn't correspond to anything useful that I can think of (an angular momentum divided by a force, I suppose, but I can't think why you would ever do that). --Tango (talk) 05:32, 5 March 2010 (UTC)[reply]
It could have a meaning if there is some other property dependent on the position (∫f(x)⋅dt rather than just ∫x⋅dt). Two ideas:
  • A compressed spring. Force is based on displacement, so k⋅distance⋅time is the impulse or momentum-change of the object against which the spring is pushing.
  • Some other sort of energy transfer where the transfer/flux falls off linearly over distance. Total transfer is like k⋅(1-distance)⋅time. Hrm, actually all the specific ones I can think of are squared relationships not linear:( Something involving a dashpot/damper or other particle motion through viscous material I guess.
DMacks (talk) 21:39, 5 March 2010 (UTC)[reply]

Why would you only end up with a constant through iterated integration? Wouldn't that be what happens through iterated differentiation? —Preceding unsigned comment added by 70.122.117.52 (talk) 05:31, 5 March 2010 (UTC)[reply]

Yeah yeah yeah. Stop piling on. --Jayron32 05:34, 5 March 2010 (UTC)[reply]
Two people edit conflicting does not a pile make! --Tango (talk) 05:38, 5 March 2010 (UTC)[reply]
That depends on the size of the people. I have met people who constitute a pile all by themselves. --Jayron32 06:19, 5 March 2010 (UTC)[reply]
Fair point. I don't know about the OP, but I'm pretty skinny - I think you would need at least 3 or 4 of me to constitute a pile. --Tango (talk) 06:24, 5 March 2010 (UTC)[reply]
Neither iterated integration, nor iterated differentiation guarantee that you'll end up with a constant. Even with iterated differentiation, that only happens for polynomials. —Preceding unsigned comment added by 157.193.173.205 (talk) 08:22, 5 March 2010 (UTC)[reply]
The integral of displacement is sometimes called absement - our article on it has been deleted on account of its low notability, but here is a short-but-good explanation of the concept. --Link (tcm) 22:55, 6 March 2010 (UTC)[reply]

Sun

Could a star like the earths sun just collapse in on itself, like due to chaos theory or something? —Preceding unsigned comment added by Jetterindi (talkcontribs) 09:27, 5 March 2010 (UTC)[reply]

No. The sun is not massive enough to become a neutron star or a black hole. In about 5 or 6 billion years it will become a red giant, and then finally a white dwarf surrounded by a planetary nebula. See stellar evolution for more details. Gandalf61 (talk) 09:39, 5 March 2010 (UTC)[reply]
I heard this same thing in grade school about thirty years ago. Don't you mean "4,999,999,970 - 5,999,999,970 years"? Kingsfold (talk) 14:19, 5 March 2010 (UTC)[reply]
Also, "chaos theory" doesn't mean any crazy thing you can think of might happen. It just means that tiny variations in the beginning of an event can have large and unpredictable effects on the outcome. --Sean 14:19, 5 March 2010 (UTC)[reply]
Perhaps the OP is asking whether all the particles in the Sun could move inwards due to random chance (although this could apply to any bunch of matter at >0K). The probability is unfathomably low. --Mark PEA (talk) 11:53, 6 March 2010 (UTC)[reply]

Thinking brain?

The idea that our thinking activity is located inside the head seems very natural, but is it something we have learned, or do we know it because we feel the brain thinking? (like e.g. we feel our belly digesting the food). If we have learned it, when did men first have this intuition, and how did they came to it? --pma 10:07, 5 March 2010 (UTC)[reply]

comment removed. -- kainaw 21:36, 5 March 2010 (UTC)[reply]
You obviously saw it in a fictional movie (if your memory is correct), since a head transplant is totally beyond the abilities of science men. 82.113.121.110 (talk) 11:25, 5 March 2010 (UTC)[reply]
According to the Head transplant article, the procedure has been Performed with limited success on dogs, monkeys and rats.... Mitch Ames (talk) 11:34, 5 March 2010 (UTC)[reply]
even worse, if that was the end of the "experiment", it only proved: a thinking activity was either in the first monkey's head, or in the second monkey's body, and nowhere in the "scientist". But my question is more on the psychological side: how is that we feel that our thinking activity happens inside our head: is it cultural, that is we learn it when we are kids, or is it physiological, that is, we know what part of our body is thinking much the same way that e.g. we know which part of our body is eating or defecating. --pma 11:45, 5 March 2010 (UTC)[reply]
Scientists probably first knew this from people with head injuries, causing brain damage, which led to abnormal thought processes (speech problems, etc.). But people might have thought this intuitively because our perception is associated with our senses, and 4 out of 5 are located on the head alone. This is a rare case where the intuition was right on. One exception seems to be emotion, which instinct told people was in the heart, since it beats faster or slower based on our emotional state. However, emotion, just like logic, actually comes from the brain. StuRat (talk) 12:18, 5 March 2010 (UTC)[reply]
History of neuroscience has some useful information. The ancient Egyptians believed intelligence was situated in the heart; Alcmaeon of Croton (c. 500 BC) is claimed to be the first person to suggest the brain was used for thinking. Galen (AD 129 – 199/217) also did work on brain structure. Although little is known about him, Alcmaeon probably used vivisection and dissection to make his discoveries, e.g. cutting the optic nerve of live animals. --Normansmithy (talk) 12:24, 5 March 2010 (UTC)[reply]

if thez thought intelligence was situated in the heart, what did they think the material in the skull was for? Also, presumably they knew what some of the other organs did because people injured in those organs all suffered in the same way as a result. Didn't a single person in ancient times suffer traumatic head injury that caused brain damage, so it became obvious to everyone that intelligence and cognitive functions are done there? I seriously can't imagine how they could have thought brain-stuff was for anything else than thinking... what did they think it was? 82.113.121.110 (talk) 13:06, 5 March 2010 (UTC)[reply]

The first article linked by Normansmithy above answers your questions directly. Remember that for much of the times under discussion, there were almost no means of communication other than face-to-face, few libraries or other centres of learning existed, and very few people were literate anyway: consequently, most people's opinions could only be based on their own direct experiences. While some did deduce that the brain was the seat of thinking, some thought instead that the brain was merely packing material, and some (following Aristotle's opinion) that its function was to cool the blood which, since the brain does indeed have a very generous blood supply and does indeed create and radiate a significant proportion of the body's heat was, though wrong, not at all unreasonable. 87.81.230.195 (talk) 13:52, 5 March 2010 (UTC)[reply]

The best discussion of this issue that I've seen is in The Mind's I, in two essays called "Where am I?" (by Daniel Dennett) and "Where was I?" (by David Hawley Sanford). I think the basic answer is that we feel ourselves to be located in our heads because that's where our eyes are -- we are vision-dominated creatures. By using remote-vision equipment it is possible to create a very strong sense of being located outside your body in various ways. Looie496 (talk) 17:11, 5 March 2010 (UTC)[reply]

Thany you all! very interesting information. --pma 08:35, 6 March 2010 (UTC)[reply]

Goedel and Wikipedia

Does the Goedel incompleteness theorem imply that insofar as Wikipedia is comprehensive, it cannot be accurate, and insofar as it is accurate, it cannot be comprehensive? 82.113.121.110 (talk) 11:30, 5 March 2010 (UTC)[reply]

Gödel's incompleteness theorems deals with questions about the fundamental definitions of maths. It is quite a stretch to apply it to wikipedia, without taking more poetic licence than is normally allocated to pure maths. I think it is nonsense to suppose that wikipedia will ever be complete (even if we knew what that meant in wikipedia terms) not completely accurate. We can duduce all of that without the need to appeal to Gödel. --Tagishsimon (talk) 11:43, 5 March 2010 (UTC)[reply]
Wikipedia will not be complete until it contains an article that is about itself. Looie496 (talk) 17:02, 5 March 2010 (UTC)[reply]
Please see our article Wikipedia, which is both (IMHO) accurate and comprehensive. So there you go - Goedel disproved, once and for all. Phew! --NorwegianBlue talk 19:15, 5 March 2010 (UTC) [reply]
Gah, lexical scope ambiguity! I didn't mean an article about Wikipedia, I meant an article with a title something like This article. Looie496 (talk) 19:40, 5 March 2010 (UTC)[reply]
Gödel's theorem relates to provability in a formal axiomatic system. It has nothing whatever to tell us about completeness (or truth) in the phenomenal world. --ColinFine (talk) 17:32, 5 March 2010 (UTC)[reply]
The problem is the definition of "comprehensive".
  1. To truly contain all knowledge (and thereby to be complete) we would have to carefully document (for example) the precise position of every object in the universe and keep that information up to date on a moment-by-moment basis. For truly comprehensive information, we'd require to provide the location of every fundamental particle. The required data storage for such a system would be larger by far than the universe itself - and is therefore quite impossible. This is a kind of "diagonal" argument that Godel and Cantor would have been happy to provide...but it's unrelated to Godel's famous theorem. Hence, true "completeness" is indeed impossible...not just impractical.
  2. Fortunately, Wikipedia's own definition of "comprehensive" requires us only to describe things that are "notable" and "referenceable". It probably is theoretically possible to be fully comprehensive within that definition. If every human on earth were to sit down and write articles about every single notable thing that happened to them personally, or which they had written about in a document that Wikipedia would accept as a reference - then they could easily do it within their lifetimes. Most people would be done with it very quickly...and those people would have plenty of time to fill in the notable/referenceable information from past generations...especially since the world population is much larger than it was in previous generations. Doing that would result in a complete, "comprehensive" encyclopedia (within our own definition of that word) in rather a short period of time.
SteveBaker (talk) 19:42, 5 March 2010 (UTC)[reply]
First, Wikipedia isn't written in a formal language, so Godel's Incompleteness Theorem doesn't apply. If it were, it would be more like a big list of facts ("1+54 = 55" "7 is prime" "65=65") than a small set of axioms (like the nine Peano axioms, that start "for all x, x=x"). Incompleteness is about the limits of derivable facts. Any list of facts (about an interesting system) is trivially incomplete by virtue of not being infinitely long (but an infinitely long list of facts totally could be complete.) Paul Stansifer 23:08, 5 March 2010 (UTC)[reply]
Exactly. For example true arithmetic is complete. True arithmetic satisfies all the stipulations for incompleteness, except the one saying that it can be axiomatized by a computably enumerable set of axioms.
Basically the whole endeavor of trying to apply incompleteness outside of mathematics is fraught with pitfalls. That's not to say it can never be done, but it is to say that some very smart people have wound up making unsound arguments of this sort. Torkel Franzén has a whole book on the subject. --Trovatore (talk) 23:15, 5 March 2010 (UTC)[reply]

do we benefit in any way from c?

Is there any benefit for anyone from having a current universal speed limit of c? ie, if the Universe were a democracy, and its lawmaker obeyed the will of the people in it, is there any reason we would do well to have the lawmaker keep c? At least two good reasons for abolishing c from the laws of the universe would be: 1) easier intergalactic space travel, easier communication with probes on mars, etc, 2) smaller ping times to China, and faster processing within a single piece of electronics. In fact, in today's 3 GHz processors, I heard that electrons only have enough time between clock ticks to travel a few centimeters (you can verify this for yourself with a simple Google calculation). Now this means we can't possibly make processors much bigger than that, with logic that has electrons travelling far more than that distance, depending on what path they take. So, this might be a simplification, but abolishing c could also bring better computation power. But as I asked above, my question is now: is there ANY benefit at all from actually having c? 82.113.121.110 (talk) 12:41, 5 March 2010 (UTC)[reply]

Your question is nonsense. c is a fundamental constant. Okay, technically there are some limited theoretical circumstances where it bends (shortly after the Big Bang), but we have no control over it. Nature's laws are not up for a vote. —ShadowRanger (talk|stalk) 13:13, 5 March 2010 (UTC)[reply]
In English, when you say "if the Universe were", the were means that the speaker knows he or she is asking about an imaginary case that is contrary to actual reality. (Otherwise he or she would say "if the Universe turns out to be" or even "if the Universe really is"). So, your point is totally invalid, instead please answer my question: is there any benefit to c, and even though we can't, if we could petition to have this law abolished, would there be any practical benefit to us if we could and we did and it were? —Preceding unsigned comment added by 82.113.106.97 (talk) 13:22, 5 March 2010 (UTC)[reply]
There are obvious benefits, as you listed. The question is whether the universe could still exist (in any way still somewhat pleasant to us) if we raised c. After all, it's not a speed limit in isolation of the rest of physics. Fine-tuned universe addresses variations on the question. --Sean 14:32, 5 March 2010 (UTC)[reply]
Thanks. Unfortunately your benefit to c in that without it perhaps the Universe "couldn't exist in a way still pleasant to us" is so very broad. Could you, or anyone else, possibly give a more direct, narrow benefit to having c as a speed limit? Does this actually help in a practical, specific (rather than an overarching universal) way? Thanks. 82.113.121.103 (talk) 14:44, 5 March 2010 (UTC)[reply]
The slow speed of light has so far prevented the evil Zorn empire from invading from their galaxy and eating our brains. :-) StuRat (talk) 14:42, 5 March 2010 (UTC) [reply]
But, seriously, there may well be (or have been) other intelligent species out there which would, at some point, have colonized the Earth if they could only get here, meaning we may never have existed. We don't need to attribute evil motives to them, as they might not have found any intelligent life if they arrived long ago, but maybe they wondered what that slime was in those volcanically heated ponds. StuRat (talk) 14:48, 5 March 2010 (UTC)[reply]
This is exactly the type of answer I'm looking for. Do you know of any other answers of this specific nature? 82.113.121.103 (talk) 14:44, 5 March 2010 (UTC)[reply]
Perhaps next time, you could specify which type of answer you are looking in front, and save us and yourself some time? :-) DVdm (talk) 14:53, 5 March 2010 (UTC)[reply]
um, I didn't have any idea of this answer. I like it because it is specific, and OBVIOUSLY a benefit. I would easily pay any amount of my money not to have my brain eaten by Zorns. Are there any other, specific benefits like this you can list? 82.113.121.103 (talk) 14:58, 5 March 2010 (UTC)[reply]
George Gamow wrote a little book "Mr. Tompkins in Wonderlandwherein the Mr. Tompkins dreams about a world where the speed of light was 30 miles per hour. Relativistic effects are seen when someone rides a bicycle. Surely someone has written a similar work where c was orders of magnitude higher. Radio antennas would get bigger, at least for the same frequency. Optics might have to change their size, at least to focus the wavelengths used presently in vision and photography. I wonder if electron orbitals would have to change, along with the the size of atoms and molecules? Edison (talk) 15:03, 5 March 2010 (UTC)[reply]
Changing c would change the fine structure constant, and Fine-structure_constant#Anthropic_explanation suggests that that would result in a very different, and probably inhospitable, universe. Also, if you are talking about an infinite c (instead of just a larger c), I'm not sure you would have electromagnetic waves at all anymore. -- Coneslayer (talk) 15:11, 5 March 2010 (UTC)[reply]

Okay, the above is a real benefit: smaller radio antennas than if C were larger. Then Cones Layer says the same thing others have been saying, that it's a general requirement for our whole universe. Guys, I got this part. Are there any other specific benefits to the current c, as the bit about smaller antennas is? Thanks. 82.113.121.103 (talk) 15:38, 5 March 2010 (UTC)[reply]


I can think of a couple of Japanese cities that would not have suffered as much if the value of c in E=mc2 were not so large. TimBuck2 (talk) 17:07, 5 March 2010 (UTC)[reply]

Awesome answer. We have all benefited, since if c were larger we may have destroyed all of Japan, or even the entire earth with the first atomic test. Also, if c were a lot smaller, I imagine that we wouldn't be able to generate much electricity through nuclear reactors.24.150.18.30 (talk) 17:44, 6 March 2010 (UTC)[reply]

new Best answer (so far) as chosen by OP:Reduced from a spurious subheading that intruded in the contents list. Cuddlyable3 (talk) 18:09, 6 March 2010 (UTC)[reply]

As you increase c, I think it would get harder to implement the Global Positioning System with the same accuracy. Obviously it wouldn't work at all with infinite c. If you must split my name in two, it should be Cone Slayer, not Cones Layer. -- Coneslayer (talk) 15:43, 5 March 2010 (UTC)[reply]
Along the same lines, it would be more difficult for us to precisely measure the distance to the moon. —Bkell (talk) 15:54, 5 March 2010 (UTC)[reply]
the GPS is the best practical answer anyone here has given so far, and it is spot on, since it relies on specific timing of distances that are travelled at or nearly at c. Therefore, with an infinite or much larger c, this would become difficult. Can anyone come up with other practical aspects of our life that could not work but for c on the scale it currently is? Thank you. 82.113.121.103 (talk) 16:52, 5 March 2010 (UTC)[reply]
You're pretty insistent, aren't you? —Bkell (talk) 18:00, 5 March 2010 (UTC)[reply]
This is a meaningless question. If 'c' were even slightly different (either more or less) than it actually is, then humans would not exist...but radically different life-forms might. If it were significantly different then probably galaxies and stars wouldn't exist. One of the things you learn after enough years answering questions on the reference desks is that once one utter impossibility has been injected into a question, all else falls to the ground. We can't meaningfully list trivia like GPS being more or less accurate when the elephant in the room is that the existence of all things pretty much depends on 'c' being precisely what it is. GPS could not possibly function in any way whatever if 'c' were more than a percent or two different than it actually is because nobody would have been here to invent it...and in all likelyhood, there wouldn't even be a "here". Sorry - but you don't get to pick between answers you like and answers you don't. You get answers...hopefully true ones. SteveBaker (talk) 19:30, 5 March 2010 (UTC)[reply]
I thought he showed outstanding judgment and discernment with his choice of answer. -- Coneslayer (talk) 19:35, 5 March 2010 (UTC)[reply]


If c were infinite, and the universe were also spatially infinite, then the sky would be blindingly bright 24/7, would it not? Vranak (talk) 21:44, 5 March 2010 (UTC)[reply]
thank you for the response, can you explain your thinking in more detail, specifically what causes the sky to be brighter than it is now? thank you. 82.113.121.94 (talk) 21:57, 5 March 2010 (UTC)[reply]
Olbers' paradox --ColinFine (talk) 00:24, 6 March 2010 (UTC)[reply]

If you look up in the sky there is a giant thermonuclear furnace that relies on E=mc2. If you start playing with that c you could either turn off or explode the sun. Even a ~5% change in solar luminosity would change the temperature on Earth about 10 C, so that isn't a balance to be trifled with. Dragons flight (talk) 02:09, 6 March 2010 (UTC)[reply]

OK, I'll bite. Your question presumes that a "benefit" must be a "benefit" to "us," i.e., people. We have existed only a fraction of a million years, whereas the universe is 13.8 billion years old. So obviously nothing in the universe exists for "our" benefit. Perhaps you mean "life in general" -- does c benefit life? If light had no speed limit, then light would be infinitely fast. If so, every form of life in the universe would be blind, because all the light in the universe would endlessly travel around the universe; nothing could evolve "eyes," because no organic organ (developing from a primitive predecessor) could adjust to infinite stimulation. So, yes, the speed of light helps "us" because "we" like to be able to see things, which "we" couldn't if "we" were incapable of evolving optical organs. 63.17.82.123 (talk) 04:06, 6 March 2010 (UTC)[reply]
I disagree completely. Why would there be any more light, on average, at any given point ? We would see the light emitted from Proxima Centauri now, instead of the light it sent out some 4.2 years ago, but how would that change the total amount of light we see from that source ? Now apply that same logic to every other light source. If you're thinking there are an infinite number of stars out there, and the light from most of them hasn't reached us yet, due to a finite value of c, I don't think that's right. I believe there's a very large, but still finite, number of stars. Also, light doesn't travel an infinite distance, as eventually it gets absorbed by something, like interstellar dust. StuRat (talk) 16:12, 7 March 2010 (UTC)[reply]
Just for kicks, Wikipedia has a page on the variable speed of light (http://en.wikipedia.org/wiki/Variable_speed_of_light) which has some information on cosmologists investigating the possibility of c not always being what it is known as today.24.150.18.30 (talk) 17:52, 6 March 2010 (UTC)[reply]

Entropy

Is the Moon a higher or lower entropy environment then the Earth? What caused it? TheFutureAwaits (talk) 12:59, 5 March 2010 (UTC)[reply]

Our article on entropy suggests the definition "entropy is as such a function of a system's tendency towards spontaneous change." As such, the Earth is a lower entropy system than the Moon, as it is more prone to spontaneous change, and it is such primarily because it is larger -- large enough to retain an atmosphere and an active volcanic system. — Lomn 14:06, 5 March 2010 (UTC)[reply]

I think it's the opposite way : low entropy means highly ordered, for example instead of a moon, a perfect sphere made entirely of a single element and uniformly a single temperature would be highly ordered: you could basically describe it entirely in half a sentence, giving the diameter, the element it is made of, and the temperature (maybe I'm leaving out one or two things). Because you can describe it in very few words, it therefore has a very low entropy. Now the moon has much higher entropy than a perfect sphere made of a single element. You would need far more space to describe it fully. But the earth has a higher entropy still: it is much more complex. So, I would say that the Earth is a higher-entropy environment. To put it another way, as a percentage, you increase entropy far more when you put an American flag into the low-entropy conditions on the moon than when you place on in the high-entropy conditions on an Earthly mountain. Can someone better versed in math and science confirm my interpretation? Thanks. 82.113.121.103 (talk) 14:30, 5 March 2010 (UTC)[reply]

That's completely wrong. Entropy is not a measure of how many words it takes to give a macroscopic description of an object. It is measure of how many microscopic states are consistent with the macroscopic description. To labor on your example, if you were to melt the moon, mix if thoroughly and find a way to cool it fast enough to keep the mix uniform the final sphere would have a higher entropy. Read entropy of mixing. Dauto (talk) 15:42, 5 March 2010 (UTC)[reply]
I find this extremely hard to believe. You are telling me if we took the universe, melted it all together, and made a black hole out of it, with precisely 0 information in the black hole other than maybe it's total mass (a single real number, in grams) and MAYBE one or two more variables such as it's spin and charge (maybe) then there would be MORE entropy in Universe (even though you can just describe it as "1 black hole, in the "center" (ha ha) of nothing else, having mass x, charge y, and angular momentum z". Even if you give all of these to an obscenely unrealistic level of exactness, you still will use maybe a paragraph of digits. A paragraph, even using the best theoretically possible compression, is not enough to accurately describe (ie represent a compressed version of) even a single book (say, a collection of Shakespeare plays). So it seems to me that a SINGLE book would have more entropy than all of the universe, if you reduced the universe to a black hole. Likewise, it seems to me that a SINGLE city on Earth would have more entropy than the Moon, if the moon were a uniform substance you can perfectly describe in a few words. If I really am wrong, maybe it's because I'm conflating physical entropy with information entropy? For me, the fewer words you can use to give a second God in a different Universe enough information to fully reproduce an exact copy of something, the lower entropy it has. Our God would need to give a LOT of information to a second God in a different Universe to reproduce the Earth, but considerably less if the Earth were a uniform ball that is an exact geometrical sphere, of fixed temperature, density, etc. Don't you think? Can someone confirm whether I'm right, or whether Dauto above is right? Thank you. 82.113.121.103 (talk) 17:13, 5 March 2010 (UTC)[reply]
Yes, that's what I'm telling you. In fact for any given mass a black hole will be the state of maximum entropy. See black hole thermodynamics.Dauto (talk) 01:31, 6 March 2010 (UTC)[reply]
A small correction: I meant to say that for any given volume a black hole will be the state of maximum entropy. Dauto (talk) 04:20, 6 March 2010 (UTC)[reply]
These answers are just leading to more confusion. Dauto, is the Moon higher or lower entropy than the Earth? Why? TheFutureAwaits (talk) 15:56, 5 March 2010 (UTC)[reply]
If you take the planet as a whole I would venture that earth's mean specific entropy (entropy per unit volume if you will) will be higher simply because earth's core temperature is higher. Dauto (talk) 16:05, 5 March 2010 (UTC)[reply]

My wording may be off, what I'm getting at is which way is the energy exchange moving? So for example in the Sun-Earth system the Sun is increasing in entropy while the Earth is decreasing. How does this work in the Earth-Moon system? TheFutureAwaits (talk) 16:20, 5 March 2010 (UTC)[reply]

I doubt there's any meaningful give-and-take between the two. Entropy increases. — Lomn 16:36, 5 March 2010 (UTC)[reply]
I don't think the sun's entropy is increasing since the most important factor here is likely the fact that it is losing a massive amount of heat through radiation so its entropy is actually decreasing. The radiation exchange between the earth and the moon is not a very important factor since they are a similar temperatures which means radiation is moving in both directions. Dauto (talk) 16:40, 5 March 2010 (UTC)[reply]
Correct me if I'm wrong, but it seems relevant to highlight the increase in entropy of the system, e.g. as the sun radiates energy and mass. The mass that we call the sun one moment becomes a geometrically larger object the next, and the entropy of that system increases (while the entropy of the circumscribed orb we call "the Sun" may decrease). -- Scray (talk) 17:29, 5 March 2010 (UTC)[reply]

Probably the most important point is that theoretical thermodynamics shows that entropy per se is not very important -- what matters for understanding interactions is the dependence of entropy on energy, which is measured by a quantity we call the temperature. Looie496 (talk) 16:58, 5 March 2010 (UTC)[reply]

Since a black hole decays into hawking radiation, it can't be the state or maximum entropy, rather a bunch of randomly spread photons and leptons is a state of higher entropy, see Heat death of the universe and Black hole#Entropy and Hawking radiation 82.132.136.207 (talk) 00:32, 7 March 2010 (UTC)[reply]

The black hole is the state of maximum entropy for a given volume. The Hawking radiation that replaces the hole will ocupy a larger volume and can have a larger entropy then the hole as you've shown it must. Dauto (talk) 02:18, 7 March 2010 (UTC)[reply]

panda

Where can you find a giant panda in the United States? —Preceding unsigned comment added by Yelopiclle (talkcontribs) 14:02, 5 March 2010 (UTC)[reply]

From the Giant Panda article...

As of 2007, five major North American zoos have Giant Pandas:

  • Chapultepec Zoo, Mexico City – home of Xi Hua, born on June 25, 1985, Shuan Shuan, born on June 15, 1987, and Xin Xin, born on July 1, 1990 from Tohui (Tohui born on Chapultepec Zoo on July 21, 1981 and died on November 16, 1993), all females
  • San Diego Zoo, San Diego, California – home of Bai Yun (F), Gao Gao (M), Su Lin (F), Zhen Zhen (F), and Yun Zi (M).
  • US National Zoo, Washington, D.C. – home of Mei Xiang (F) and Tian Tian (M).
  • Zoo Atlanta, Atlanta, Georgia – home of Lun Lun (F), Yang Yang (M) and Xi Lan (M)
  • Memphis Zoo, Memphis, Tennessee – home of Ya Ya (F) and Le Le (M)

Googlemeister (talk) 14:18, 5 March 2010 (UTC)[reply]

Who's the best when nature calls?

Hi

1.I've often wondered when you have to go, (pee) but sometimes in a situation where you can't (whatever the reason might be). Who's better at holding it, males or females? -However I don't think females can hold it that long.

2. What are the complications of holding a number 2 for too long?

3. I'm sure most guys have had this happen to them somewhere along their lives. However if you've still somehow managed to elude this experience you're in for a treat. I've been hit a couple of times in the groin, but on one or two occations it hurt so bad that I felt I was going to need a new pair of undergarments.

3.1 Is this normal and what are the complications when the injury is serious?


Thanks, NirocFX

41.193.16.234 (talk) 14:05, 5 March 2010 (UTC)[reply]

Women can hold for slightly longer in tests, but it's negligible. There are no adverse affects to health of holing a poo in too long, you'll simply lose control of the bowls and crap your pants. And it's very normal to experience massive pain for both men and women when impacted on the genitalia area. They risks include hemorrhaging and sterility. R12IIIeloip (talk) —Preceding undated comment added 14:47, 5 March 2010 (UTC).[reply]
I disagree on "holding in poo". The large intestine removes water, and poo held in too long (several days) will thus be dried out and cause constipation. StuRat (talk) 15:37, 5 March 2010 (UTC)[reply]
Cecil Adams covered the potential hazards of holding in "Number 1" for too long here. Doesn't mention the other, though. APL (talk) 15:43, 5 March 2010 (UTC)[reply]

It's possible to get so constipated it comes out the wrong end. So you might want to hold your tongue too . . . or at least your breath. —Preceding unsigned comment added by 71.108.171.138 (talk) 23:32, 5 March 2010 (UTC)[reply]

Was that comment necessary? It is not pertinent to the discussion.--79.68.242.68 (talk) 01:24, 6 March 2010 (UTC)[reply]

Microbial locomotion time

If I am trimming the fat from some chicken thighs with a 5" chef's knife, how long could I expect it to take for the potentially harmful bacteria from the blade to make their way to the handle such that my right hand (holding the knife) should be reasonably assumed to be contaminated? I ask because I usually tend to perceive my right hand to be totally clean and use it without much discretion in terms of touching other things in the kitchen while preparing raw meat, such getting a pot from the cabinet, taking things from the fridge, etc. DRosenbach (Talk | Contribs) 16:40, 5 March 2010 (UTC)[reply]

Our Swarming motility article describes this as "rapid (2–10 μm/s) and coordinated translocation of a bacterial population across solid or semi-solid surfaces". If we take the upper end of that range (10 μm/s), then even if the bacteria were to move directly toward your hand, it would take more than 40 minutes for them to move an inch (or a couple of hours for a few inches). Add to that the less-directed nature of bacterial movement, the less-than-ideal culture conditions of a knife surface, and the fact that I assumed the upper end of the range of rates. For most users (perhaps not a trained health professional like yourself), it seems extremely unlikely that this mechanism would account for more contamination of your kitchen than a slip in technique (such as setting the knife down in a "clean" versus "dirty" area). -- Scray (talk) 17:13, 5 March 2010 (UTC)[reply]
I dislike that answer. By far the most likely way bacteria would get from something you're cutting onto your hand would be as a fine aerosol of particles sprayed from the object being cut. The mere act of cutting something is going to first stretch - then break and release fibers in the material. As they elastically return to their former shape, they could easily flick microscopic droplets from within the meat or whatever onto your hand. Also air currents and other things could easily be involved. Just measuring the speed a bacterium can move under its' own steam is not going to give you anything but a low-end estimate. I'd guess that the high end is probably 100 mph or something. SteveBaker (talk) 19:18, 5 March 2010 (UTC)[reply]
Not sure why you "dislike" that answer enough to make a point of saying so (twice). It's a direct answer to the OP, who asked whether the hand with which he holds the cutting knife is likely to get contaminated by direct spread of the bacteria. The question was not, "how do bacteria spread in the kitchen". I do like your conjectures - they seem plausible (except for the bit about bacterial coming from within intact meat). -- Scray (talk) 20:05, 5 March 2010 (UTC)[reply]

120 volt equipment

Hello, Q-what happens to 120 volt 60 hz equipment when it is plugged in to 120 volt 400hz power source? my theory is that it will run for awhile, but eventually it would overheat, example such as an electric motor that's 120 volt hz? —Preceding unsigned comment added by 205.200.77.222 (talk) 16:48, 5 March 2010 (UTC)[reply]

It's a difficult question because it depends entirely on the appliance. Somethings will die instantly, others will run perfectly happily, others will be somewhere inbetween. I live in the USA (60Hz, 120v) but have some things I brought with me from the UK (50Hz, 240v) - the cheaper kinds of converters convert to the correct voltage but put out the wrong frequency (ie 60Hz, 240v) and I have several gadgets that don't like that - one overheats, two others just don't work - and that's with just a 10Hz difference! A more expensive converter that I found corrects the frequency too - and that allows those gadgets to work OK. SteveBaker (talk) 19:12, 5 March 2010 (UTC)[reply]
In general, using electronics designed for low-frequency power on a high-frequency power input is safer than the other way around -- you don't need to worry about transformer cores saturating and overheating. Electric motors are one of the exceptions: if the input frequency is faster than the motor can spin, the motor won't move at all, and will act as a short circuit. --Carnildo (talk) 02:07, 6 March 2010 (UTC)[reply]
How about if it is a universal motor? Would a 120 volt universal motor run just as well on DC or 60 Hz or 400Hz? I would expect a purely heating appliance or an incandescent light bulb similarly should run on DC to 60 Hz to 400 Hz. Edison (talk) 21:28, 6 March 2010 (UTC)[reply]

Matter

I know matter can be converted to energy, but can it be converted to anything else? Dark matter perhaps?

Its not that matter can be converted into energy, like a magic trick. Its that matter and energy are different expressions of the same fundemental concept. Matter is merely energy which has been confined by the limits of a set of quantum numbers, but fundementally matter is energy and energy is matter. See Mass–energy equivalence for more information. --Jayron32 17:43, 5 March 2010 (UTC)[reply]
yeah, you might have heard of this equivalency as e=m times a constant which I forget. 82.113.106.100 (talk) 21:26, 6 March 2010 (UTC)[reply]

Ant

Which species of any has a stinger AND bites to inject formic acid into the wounds? —Preceding unsigned comment added by 518c&e (talkcontribs) 17:04, 5 March 2010 (UTC)[reply]

Species upper limit?

Have scientists discovered almost all species that exist? —Preceding unsigned comment added by SpiderLighting (talkcontribs) 17:09, 5 March 2010 (UTC)[reply]

As surprising as it might seem, only a minority of species have been identified. -RobertMel (talk) 17:15, 5 March 2010 (UTC)[reply]
Obviously we have not identified them all, but without knowing how many there are, how do we know we have more then 50% to go? Googlemeister (talk) 17:29, 5 March 2010 (UTC)[reply]
And in order to know if we've discovered them all, we'd first have to know how many there are. Which we don't. The species article has some things to say about this. Dismas|(talk) 17:18, 5 March 2010 (UTC)[reply]
Certainly not. Researchers have detected over 700 species of periodontal pathogens in the diseased gum tissue but have characterized and positively identified and marked less than 300 of them -- and that's just bacteria of the gums, let alone of the entire mouth and let alone the entire body and let alone the entire world. But in case you were referring to animal species, the answer would still be no. DRosenbach (Talk | Contribs) 17:21, 5 March 2010 (UTC)[reply]
Likewise, I have heard it speculated that if we counted up all of the discrete animal species we have identified, it would still be less than the number of beetle species yet unidentied. We are no where near ending the catalogueing of species. Furthermore, depending on how you define species, there are some life (bacteria) or pseudolife (virus) forms which speciate at a rate which means that we can get new ones within a human lifespan, meaning that we will never be done. Even if we confine ourselves to macrolife (plants & animals) we aren't even close to being done catalogueing them. --Jayron32 17:41, 5 March 2010 (UTC)[reply]
To add that many species will just vanish before we identity them. -RobertMel (talk) 18:04, 5 March 2010 (UTC)[reply]
The question of how to estimate the total number of species is interesting. One approach might be sampling. For example, if you take one small island and study it to death to hopefully identify every species, and find that there are 10 times as many as you knew initially, that might be some indication that we know less than 50% of the species worldwide. StuRat (talk) 17:51, 5 March 2010 (UTC)[reply]
I suppose you might also plot a graph of the number of newly discovered species per year (adjusted for the number of people hunting them and the amount of effort they put into it) and see if the graph was showing any signs of asymptoting out - which would be a clue that we were close to finding them all. However, (as others have pointed out) there is no way that we're 50% of the way through the process if you include microscopic stuff like bacteria and algea. That 50% number could only possibly be for things the size of insects and above. SteveBaker (talk) 19:06, 5 March 2010 (UTC)[reply]
Plus, the definition of the word "species" is far from settled, so you have to argue about that for another few hundred years before you even start counting beetles. --Sean 21:28, 5 March 2010 (UTC)[reply]
We've mapped maybe 3% of the floors of the oceans. It is unilkely that this is the coolest 3% out there. There are lots and lots of species in the oceans alone that we have not ever seen. Comet Tuttle (talk) 22:33, 5 March 2010 (UTC)[reply]
There are also likely millions of species of microorganisms in soil that we have not yet identified. Do those count? ~AH1(TCU) 04:01, 6 March 2010 (UTC)[reply]
Do you mean species on Earth? If there is life elsewhere in the universe, we are probably nowhere even close to discovering all of the species that exist.24.150.18.30 (talk) 18:01, 6 March 2010 (UTC)[reply]

For microorganisms, estimates of the number of species are made by counting the number of distinct DNA sequences in a sample of earth or water, and then doing calculations. This method yields numbers far higher than attempts to count species directly. Looie496 (talk) 18:07, 6 March 2010 (UTC)[reply]

relationship between entropy in physics and entropy in information science?

if you look at my above edit, you will see that I might be very confused indeed. Can someone help explain to me in simple terms what the relationship, if any, is between entropy as understood in physics and entropy as understood in information science? If there is no relationship, why is it the same word? Thank you. —Preceding unsigned comment added by 82.113.121.103 (talk) 17:19, 5 March 2010 (UTC)[reply]

We have an article about this. See Entropy in thermodynamics and information theory. In simple terms, entropy is used in both cases to refer to a loss of usability. In thermodynamics, it refers to the loss of Free energy in the universe, or in any closed system. In information theory, it refers to the loss of predictability or understandability in some bit of information. Its sort of like how both biology and chemistry use the word "nucleus" to mean "the bit in the center" (either a cell nucleus or an atomic nucleus.) However, the two entropies are far closer related than that. The mathematics of both is still controlled by the Boltzmann equation, which in information theory takes the slightly different form known as the Hartley function. There are also some gedanken experiments in physics which tie the two fields together nicely. Schroedinger's cat is essentially about information entropy (uncertainty in knowing the state of decay of a single particle is a form of information entropy, as is the fate of our poor cat). The paradox of Maxwell's demon, which conceives of an energy-less way of reducing entropy (and thus SHOULD be a violation of the second law of thermodynamics) can be resolved if we consider information to be negative entropy. I have a nice little "physics for laypeople" primer I should dig up which discusses the connections between information entropy and thermodynamic entropy. --Jayron32 17:37, 5 March 2010 (UTC)[reply]
Thank you. Again looking at my above edit, as I referenced at the start of this thread, could you try to understand the way in which I was trying to see black holes, planet-sized highly ordered/uniform/geometrically perfect and easily described objects, and the actual Earth, in terms of "entropy" (which I was using in more the informational sense) and tell me why it doesn't apply (if it doesn't) in a physical sense. ie someone in the linked thread said that I was wrong, and I wonder why (if it's true). Contrary to the implications you make above, physical and information entropy seems to me would have to be opposites in direction/sense in order for me to be wrong above... 82.113.121.103 (talk) 17:49, 5 March 2010 (UTC)[reply]
Well, the deal is that entropy is often a slightly misused term in the physical sciences (thermodynamics). Entropy is really just a mathematical concept, after all Ludwig Boltzmann was basically a mathematician. Entropy is just the relationship between the number of possible states of a closed system and the uncertainty of knowing which of those states you are in at any given moment. In thermodynamics, the term is used to basically mean the disorder in a system; it is expressed as negative free energy, or in other terms, entropy is expressed as "the energy needed to return a system to a state of perfect order, where all variables are eliminated and the location of all particles is known with perfect precision". Entropy as a concept is still the mathematical thing, but in thermodynamics we discuss it in terms of the free energy lost in reordering a system. In information theory, the entropy is the same idea; it is based on the number of possible states of a system and the uncertainty of knowing which state the system is in. Information theory actually uses a more pure form of entropy, in that it doesn't use a concept like energy as a surrogate for entropy; it uses a unitless value to express "pure" entropy. But the sign and directionality of the two are the same; the more disordered the system, the higher the value the entropy is. --Jayron32 01:26, 6 March 2010 (UTC)[reply]
Jayron, your final conclusion is essentially correct but your argument is stuffed with a potpourri of misconceptions. I won't go into all of them. Let me just point out that entropy and Free energy are not even measured with the same units so your explanation makes no sense. BTW your rant abount physical sciences misusing entropy is also a gem of nonsense. Dauto (talk) 03:20, 6 March 2010 (UTC)[reply]
Well, its not that they misuse the term entropy really. Just use it a bit differently than does information theory. And entropy is essentially negative free energy (for any given temperature). At a conceptual level, the two are basically opposite concepts. Entropy is merely disorder, the thermodynamic definition expresses this disorder as a unit of energy per temperature. Free energy is energy availible to do work; we can consider the theoretical "free energy of the universe" to have been at a maximum at the Big Bang, and it has been consitantly decreasing over time. Thus, the for the universe, spontaneousness is represented by a decreasing free energy, or ΔG < 0. The early physical chemists and mathematicians working in this area recognized that this was essentially the functional opposite of entropy; the entropy of the entire universe tends towards a maximum as the free energy tends towards a minimum. When we look at a chemical process, by Helmholtz's and Gibbs's conventions, we tend to take the perspective of "free energy". But removing the math for a second, and looking at the conceptual nature of it. Every process has two contributions to make towards affecting the entropy of the universe. Entropy is temperature dependant (warmer things have faster moving particles, so their position is less certain than cooler things), so any process that tends to heat its surroundings tends to increase the entropy at the universe level. However, most chemical processes also involve a change in organization of the particles, which is a direct effect on the entropy itself. So, if we want to look at a process, there are two things going on: Release or absorbtion of heat, which affects the entropy of the surroundings and reorganization of the substances involved, which affects the entropy of the system. Conceptually, entropy and free energy are basically the same thing in the opposite direction, and in thermodynamics, we tend to express everything in terms of energy, so by convention, the entire thing is expressed in terms of energy values, the classic G = H - TS (Or A = U - TS for the Helmholtzally inclined) which contains the expression of entropy as an energy value (J/K). But this is a convention to give us a number we can find meaning with (usually looking at the "spontanaity" or "extensiveness" of a chemical process) to compare different processes. --Jayron32 04:42, 6 March 2010 (UTC)[reply]
You spend a whole paragraph explaining how entropy and free energy are supposed to be the same concept and then write a (correct) equation that shows that they are related but are definitely different concepts and then conclude by saying that the equation is just a convention? Are you a Chemist? Dauto (talk) 15:45, 6 March 2010 (UTC)[reply]

Any known planets with satellites with satellites?

Are there any known cases in astronomy of a planet which has a moon orbiting it and that moon has another smaller moon orbiting it?20.137.18.50 (talk) 17:39, 5 March 2010 (UTC)[reply]

This question was answered a few years ago - the short answer is "No". The longer answer is that the nature of gravitational and tidal interactions between planet and moon would make a moon-of-a-moon orbitally unstable - so if one ever did somehow come into existence, it would either smack into the planet, the parent moon - or spin off into space within a relatively short period of time. The only exception to that are the various artificial satellites that humans put in orbit around various moons within the solar system - which were there only for short periods of time. SteveBaker (talk) 18:55, 5 March 2010 (UTC)[reply]
Thanks, Steve. When Lomm said in that original question "The key consideration (discussed at n-body problem) is that the smallest body must have a mass insignificant with regards to the largest body." with respect to the word "insignificant" are we talking 1/10, 1/100, 1/1000? I followed the link to the n-body problem and didn't see any elaboration of significance of mass difference there, though I have a little more idea of how crazily complex things get with moon-on-moon action. 20.137.18.50 (talk) 19:08, 5 March 2010 (UTC)[reply]
A close possibility is the tiny moon Nix and Hydra. They are orbiting a double body system, so in a sense, the satellite is orbiting a pair of satellites where Pluto is orbiting Charon while Charon is simultaneously orbiting Pluto. I suppose someone will come by soon and point out that none of these are technically planets. Googlemeister (talk) 19:16, 5 March 2010 (UTC)[reply]
Drat, I was just going to say that. I also wonder if Mercury and maybe Venus lack moons for the same reason, basically that the nearby Sun would knock the moons out of orbit. StuRat (talk) 21:17, 5 March 2010 (UTC)[reply]
The Lunar Reconnaissance Orbiter now orbiting the moon is a small artificial 'moon' of the Moon. You may be familiar with the planet. Cuddlyable3 (talk) 03:07, 6 March 2010 (UTC)[reply]
The LRO has the mass of a large automobile. Astronomical definitions are notoriously sketchy, but it's doubtful anything the size of a Lincoln Continental would ever be defined as a "moon," artificial or otherwise. 63.17.82.123 (talk) 04:25, 6 March 2010 (UTC)[reply]
@63.17.82.123 Enter "artificial moon" in the Wikipedia search box and you are taken to the article about artificial satellites. That is correct. The first satellite in fiction was called The Brick Moon. There is nothing that an artificial moon can be other than a manufactured satellite. Your objection to my use of the term "artificial moon" is groundless. Cuddlyable3 (talk) 18:04, 6 March 2010 (UTC)[reply]
Well clearly, the sun-earth-moon system has been stable for quite a while, so mass_moon/mass_sun = 3.69e-8 is "insignificant". —Preceding unsigned comment added by 83.134.176.244 (talk) 08:37, 6 March 2010 (UTC)[reply]
Doesn't distance play a role ? That might explain why Mercury (and maybe Venus) doesn't have any moons, while Pluto has at least 3. StuRat (talk) 13:22, 6 March 2010 (UTC)[reply]

Telescopes

Is there a functional limit on the level of magnification a telescope can achieve? So for example, how big would the lens have to be to read a car's license plate on Earth from Alpha Centauri?

Are there any technologies would could implement that would reduce that size or is it a hard limit? TheFutureAwaits (talk) 18:06, 5 March 2010 (UTC)[reply]

Generally speaking, the main purpose of a telescope is not to magnify to a great extent, but to collect as much light as possible, to study faint objects. That said, the angular resolution of a telescope depends primarily on its diameter. The larger the diameter of the telescope, the finer the detail it can resolve. On the surface of the earth, however, atmospheric seeing spoils the resolution of optical telescopes larger than ~10 cm, so you either need advanced technologies like adaptive optics or interferometry, or you need to put the telescope in orbit. -- Coneslayer (talk) 18:13, 5 March 2010 (UTC)[reply]
To address your question quantitatively, to read a license plate on Alpha Centauri, you need an angular resolution of roughly θ = (1 cm) / (1 pc) = 3e-19 radians. Using θ = 1.22 λ/D (as explained in angular resolution), and assuming λ = 500 nm (visible light), you need a space telescope with diameter D = 2e12 meters, or 2 billion kilometers, or 13 astronomical units. That's bigger than the orbit of Jupiter. -- Coneslayer (talk) 18:21, 5 March 2010 (UTC)[reply]
see http://en.wikipedia.org/wiki/Diffraction_limit —Preceding unsigned comment added by 83.134.176.244 (talk) 18:14, 5 March 2010 (UTC)[reply]
(ec) Even ignoring such things as atmospheric distortion and nearby bright things such as the Sun, one important question to consider (I don't know the answer) is how many photons you would need reflected from a license plate in order to be able to read it, and the rate at which these photons would reach Alpha Centauri. Probably you don't even have the remotest chance of being able to read a license plate, because at Alpha Centauri you'd receive one photon reflected from the license plate every 300 years (totally out-of-my-ass guess, but you get the idea). —Bkell (talk) 18:15, 5 March 2010 (UTC)[reply]
You could (in theory) solve the issue of the rare appearance of a photon reflected from that license place by integrating the incoming light over a very long period - but now we have a mirror the size of the orbit of Jupiter that's got to stay pointing in the right direction for (perhaps) tens of thousands of years - getting that kind of stability would be really tough - and in any case, that gigantic mirror would have to be moved to track the motion of the license plate relative to the telescope. If it has to point accurately to within 3x10-19 radians yet move by enough to track the motion of the license plate parent planet rotation - and that planets' orbit around the parent star - and the relative motion of that star with respect to the telescope (not negligable over the amount of time you'd need to be gathering light over)...then the mechanism that moves the mirror would be a horror to construct! This is so far from being a practical possiblity that we're going to have to say that the answer is "No" - you can't do that. SteveBaker (talk) 18:49, 5 March 2010 (UTC)[reply]
I wouldn't hold the mirror still, I would just measure its movements and have a computer compensate for them in the final image. --Tango (talk) 19:46, 5 March 2010 (UTC)[reply]

Interesting, so then how big of a diameter lens would be required to achieve sufficient angular resolution to read a license plate of a car on Earth assuming there was no atmosphere? TheFutureAwaits (talk) 18:17, 5 March 2010 (UTC)[reply]

Hmmm. I read license plates on earth without any lens all the time ;-). --Stephan Schulz (talk) 18:48, 5 March 2010 (UTC)[reply]
As Coneslayer already calculated...about the size of the orbit of Jupiter. SteveBaker (talk) 18:49, 5 March 2010 (UTC)[reply]
I was still drafting my calculation when TheFutureAwaits asked above. Given the "on Earth" part of this question, I'm not sure if it's meant to be a restatement of the Alpha Centauri question, or a different question. -- Coneslayer (talk) 19:01, 5 March 2010 (UTC)[reply]
The mirror does not need to be a disc 13 AU in diameter. You can have a series of smaller telescopes, all coordinated together. This already happens with radio telescopes and I think more rarely with optical telescopes. I cannot recall what this is called - I'm sure there must be an article on it. Edit: see Cambridge Optical Aperture Synthesis Telescope and http://www.mrao.cam.ac.uk/telescopes/coast/handout.html Very Long Baseline Interferometry Perhaps you could just have one telescope at Jupiter and put together what it sees with what it sees six "Jupiter months" later. 89.243.198.135 (talk) 19:13, 5 March 2010 (UTC)[reply]
That would be interferometry (or aperture synthesis), which I linked to in my original response. -- Coneslayer (talk) 19:14, 5 March 2010 (UTC)[reply]
In regards to Perhaps you could just have one telescope at Jupiter and put together what it sees with what it sees six "Jupiter months" later, no. Interferometric observations must be combined coherently. Without a direct way to measure optical phase, I do not believe there is any way to interferometrically combine optical observations taken at different times. -- Coneslayer (talk) 20:42, 5 March 2010 (UTC)[reply]
The Aperture synthesis article says that one telescope with the rotation of the earth is used. What stops a one-year rotation being used instead of a twenty-four hour rotation, at least for radio waves? 89.243.198.135 (talk) 20:53, 5 March 2010 (UTC)[reply]
Can you point me to where you're seeing "one telescope"? With a single baseline (two telescopes), you use the rotation of the earth to change the orientation of the baseline relative to the target, which lets you achieve high resolution along different axes at different times. But in any case, that's radio interferometry, not optical interferometry. It's possible to directly record phase of radio signals, which lets you do the interferometry in post-processing. You can't do that in the optical. -- Coneslayer (talk) 20:57, 5 March 2010 (UTC)[reply]

The alphacentaurians(?) could at best see only the license plate you had over 4 years ago. Cuddlyable3 (talk) 02:59, 6 March 2010 (UTC)[reply]

Even from earth orbit it is very hard to read license plates. I'm wondering why the alphacentaurians stick their license plates on the top of their cars instead of at the ends. Dmcq (talk) 08:44, 6 March 2010 (UTC)[reply]
Of course you don't look for license plates in the center of the Earth's disk, but at the very corner, when they are just driving over the horizon. You can even use head and tail lamps for targeting. Duh! --Stephan Schulz (talk) 09:10, 6 March 2010 (UTC)[reply]

OTC Cryogenics to treat warts or moles

Are OTC Cryogenics to treat warts or moles? Are they effective? What if a person would apply it on something else? For example, it is for moles and he applies it on a wart or the other way round? Quest09 (talk) 18:38, 5 March 2010 (UTC)[reply]

They are to treat warts (and are effective) and their instructions specifically state that they are not to be used on moles. I'll think on the rest of your question, but I think it's bordering on medical advice. -- Flyguy649 talk 18:56, 5 March 2010 (UTC)[reply]
(edit conflict)I have heard of warts being treated with liquid nitrogen which, amongst other things, stimulates the immune system into responding to the hpv virus that causes the wart. The effectiveness probably depends on the person and type of wart. I'm not sure about moles. — Ƶ§œš¹ [aɪm ˈfɹ̠ˤʷɛ̃ɾ̃ˡi] 18:59, 5 March 2010 (UTC)[reply]


I think in general it's a bad idea to think of warts and moles as having anything to do with each other. Warts are viral infections; they need to be gotten rid of, so the virus doesn't spread. Most moles, on the other hand, require no treatment at all; you can just leave them be unless they're bothering you in some way. There are of course exceptions — see malignant melanoma for the worst-case scenario. --Trovatore (talk) 19:01, 5 March 2010 (UTC)[reply]

120v v. 240v

How much power could you safely continuously draw through an ordinary domestic 120 volt American electricity socket versus an ordinary domestic British 240 volt (or possibly 220 volts) socket? British electric sockets are installed on a ring main which I think also allows more power to be drawn. Thanks 89.243.198.135 (talk) 20:30, 5 March 2010 (UTC)[reply]

I can't speak for UK circuits, but a North American 15 amp circuit is good for about 1800 watts, assuming no other loads. At that rate the circuit breaker may eventually trip if load is continuous. Acroterion (talk) 20:55, 5 March 2010 (UTC)[reply]
UK must be more than that because I had a 2000 watt electric fire plugged into the mains. —Preceding unsigned comment added by Dataport676 (talkcontribs) 21:25, 5 March 2010 (UTC)[reply]
The maximum current you can get from a UK socket is 13A - the nominal UK mains voltage is 230V ± 10% (see Mains power around the world). This means that 3kW (13A @ 230.8V) _might_ blow the fuse if the voltage is a bit high (or with a constant-power device and the voltage a bit low). 2kW and 2.5kW devices are common, though. Tevildo (talk) 21:26, 5 March 2010 (UTC)[reply]
You can also still get the old BS 546 unfused 15A round-pin plugs (giving you 3.6kW @ 240V), but that's not really an _ordinary_ domestic socket these days. Tevildo (talk) 21:35, 5 March 2010 (UTC)[reply]
In Britain the 13A limit mentioned by Tevildo is imposed by a fuse in the standard plug, not the socket. Modern UK houses are wired with a ring main that is capable of delivering 13A to multiple sockets. Therefore by bypassing or up-rating the 13A plug fuse one can draw more current from a British socket, limited by the main fuse. This is not a safe procedure for an amateur but it is possible. Cuddlyable3 (talk) 21:53, 5 March 2010 (UTC)[reply]
I would advise very, very strongly _indeed_ against this (unless one wants to burn one's house down). If you need more than 3kW from a socket, replace it with a proper BS 546 15A one. You _can_ get more than 13A out of an ordinary socket by shorting out the fuse. You _can_ get a higher steam pressure in a boiler by welding down the safety valve. The OP, however, included (quite rightly) the word "safely" in the question. This solution does not satisfy that criterion. Tevildo (talk) 22:15, 5 March 2010 (UTC)[reply]
As long as you only do it on one device, you should be fine, but if you do it too much you'll trip the circuit. The mains breaker would normally take more effort to trip. --Tango (talk) 22:05, 5 March 2010 (UTC)[reply]
Concern for safety is admirable if it is based on facts. BS546 is an outdated British standard for round-pin plugs and sockets that are incompatible with modern 13A BS1363 sockets and lack safety features of BS1363: shuttered socket, finger shrouds, fuse in plug and flat-wiping contacts. If recommending someone install new sockets for high current supply the Europe wide IEC 60309 standard family is preferable because of its modern design. Connectors are colour coded yellow 100-130V, blue 200-250V. Versions for 16 A, 32 A, 63 A and higher are available. Cuddlyable3 (talk) 02:52, 6 March 2010 (UTC)[reply]
Similarly in North America, while the standard 120 V socket is intended for circuits fused at 15 A or 20 A, there are higher-current sockets available (yes, for 120 V as well as for 240 V); but in any country such higher-current sockets are meant only for use on a special circuit with heavier wiring than usual. --Anonymous, 05:23 UTC, March 6, 2010.
In UK,ring mains are normally fused at 30 Amps.--79.68.242.68 (talk) 00:17, 6 March 2010 (UTC)[reply]
This means that, in the UK, 3kW appliances are common. (The 13a fuse allows up to 3.12 kW at the standard UK voltage of 240v (yes, I know that 220 is quoted, but 240 is standard)). For anything larger than this, it would (in theory) be possible to use multiple plugs and cables, but this would be very dangerous for obvious reasons (live plugs!), so any appliance that draws more than 3 kW must be permanently wired with a high current switch. Instant showers up to 9.8kW are available, fused at 40 amps. Drawing more than 3kW by shorting out the fuse will result in a dangerously hot plug and socket because the connectors are not designed to carry more than 13a. Dbfirs 11:30, 6 March 2010 (UTC)[reply]
Are you sure that drawing more than 13A via a 13 Plug /socket arrangement will result in a dangerously hot plug and socket? Since P = I^2*R, what resistance would the plug/socket have to have to make it 'dangerously hot'? And how does this square with the commonly accepted standard safe current density in conductors of 1000A/cm^2? —Preceding unsigned comment added by 79.76.171.183 (talk) 21:49, 6 March 2010 (UTC)[reply]
A compliant plug and socket has a safety factor that should allows for 3 kVA plus a 'bit more' . This is because the 'contact points' between the cable and the screw terminals are a fraction of their cross sectional areas (cm^2). Yes, plugs and sockets can get warm. Properly wired and carrying no more that the rated kVA they should not generate undue heat though. So, as stated above: do not over load you circuit. I have seen charred plugs (more than once) where people have wired up two appliances to one plug and drawn toooo much current. --Aspro (talk) 22:23, 6 March 2010 (UTC)[reply]


I've not seen "ordinary 120 volt domestic outlets" rated above 20 amps, but U.S. electric codes usually want you to stay below 80% of that rating, which would be 16 amps. That said, if it is rated at 20 amps it could probably supply 20 amps. RV hookups have 30 amp 120 volt outlets. 50 amp 125 volt connectors are available.For more than 20 amps it is common to go to 240 volt supply. There is really no physical limit on how many amps a 120 volt connector could be built to supply for industrial or utility applications. Some low voltage network grids in large cities are built to carry thousands of amps at 120 volts, in a 120/208 three phase circuit, and these have connectors. Edison (talk) 21:24, 6 March 2010 (UTC)[reply]

Genus name

From where did the Genus Zyzzyx derive it's name? Googlemeister (talk) 20:34, 5 March 2010 (UTC)[reply]

First paragraph of that article you linked to, emphasis added:
Zyzzyx is a monospecific genus of sand wasp, containing a brightly-colored, medium-sized species, Z. chilensis, named after the sound they make while flying. They were first studied in detail by H. Janvier (a.k.a. Claude-Joseph) in 1928, more than 100 years after they were first described. The unusual name is onomatopoeia for the buzzing sound they make.
Hope that answers your question. —Bkell (talk) 20:36, 5 March 2010 (UTC)[reply]
I read that but suspected it was maybe vandalism. Googlemeister (talk) 20:42, 5 March 2010 (UTC)[reply]
Ah, okay, that's possible. —Bkell (talk) 20:48, 5 March 2010 (UTC)[reply]
This source says Alan Solem named Zyzzyxdonta and Aaadonta "with the idea of being the first and the last entries in any list of endodontoid snails." Not sure if the sand wasp is related. Gobonobo T C 21:28, 5 March 2010 (UTC)[reply]
I've fact-tagged the claims that it's supposed to represent the sound made while flying. There's no citation. Of interest of course is that there is a "Zyzzyx Road" in California, found on Interstate 15 between Los Angeles and Las Vegas. Two films are named after the road, according to our articles; the film Zyzzyx Road is said to have earned a US box office total of US$30, and an international take of over 10,000 times that amount. Comet Tuttle (talk) 23:58, 5 March 2010 (UTC)[reply]
The real road is actually Zzyzx Road. —Bkell (talk) 03:57, 6 March 2010 (UTC)[reply]

A little confusion concerning E=mc^2

I've been told that the E in E=mc^2 stands for energy in general. But, the equation itself is derived from solving the work integral (with m(v) = γ*m_o), and so I would assume that it is only applicable for kinetic energy. When an object is heated, I can see why its mass would increase: the increased temperature means the atoms are vibrating faster, implying their individual masses are higher (by m(v) = γ*m_o), and thus the object has a higher mass. But for something like binding energy, I'm at a loss as to how the equation E=mc^2 can predict that CO2 would have a lower mass than a carbon atom and two oxygen atoms at the same temperature, or that an object at a higher altitude would have more mass. —Preceding unsigned comment added by 173.179.59.66 (talk) 21:02, 5 March 2010 (UTC)[reply]

You might like to ask the maths desk to help with this one
I don't see why. It is clearly a science question. --Tango (talk) 21:27, 5 March 2010 (UTC)[reply]
Yes, but it's also maths. I just felt the OP could benefit from a wider insight into his question, for example making a thread at the maths desk directing them here. —Preceding unsigned comment added by Dataport676 (talkcontribs) 21:29, 5 March 2010 (UTC)[reply]
It's only maths to the extent that all quantitative science uses maths. We don't notify the maths desk every time a science question is asked on the science desk. --Tango (talk) 21:54, 5 March 2010 (UTC)[reply]
E=mc^2 doesn't predict anything about energy, it just tells you the relationship between energy and mass. CO2 will have a lower energy than the atoms separately and E=mc^2 tells us that that means it will have a lower mass. To realise that it has a lower mass you need some results from physical chemistry, not relativity. --Tango (talk) 21:27, 5 March 2010 (UTC)[reply]
Right, but the derivation of E=mc^2 seems to be only applicable to kinetic energy. My question is how it can encompass potential energy too. 173.179.59.66 (talk) 21:45, 5 March 2010 (UTC)[reply]
There are more complicated derivations which are more general, I think. --Tango (talk) 21:54, 5 March 2010 (UTC)[reply]
E=mc^2 is applicable to all energy and mass, not just "kinetic". So potential energy stored between the bonds of a compound makes that compound slightly heavier than the atoms would be seperately. A hydrogen atom is a slightly lower mass than the sum of the mass of a proton and an electron because of the energy released because of the force of attraction between a positive and negative charge, etc. etc. Concepts like "kinetic energy" and "potential energy" aren't relevent to mass-energy equivalence. It's all just energy. So objects moving faster are heavier because of the higher kinetic energy, AND stored-up energy also generates more mass. --Jayron32 01:14, 6 March 2010 (UTC)[reply]
As you said, the derivation usually given in introductory books seems to be applicable only to kinetic energy but in fact is is applicable to all forms of energy. Please do the following gedanken experiment: Put some carbon and oxygen inside a box and burn the carbon. During the burning potential energy gets converted into kinetic energy leading to a higher final temperature. Now assume that an observer was passing by with a speed v. From the point of view of that observer the box was moving at constant speed. Since momentum is also conserved, the mass of the box must have remained constant throughout the process. That means that the mass-(kinetic energy) equivalence at the end must have come froma mass-(potential energy) equivalence at the start. Dauto (talk) 01:16, 6 March 2010 (UTC)[reply]
Interesting, I never thought of that! Out of curiosity, is this the official derivation, or is there a more "mathematical" method of coming to this conclusion (like through 4-vectors or something of the sort)? —Preceding unsigned comment added by 173.179.59.66 (talk) 07:06, 6 March 2010 (UTC)[reply]
If you want a derivation of e=mc^2 then this comes from special relativity. It comes from the fact that in natural units from a reference frame of time t E/m = dt/d(tau) where tau is proper time. Viewing in a frame t where the particle is at rest, t and tau are the same leading to E=m or E=mc^2 in conventional units. See Mass–energy_equivalence#Background 82.132.136.207 (talk) 01:00, 7 March 2010 (UTC)[reply]

Protein architecture, motif, domain, fold, topology.

I'm taking a 4th level biochemistry class titled Protein structure and function and, though I already learned them intro biochem courses, the way the materials are covered is much more detailed. I don't really understand the following: architecture, motif, domain, fold, topology (used in a different sense instead of the more common definition of how the transmembrane protein is embedded). Many of these terms are used interchangeable and I don't have a clear idea of the true definition of each. For example, I saw Alpha/Beta/Alpha domain being also referred to as Alpha/Beta/Alpha architecture. Also, it seems that Greek key motif is also called a Greek key domain, which can also be said to be an architecture. Also, fold and motif seem to be complete synonyms. I'm really confused. Please clarify with clear examples. (PLEASE DON'T JUST REDIRECT ME TO LINKS HAVING GENERAL DEFINITIONS OF EACH, BECAUSE I ALREADY CHECKED MANY OF THEM OUT) —Preceding unsigned comment added by 142.58.129.94 (talk) 21:43, 5 March 2010 (UTC)[reply]

Some of these terms are broader or narrower. Architecture is the most broad covering the construction from small to big. The architecture can be made up of those other components. Motif is a pattern that you can see over and over again in different places. domain is a piece of the protein that has a stable shape in itself, so if you chop it off te part will retain this shape. Domain is narrower term or architecture, and may include folds and motifs as components. Fold is pretty clear that it is a bend or change in direction, I would count it as a narrower term of motif, but some think otherwise. The term topology seems to be abused in biochemistry, referring to where parts of the protein chain is, whether embedded in the membrane or poking out. Toploogy in maths will refer more to how it is connected, loops, or multiple elements. Is that 4th year at a uni? I have never studied biochemistry in a class. Graeme Bartlett (talk) 23:31, 5 March 2010 (UTC)[reply]
First off, I'm a cell biologist, not a structural biochemist. However, I've seen and used these terms thus:
  1. Architecture - the general design of a protein, consisting of perhaps one or more domains or motifs. The architecture of the Src protein consists of an SH2 domain and an SH3 domain
  2. motif - a short, often linear, amino acid sequence that can be a target recognition sequence, e.g. LLDLD clathrin interaction motif or PxxP, SH3 binding motif where x = unimportant. These can have different possible amino acids at certain positions, e.g [D/E][G/A](0-1)F[G/A][D/E]F binds gamma ear domains of AP-1.
  3. domain - usually a larger part of a protein with a specific function and usually a certain fold/folds (i.e. its own 3D structure). e.g. SH3 domains have a consensus sequence and bind certain protein sequences containing a PxxP motif. ENTH domains are large (145 aa) domains at the N-terminus that bind membranes containing specific PIPs. BUT some people use "domain" for motif
  4. fold - usually a change in direction in the secondary structure of a protein, but it can mean a fold in the 3D shape of a protein, context dependent
  5. topology - the specific 3D shape of a protein either overall or locally, e.g. folds and pockets in a globular region.
Perhaps a structural biologist/protein biochemist can come around and weigh in. -- Flyguy649 talk 00:53, 6 March 2010 (UTC)[reply]
I dabble in structural biology, though it's not the central focus of my work. I'd like to emphasize that 'fold' in the context of structural biology usually doesn't mean the same thing as it would in a nontechnical context — it's not just a bend or a change in direction. A 'fold' in structural biology generally refers to a particular tertiary structure (at least, an arrangement of secondary structures) which may or may not have a common underlying primary amino acid sequence. For example, the ubiquitin-like proteins (including ubiquitin, NEDD8, and SUMO, among others) all share a 'beta-grasp' fold, in which a beta sheet snuggles up adjacent to (and partially around) an alpha helix. Despite having a very similar tertiary structure, the ubiquitin-like proteins have a relatively limited similarity in sequence. A 'fold' may describe the structure of an entire protein, or a common structure shared by parts of several proteins. (The beta-grasp fold, for instance, shows up in a lot of places.) TenOfAllTrades(talk) 03:34, 6 March 2010 (UTC)[reply]

Positive and negative features of CATH and SCOP

They both offer similar services, but how are they different? Can you say which is better? I checked out the tutorial manuals for CATH and SCOP but they aren't very user-friendly. —Preceding unsigned comment added by 142.58.129.94 (talk) 22:22, 5 March 2010 (UTC)[reply]

You want the article Systematic Comparison of SCOP and CATH: A new Gold Standard for Protein Structure Analysis, full text here. Cuddlyable3 (talk) 02:15, 6 March 2010 (UTC)[reply]

Atoms

Without using a microscope, would it actually be possible to see an atom using just normal magnifying glasses, perhaps using lots of them lined up to increase the zoom? —Preceding unsigned comment added by Firemansam490 (talkcontribs) 22:52, 5 March 2010 (UTC)[reply]

No. The most powerful microscopes are scanning electron microscopes. By comparison, optical microscopes have much poorer resolution. Dolphin51 (talk) 23:17, 5 March 2010 (UTC)[reply]

Uh, no. We can't even really "see" atoms with a microscope - we need a special kind of microscope (scanning electron microscope) that interprets the image for us, rather than just using lenses to make things look bigger than they are. By the time you lined up enough lenses to hope to see atoms through them, what you saw would just be a big blur anyway - you'd be magnifying the imperfections of the lenses over and over again and the image would be distorted beyond recognition far before you got to the atomic level. - AJ —Preceding unsigned comment added by 71.108.171.138 (talk) 23:22, 5 March 2010 (UTC)[reply]

If you line up good magnifying glasses in the right way you will make a microscope, but dont expect much magnification or clear view. Your best chance will be to see gazillions of atoms together making up a visible object. The wavelength of light is about 5000 times bigger than ab atom, so it is like trying to feel a grain of sand with a truck. Graeme Bartlett (talk) 23:37, 5 March 2010 (UTC)[reply]
Although it's not optical microscopy, atomic force microscopy lets people image individual molecules and even atoms. Brammers (talk) 00:34, 6 March 2010 (UTC)[reply]
Another part of this problem is that it's not at all clear what we mean by a "picture" of an atom. These scanning electron and atomic force microscopes are measuring the field surrounding the atom and turning that into a picture - but that doesn't mean that this is what an atom "looks like" any more than a congenitally blind person can imagine what someone's face "looks like" by feeling it with their hands. With quantum uncertainty and wave-particle duality effects, it's really meaningless to ask what an atom looks like anyway - even in principle. SteveBaker (talk) 03:22, 6 March 2010 (UTC)[reply]

The problem with visible light is that its wavelength is on the order of a few hundred nanometers - more than a thousand times as large as an atom. With ordinary techniques you cannot see details smaller than approximately half the wavelength. Icek (talk) 18:16, 6 March 2010 (UTC)[reply]

Anthropophobia?

I'm wondering if the following would fall under the diagnosis of anthropophobia or some other term. . . . It's a fear of people, but's it's not "phobic" in the sense of being intense or causing panic symptoms, except occasionally in crowds (at the mall or in line at the grocery store, for example). It's more of an acute discomfort in the presence of human beings in general. It's not sociopathy, because conscience and sympathy aren't lacking. It's not an inability to love or care for others as individuals - quite the contrary. It's just that the presence of even someone very well loved causes a kind of elemental discomfort, and as much as the dear one may be missed in his or her absence, there is a relief and a comfort in solitude that has NEVER been experienced in the presence of any human being under any circumstances. This is so to the point that anticipation of seeing the loved one, even after a long absence or in circumstances such as the loved one's coming home from the hospital which are otherwise incontestably a good thing, prompts mixed feelings. A person in this condition would thrive when left in solitude, but struggle to explain why simply living in close proximity to others induced a sort of emotional paralysis that made it hard to be helpful and productive, since it's hard to be taken seriously when the message they're conveying is "It's not you, really, it's just that you're human," especially when everyone's trying to convince them that what they really need is to get out and socialize more.

Is there a name for this?

It's called Social anxiety disorder.

Wouldn't social anxiety disorder be more about inability to deal with what are conventionally thought of as "social" situations (school, the workplace, gatherings of friends), rather than desiring to hide out from your own mother or spouse?

From our Schizoid personality disorder article: Schizoid personality disorder (SPD) is a personality disorder characterized by a lack of interest in social relationships, a tendency towards a solitary lifestyle, secretiveness, and emotional coldness. Is that close? Googling "schizoid" even yields message boards populated by people who claim they have this order, and discuss their behavior and feelings. (By the way, please type ~~~~ after each post, to sign it, so we can keep track of who is saying what.) Comet Tuttle (talk) 23:53, 5 March 2010 (UTC)[reply]

In the movie Men in Black the morgue doctor Laurel Weaver (Linda Fiorentino) has this line "I hate the living." Cuddlyable3 (talk) 01:48, 6 March 2010 (UTC)[reply]

Almost all of these 'phobia' words are made up and used indiscriminantly (check out reference 2 in List of phobias for example!). Basically, almost all phobias are basically anxiety disorders - but with different triggers for that anxiety that are probably as varied as the people who suffer from the disorder. SteveBaker (talk) 03:15, 6 March 2010 (UTC)[reply]

Changed the earth's axis??

This week's TIME magazine reports that earth's day has been shortened by 1.26 microseconds as one result of Chile's earthquake, and the reason is because the earth's axis got shifted about 3 inches.

Could someone expand on this, provide a little more detail on how one caused the other? The only explanation I can imagine -- that the diameter of the earth is now slightly smaller due to subduction -- doesn't make ANY sense at all! DaHorsesMouth (talk) 23:16, 5 March 2010 (UTC)[reply]

You are basically correct, the Earth's moment of inertia was reduced by about 15 parts per trillion as dense rock subducted deeper into the earth and displaced less dense rock (and/or changed the shape of the sea floor). Along 700 km of the fault, the Earth jumped up to 10 m, which is enough to have a tiny but perceptible effect of the Earth as a whole. Dragons flight (talk) 01:38, 6 March 2010 (UTC)[reply]
And, when you move the mass inward on a spinning body, it spins faster, resulting in a shorter day, due to conservation of (angular) momentum. StuRat (talk) 02:51, 6 March 2010 (UTC)[reply]
But how would that shift the earth's axis? 95.112.175.41 (talk) 09:02, 6 March 2010 (UTC)[reply]
The stuff about the Earth's "axis" is a bit of a red herring, and not surprisingly most of the newspapers have got the story confused. The BBC's version is particularly bad, showing a completely irrelevant diagram of the precession of the axis. If Richard Gross of NASA JPL is to believed, this is what happened: the Earth's mass distribution shifted, as Dragons flight correctly said. This shift had two effects. 1: the Earth's "figure axis" (the one that passes through its centre of mass) shifted by a few metres; 2: the rotation sped up due to the reduction in the moment of inertia. Effect 1 is not related to the length of the day, although I guess it might cause a few places on Earth to shift time zones by a few nanoseconds. Effect 2 is what (is predicted to have) shortened the day by a microsecond. --Heron (talk) 10:28, 6 March 2010 (UTC)[reply]
Sorry, can't find anything about shifted axis in Richard Gross article. Wouldn't that be even violating momentum conservation? 95.112.175.41 (talk) 11:51, 6 March 2010 (UTC)[reply]
I can't remember exactly where I got that claim from, but there are plenty of other examples on the web. For example, this similar article by National Geographic attributes the figure axis shift to Gross. --Heron (talk) 15:36, 6 March 2010 (UTC)[reply]
Now, well, that's better. Not the rotation axis has shifted but the figure axis. 95.112.175.41 (talk) 19:33, 6 March 2010 (UTC)[reply]

voltage

what is the maximum voltage that can be artificially generated within the limitations of current technology? —Preceding unsigned comment added by 129.67.118.243 (talk) 23:59, 5 March 2010 (UTC)[reply]

I aksed my science teacher this once, they said that voltage is infinite it's current that matters. You could have a balizzion volts with no current and it wouldn't kill you, or you could have 1 volt with a balizzion currents and it'd fry you like a deep fried mars bar. —Preceding unsigned comment added by Zonic4 (talkcontribs) 00:03, 6 March 2010 (UTC)[reply]
Some modern Van de Graaff generators can create a potential difference of up to 5 megavolts (that's 5 000 000 volts), Our high voltage article says that lightning can have a potential difference of up to 1 gigavolt (1 000 000 000 volts) (though I don't see a reference, and that isn't artificial). Buddy431 (talk) 01:03, 6 March 2010 (UTC)[reply]
According to our Van de Graaff generator article, the record is 25.5 million volts at Oak Ridge, although that particular statistic is uncited. The limiting factor is the breakdown voltage of the insulation between the collector dome and the rest of the structure - the Oak Ridge machine (and others like it) use sulphur hexafluoride. Tevildo (talk) 01:14, 6 March 2010 (UTC)[reply]
(edit conflict)Voltage merely means that you have two points in space with electrons at different levels of potential energy. When the potential energy difference becomes great enough, the electrons are "forced" to move from the higher energy state to the lower one (i.e. generate a current). Once the current starts to flow, the voltage will lower slightly, until the current reaches a steady state, and the system reaches equilibrium according to Ohm's Law. However, back to the main question, there is no theoretical upper limit to voltage, this article at google news: [5] shows that 5,000,000 volts was attained in 1923. We can only assume the number to be orders of magnitude higher today. I can't find anything else using google, but if you play around with search terms, you may be able to find something. --Jayron32 01:09, 6 March 2010 (UTC)[reply]
I would say tens of megavolts for a short period: but thats just my gut feeling. No evidence Im afraid.--79.68.242.68 (talk) 01:12, 6 March 2010 (UTC)[reply]
Plasma acceleration has shown a transient electron acceleration equivalent to 40 billion volts, but the process is not really the same as the way we usually think about voltage, and field only exists for about a nanosecond. Dragons flight (talk) 01:25, 6 March 2010 (UTC)[reply]
An Electron microscope built[6] at Osaka University uses a 3 000 000V supply from a Cockcroft–Walton generator. Cuddlyable3 (talk) 02:01, 6 March 2010 (UTC)[reply]
I was hoping to find answers in Orders of magnitude (voltage) - but the article didn't exist, so I had to write it - which means that there is nothing there that hasn't already been said. SteveBaker (talk) 03:08, 6 March 2010 (UTC)[reply]

There is some confusion above between electric fields and voltage, which is the antiderivative of the electric field. There are limits on the strength of an electric field because any material will undergo dielectric breakdown at some point, but it is difficult to set a limit on voltage. If there is one, it would arise from limited ability to concentrate charge within a finite region -- but the fact that black holes can be charged means that means that they can at least accumulate charge up to the point where gravitational attraction is outweighed by electrostatic repulsion. Looie496 (talk) 17:57, 6 March 2010 (UTC)[reply]

March 6

Genetics and cousins

If a man marries a woman then his brother marries the woman's sister and both couples have children, how many genes do the children share with their cousins compared to regular cousins? --124.254.77.148 (talk) 02:21, 6 March 2010 (UTC)[reply]

They would be double cousins. That article says "Genetically, they are as related as half-siblings, sharing 25% of their DNA (a coefficient of relationship of 1/4)." --Tango (talk) 02:26, 6 March 2010 (UTC)[reply]
But, of course, that's just an average. They could actually share anywhere from 0% of their chromosomes (unless they are both male, in which case the cousins must share a Y chromosome) to 100%, which would also require that their two sets of parents were 100% genetically identical. Of course, the extremes are extremely unlikely, but exactly 25% is unlikely, too. (Or should I say impossible, since 46 chromosomes aren't divisible by 4.) StuRat (talk) 02:45, 6 March 2010 (UTC)[reply]
It is more complicated than that since chromosomes don't stay intact - genes can move around when the chromosomes are splitting. See Chromosomal crossover. --Tango (talk) 02:55, 6 March 2010 (UTC)[reply]
Also their parents wouldn't have to be genetically identical. Half of the parents' chromosomes aren't passed on to the children and that discarded half wouldn't have to match. Rckrone (talk) 19:24, 7 March 2010 (UTC)[reply]
Yes, you're correct, they would need to at least 50% genetically identical. StuRat (talk) 00:20, 8 March 2010 (UTC)[reply]

Please note that there is absolutely nothing tabboo about this arrangement (two siblings marrying, respectively, two other siblings), it is not incestuous or dangerous or anything like that. 82.113.121.104 (talk) 10:07, 6 March 2010 (UTC)[reply]

Except, apparently, in West Virginia and North Carolina. --80.177.170.180 (talk) 11:15, 6 March 2010 (UTC)[reply]
[citation needed] --Tango (talk) 18:34, 6 March 2010 (UTC)[reply]
80 is just slightly confused I believe. Our article linked above mentions double first cousin marriages are forbidden in West Virginia and North Carolina which otherwise allows first cousin marriages. This isn't what's being discussed here however (the OP asked what is the relationship of children not whether the children of the relationships are allowed to marry and 82 didn't discuss that either). 82 is probably right that in most or all countries there's nothing illegal about marriages that result in double first cousins (i.e. siblings marrying siblings) even if the children/double first cousins can't marry in a few places. Even identical twins marrying identical twins likely isn't illegal in most or all places although given that their children would be genetically basically siblings, probably can't marry in many places. Of course, you also get the case where after divorce or death, a person may marry the former spouse of their sibling, in some cases it was expected upon death at least for a widow (i.e. Levirate marriage) Nil Einne (talk) 19:42, 6 March 2010 (UTC)[reply]

:Each of my grandfathers had some double cousins, and I've turned out orl write normal. --80.177.170.180 (talk) 11:14, 6 March 2010 (UTC)[reply]

red shift versus blue shift

Redshift and blue shift

In considering red shift and blue shift would not it make more accurate or reliable not to think in terms of eyes or ears as being stationary and the object moving as denoted by oneway pointers but rather as both observer and object being subject to motion using dual pointers such as <<<<<<>>>>>> and >>>>>>><<<<<<<? 71.100.11.118 (talk) 03:03, 6 March 2010 (UTC)[reply]

If you are talking about redshift of light in vacuum than it makes no difference. Dauto (talk) 03:12, 6 March 2010 (UTC)[reply]
I'm including a moving siren in air that passes me while riding down the highway. 71.100.11.118 (talk) 03:16, 6 March 2010 (UTC)[reply]
Than yes, you are right. It makes some difference whether the source, the observer, or both are moving.Dauto (talk) 04:05, 6 March 2010 (UTC)[reply]
For sound...yes...because the medium that it's passing through is "stationary" so the symmetry between moving observer and moving source is broken. But the redshift in light doesn't do that because in a vacuum, there is no stationary 'reference'. Light is weird. SteveBaker (talk) 06:53, 6 March 2010 (UTC)[reply]
Obligatory reading: the article Doppler effect. Cuddlyable3 (talk) 00:29, 7 March 2010 (UTC)[reply]

red shift and blue shift for objects in the Universe

Is the average amount of red shift the same in opposite directions when observing from Earth or slightly less or more in any particular direction? 71.100.11.118 (talk) 03:14, 6 March 2010 (UTC)[reply]

I'm not really sure what you mean by the average amount of redshift - what are you averaging over? The Cosmic Microwave Background radiation is redshifted more in one direction than the other (see Cosmic Microwave Background#CMBR dipole anisotropy), if that helps. --Tango (talk) 04:42, 6 March 2010 (UTC)[reply]
For instance perhaps in a binary star system where the red shift will vary in degree and possible to the extent of going int the blue (assuming the system were close enough, the orbital speed high enough, etc.) 71.100.11.118 (talk) 05:24, 6 March 2010 (UTC)[reply]
It would depend what you take the average over. For example you could quantify the amount of redshift in terms of the relative velocity of the object, and then the average value for an orbiting object would be the same as the value for an inertial object with the same average velocity by definition. If you're averaging over the Doppler factor then in general that wouldn't work since the Doppler factor isn't linear in the relative velocity. Rckrone (talk) 19:11, 7 March 2010 (UTC)[reply]

Freeze data outside U.S.

Is it possible to find information on the dates for the first freeze in the fall and the last freeze in the spring for places outside the U.S.A.? All my effort so far has yielded no fruit - all the information I can find (online at least) seems to be limited to the U.S.A. which seems very strange - don't gardeners in, say, Romania or Korea need to know just as well?

69.140.13.88 (talk) 03:45, 6 March 2010 (UTC)Nightvid[reply]

Even the data available for the USA might not be reliable during the oncoming of global warming. For instance I've lived in Florida since 1951 and I've never experienced more than two weeks of temperatures below 40 Deg F. This year we still have temperature down to freezing in early March. 71.100.11.118 (talk) 04:03, 6 March 2010 (UTC)[reply]
Here's a very specific example for a small city in Germany [7]. Aaadddaaammm (talk) 11:09, 6 March 2010 (UTC)[reply]
Northern England (UK): earliest autumn freeze in the last fifty years was September 1st when a whole row of my still-flowering hydrangeas turned a surreal black! Night-frosts occur well into June (summer here), but I don't have exact dates. Parts of Scotland will fare worse. This site [8] might be of interest. Dbfirs 12:15, 6 March 2010 (UTC)[reply]

Static electricity damage

I recently discharged static electricity accidentally on a stereo. It's not a very high tech one but ever since i was unable to power it on. When plugged in, the 12:00 keeps flashing, that's about it. Any way an electronic device like this can be repaired? What component is likely damaged? Maybe there is something I can replace inside? Thx.

Our article has only one line about this topic. --Kvasir (talk) 05:02, 6 March 2010 (UTC)[reply]

You'd have to take it to an electronic repair shop. Once static electricity "sparks" or discharges, it becomes an electric current (and thus no longer static). Sending a pulse of unregulated electrical current through a device with hundreds of tiny electronic devices and hundreds of more little connections and wires means the current could have literally "fried" anything. There are hundreds of options as to what you fried, and there is no way, without carefully analyzing the device, to identify which little connection or tidbit or doofulator you fried. It would only take one. So, you'll need to find someone that speciallizes in repairing these devices, though theres a better than 50/50 chance that the stereo is unfixable even by an expert. --Jayron32 05:07, 6 March 2010 (UTC)[reply]
Beyond the 50/50 chance of being unrepairable, there is a much higher chance that a brand new one will be cheaper than the cost of repairing the old one. Electronics keep getting better and cheaper. Many electronic devices are now considered nearly disposable, like cell phones and mp3 players. -- kainaw 05:12, 6 March 2010 (UTC)[reply]
Static damage is very insidious. It can cause chips to fail intermittently - or not fail at all for weeks after the event and then suddenly die. You could easily pay a stack of money to repair it - only to find that it fails again soon after. Also, diagnosing these faults is pretty tough so unless they do a "repair-by-replacement", swapping out all of the electronics in the unit - the cost of figuring out which chips died would be very hard. Electrostatic discharge may be of some interest. The most likely components to fail are the chips - knobs, buttons, wires, connectors, circuit board tracks, discrete transistors, resistors, capacitors, diodes and coils are unlikely to fail - and the fact that the display is flashing says it's not the display or the power supply. So probably there are a couple of chips in there - and one or more of them is 'fried'. Replacing the chips one at a time would be the most likely approach to succeed - but honestly, I agree with the others...much as it pains me to say so: forget it - buy a new one - contribute more nasty electronics waste to your local landfill. :-( SteveBaker (talk) 06:43, 6 March 2010 (UTC)[reply]

Most electrical devices have gold film to conduct the static to the sites of the device and way from the chips. This sometimes results in a slight shock when touching them, but protects the internal operations. —Preceding unsigned comment added by VCRVLC1010 (talkcontribs) 17:22, 6 March 2010 (UTC)[reply]

Old school trouble shooting: I might inject a signal in the audio output and if sound comes out the speaker, work back stage by stage until the bad one is found. No one should tinker with AC powered equipment unless he is trained in electrical safety. The spark might have fried the portion of the circuit which was touched (if the antenna, then the RF stage; if an RCA audio connector, then the audio stage. With soldered in ICs I wouldn't bother (because I am not good at unsoldering them). Once upon a time it was straightforward to replace a transistor. Edison (talk) 21:11, 6 March 2010 (UTC)[reply]
Thanks guys. I think it would be a fun project to diagnose the problem, knowing that it's *possible* to fix. I mean wouldn't hurt to play with it when you know you are prepare to replace it anyway, right? --Kvasir (talk) 08:57, 7 March 2010 (UTC)[reply]

Relativity

To my understanding, Einstein said that, in order for the laws of physics to remain consistent with the principle of relativity and the universality of the speed of light, a moving object must undergo length contraction, time dialation, and mass increase. If we were to view an object as consisting of a collection of charges, would it be possible to also derive the same results via electromagnetic theory? I saw the article on electromagnetic mass, which said that the velocity dependancy of the electron's mass was predicted before relativity using E&M, so I was wondering if the same was possible with time dialation and such, or whether they are considered to be beyond the perview of E&M theory. —Preceding unsigned comment added by 173.179.59.66 (talk) 07:15, 6 March 2010 (UTC)[reply]

None of the effects that you claim (length contraction, time dialation, and mass increase) actually occur in the reference frame of the moving object. An observer on the moving object would find everything normal, but would observe that the rest of the universe appeared distorted. (Sorry I don't know enough about E&M to answer your question, but Steve or some other expert will give a better answer.) Dbfirs 12:28, 6 March 2010 (UTC)[reply]
Let me try. You could solve the Maxwell equations for an electromagnetically bound system, say a low-mass negative charge orbiting a stationary high-mass positive charge (note: such orbits are not stable but that isn't important here). Then, you solve the Maxwell equations again for the same system, only now it's moving at a constant speed in a constant direction. Obviously, you don't get to use the Lorentz transformation to simplify your problem (that would be assuming your answer), no, you must solve it in the original frame of reference by using, among other things, a current rather than a stationary central charge. Since you solved it in the original frame of reference, you will find that your solution is length-contracted relative to your original non-moving solution. —Preceding unsigned comment added by 83.134.176.244 (talk) 12:52, 6 March 2010 (UTC)[reply]
Yes, and even Einstein himself derives "time dilation" from an argument based on a "light clock" (i.e. light bouncing back and forth between mirrors) which can be seen as a system comprised of an EM field obeying Maxwell's Equations with reflecting boundary conditions. If you want, those boundary conditions can be derived from microscopic physics based upon the Lorentz force of the light's EM fields on the charges in the conductor/mirror, which in turn reradiate. All of this could have been done with pre-1905 electromagnetic theory which is nonetheless fully relativistic! 69.140.13.88 (talk) 16:09, 6 March 2010 (UTC)Nightvid[reply]
Cool, thanks! —Preceding unsigned comment added by 173.179.59.66 (talk) 16:45, 6 March 2010 (UTC)[reply]
It's also relevant to note that Einstein's 1905 paper, in which he set out the principles of special relativity, was entitled "On the Electrodynamics of Moving Bodies" (emphasis added). See History of special relativity for more detail on the origins of the theory. Tevildo (talk) 21:23, 6 March 2010 (UTC)[reply]

Circular DNA

In published sequences of circular DNA molecules (I'm thinking chloroplast DNA specifically), how do they determine the numbering system? Ie. who/what decides which nucleotide is the first? Aaadddaaammm (talk) 11:07, 6 March 2010 (UTC)[reply]

it would make sense for the first one to be the first one that was created while the strand of DNA was being copied. 82.113.121.104 (talk) 13:18, 6 March 2010 (UTC)[reply]
That would make sense, but I seriously doubt that's the case - often which nucleotide is copied first is not known exactly. I imagine the choice is somewhat arbitrary, and made by the team who initially sequenced the DNA. They may choose a prominent feature (like the start of a gene), or they may not. For example, the start position of the E. coli genome is about a million base pairs away from the origin of replication. In the footnote to the E. coli sequencing paper (Blattner et al. Science, 277(5331) page 1453) they note that the zero position was originally defined as the first residue of the thrA gene, which had been previously used in mapping the chromosome, but that they decided to move the start position for the complete genome 189 nucleotides upstream, to include the thrA promoter with the gene. I do know that once a choice has been made for one piece of DNA (say the pBR322 plasmid), derivatives/related DNA (like the pET vector series) tend to use the same numbering system. I'll also note that even for linear DNA like nuclear chromosomes, the numbering system is arbitrary, as telomeres, centromeres, and other unsequenced/unsequencable repeat systems make a global numbering system unwieldy/impractical. -- 174.21.226.184 (talk) 16:51, 6 March 2010 (UTC)[reply]
This is more of a question than an answer: Doesn't the loop have to detach into a more conventional linear sequence in order to reproduce? With 'normal' DNA, the two sides of the helix are unzipped and replicated - that doesn't seem reasonable for circular DNA (unless it doesn't do the "double helix" thing) because you'd end up with an inextricable tangle. If my guess is right then the circle has to break somewhere when being copied - and maybe there is a special place where that has to happen? If THAT is true - then that would be the logical place to start the numbering. But I have no clue whether any of that is true - I'd like to hear the answer. SteveBaker (talk) 17:35, 6 March 2010 (UTC)[reply]
(edit conflict) Answering SteveBaker's question: Various topoisomerase enzymes either cut one strand of the double helix and rejoins it after it has rotated around the other strand or cut both strands, let another double helix pass through, and rejoin the strands. But this happens many times during the copying of the DNA. Icek (talk) 17:54, 6 March 2010 (UTC)[reply]
For general unknotting of DNA, there are a whole group of specialized proteins: the topoisomerases. (See also this link.) TenOfAllTrades(talk) 17:49, 6 March 2010 (UTC)[reply]
DNA replication forks can be generated without cutting the DNA exactly at the site of the fork, as described indirectly by Theta structure and more comprehensively in these lecture notes. That page also discusses negative DNA supercoils, which can compensate for some of the twisting related to unwinding of DNA, and which are released by the topoisomerases cited above. Rolling circle replication is another relevant concept, especially for circular bacteriophage DNA and viroids. -- Scray (talk) 19:43, 6 March 2010 (UTC)[reply]
Here's a great, freely available, textbook chapter relevant to the current discussion - specifically, it deals with various modes of DNA replication. This doesn't answer the OP, but does address Steve's question (the short answer is "no"; circular DNA does not get linearized per se, but topoisomerases are needed to relax negative supercoils and the daughter segments have to be joined to form the circular "progeny" DNA). -- Scray (talk) 20:02, 6 March 2010 (UTC)[reply]

Surviving in a Vacuum

Last nights episode of QI said that a person can survive for a short time in a vacuum, basically until they can't hold their breath anymore and die from suffocation. Is this actually true? I would have thought they'd be exploded by the lack of pressure and boil alive because the boiling point in a vacuum is much lower. Also, Stephen Fry said someone in a vacuum would simultaneously defecate, urinate and vomit because of the pressure. Is that also true?

This has been discussed many times before (well not sure about the defecate, urinate and vomit bit), you may want to search to read the previous answers. But basically Space exposure covers it well. In particular note that it's estimate people could survive about 90 seconds but holding your breath is a bad idea Nil Einne (talk) 11:14, 6 March 2010 (UTC)[reply]
And the pee, poo and puke is from Decompression sickness, linked to in the Space exposure article. Nanonic (talk) 12:44, 6 March 2010 (UTC)[reply]
The BBC should invite three representatives from the Ref Desk along to be panellists on QI. Highest-scoring copmetitors ever, I think. Vimescarrot (talk) 21:54, 6 March 2010 (UTC)[reply]
How could Steven fry still come over as the all encompassing smart arse?--Aspro (talk) 23:12, 6 March 2010 (UTC)[reply]
For this covered in fiction, watch the movie Sunshine. Aaadddaaammm (talk) 09:40, 7 March 2010 (UTC)[reply]
Cos he has all the answers on the card?

Space question

I'm trying to understand the concept of space but I just doesn't make sense to me. Space has nothing in it right? Except for planets and moons and bits of rock floating about. So why aren't those things sucked towards each other to fill the emptiness? How can there be an area with absolutely nothing in it? On earth we have air to fill the gap, but in outter space there's nothing? Please help me to understand. How can something exist when it's nothing? —Preceding unsigned comment added by W359 (talkcontribs) 16:50, 6 March 2010 (UTC)[reply]

Space (even the space between galaxies) does have very small amounts of 'stuff' in it - odd stray molecules of gas. The "air" that we breathe is also "mostly vacuum" - there are big gaps between the molecules. It's only when you get a liquid or a solid that the molecules come close to touching - and even then, the individual atoms have nuclei that are an unimaginably tiny fraction of the size of the atom - so even something as solid as a rock or a table is almost all empty space. The difference between what we call "air" and what we call "vacuum" is just a matter of degree. The reason the air stays in our atmosphere is the same as the reason we stay stuck on the ground - gravity. The reason that gravity doesn't suck all of the planets into the sun is because the are orbiting and as they whirl round in their orbits, the centrifugal force of that motion exactly counters the gravity of the sun. The sun and other stars don't get sucked into the giant black hole in the middle of our galaxy is because they too are orbiting. Galaxies sometimes do collide with each other and get pulled apart by each other's gravity. That's believed to be happening to our galaxy right now. But what (in general) stops galaxies from just being smooshed together by their own gravity is because space itself is stretching as the universe expands after the Big Bang. That expansion is enough to overcome the mutual gravitation between distant galaxies simply because gravity gets so weak over such enormous distances. SteveBaker (talk) 17:26, 6 March 2010 (UTC)[reply]
When you ask why things aren't sucked together to fill the emptiness, perhaps you're referring to the fact that on Earth a vacuum tends to collapse—for example, if I suck the air out of a glass bottle, the resulting (partial) vacuum will noisily collapse the instant I remove the bottle from my lips. The reason for this is the surrounding air pressure. There's really no such thing as a "suck"—everything is a "push". So it isn't that the vacuum is sucking air into the bottle; rather, the relatively high-pressure air outside the bottle pushes its way into the bottle. In space there is no surrounding air, so there is no similar push. In fact, if I were to put a cap on an "empty" bottle here on Earth, travel into space, and then remove the cap, the high-pressure air inside the bottle would immediately push its way out of the bottle and drift off into space, rapidly spreading out until it's almost a vacuum itself (the same almost-a-vacuum that existed there already), and leaving an almost-vacuum inside the bottle. A vacuum has no inherent "sucking" power, so the vacuum of space doesn't suck stars and planets and things together; everything that we call a "suck" on Earth is really a "push" from the surrounding air pressure. The important force at work in the vacuum of space is gravity, which Steve talked about above. —Bkell (talk) 18:06, 6 March 2010 (UTC)[reply]

Last night, I started the Orders of magnitude (voltage) article (I love the 'orders of magnitude' series of articles - they are really handy for answering Ref.Desk questions!). I've found good examples for every order of magnitude from 100V (a AA battery) up to 108V (typical lightning bolts) which is probably about the largest voltage out there. So we're good for the positive orders of magnitude. But what I'm missing is good examples (preferably common/not-obscure) of voltages of 0.1V downwards (preferably with something I can reference). Help! SteveBaker (talk) 17:49, 6 March 2010 (UTC)[reply]

Nano volts? Each is a minute capsule filled with a gel-like substance excreted by the shark, sensitive to electrical discharges as small as .005 microvolts... [9] and Ampullae of Lorenzini --Aspro (talk) 18:29, 6 March 2010 (UTC)[reply]
How much voltage is generated by a radio antenna (or the antenna in a cell phone, GPS receiver, or wireless networking card)? I can't find the answer immediately, but it can't be very much. Certainly less than a volt, I would think. —Bkell (talk) 18:37, 6 March 2010 (UTC)[reply]
Yeah - I wondered about that - it seemed like it ought to be a great example at the very low end of the scale. SteveBaker (talk) 19:35, 6 March 2010 (UTC)[reply]
As our EEG article says, typical EEG voltages measured from the scalp are in the 10µV to 100 µV range. Looie496 (talk) 18:41, 6 March 2010 (UTC)[reply]
The voltage imposed on a portable radio's antenna or a TV's rabbit ears is extremely low. The output of a typical microphone is extremely low. The voltage from Pioneer spacecraft received at the antenna is very low. The voltage imposed in a short wire by moving it in the Earth's magnetic field is quite low. I say "quite low" because I do not have time right now to go from deciBels and impedance to voltage or to do the basic physics calculations. Edison (talk) 21:04, 6 March 2010 (UTC)[reply]
The dBm article contains a suitable list of typical radio-frequency powers - conversion from dBm to volts is left as an exercise for the reader. :) Tevildo (talk) 21:26, 6 March 2010 (UTC)[reply]
Thermocouple voltages are typically in the micro Volts. This might be intersting to your average reader. Best of luck! 65.209.6.2 (talk) 22:39, 6 March 2010 (UTC)[reply]
See the article Johnson–Nyquist noise. A 50 ohm resistor at room temperature delivers upwards of 1 nV = 10-9 V r.m.s. Cuddlyable3 (talk) 00:19, 7 March 2010 (UTC)[reply]
Electronvolt? 82.132.136.207 (talk) 01:21, 7 March 2010 (UTC)[reply]
An electronvolt is a unit of energy/mass, not a unit of electrical potential (voltage). Its dimensions are "charge" * "potential difference"; so its an energy unit, just like Joule = Coulomb * Volt. --Jayron32 01:23, 7 March 2010 (UTC)[reply]
A bit like trying to use light-years to measure time. Tonywalton Talk 01:30, 7 March 2010 (UTC)[reply]
You also get a voltage due to random electrical thermal noise, this will be of the same sort of order as the weakest radio signal you can detect in an antenna. Also voltages from thermocouples are under 1 volt. Graeme Bartlett (talk) 06:17, 7 March 2010 (UTC)[reply]
Thermal noise voltage is Johnson-Nyquist noise already mentioned.Cuddlyable3 (talk) 18:01, 7 March 2010 (UTC)[reply]

Steve, we already have a list of voltages at Volt#Common_voltages. Maybe you should merge the two lists. --Heron (talk) 10:49, 7 March 2010 (UTC)[reply]

There is certainly an argument for replacing the list in Volt with a link to the new article - but there is a family of "Orders of Magnitude..." articles that have value simply from being in the same format and with common names...so merging my new article into Volt would not be acceptable. Having orders of magnitude articles for all of the common units (mass, length, time, etc) has huge value. SteveBaker (talk) 23:11, 7 March 2010 (UTC)[reply]

Uncommon features of a protein

How do you know if a feature of a protein is uncommon or common among other proteins? By features, I mean things like a binding interface made up mostly of hydrophilic residues, a disulfide bond connecting two monomers in a dimer, etc. —Preceding unsigned comment added by 70.68.120.162 (talk) 18:01, 6 March 2010 (UTC)[reply]

Hydrophilic or hydrophobic areas can be determined from the DNA sequence, which is the most easily accessible information. Long-distance interactions are much more difficult to detect. Looie496 (talk) 18:44, 6 March 2010 (UTC)[reply]
While the presence or absence of hydrophobic and hydrophilic residues (or, more likely, their relative abundances) can be readily established from the gene sequence, that information almost certainly won't tell you whether those hydrophobic residues are surface-exposed or packed snugly in a globular protein's hydrophobic core. One doesn't know if hydrophobic residues, separated in the primary sequence, are grouped together to form a binding region on the folded protein. Similarly, one can't easily determine from the primary sequence whether or not a cysteine residue is free or if it forms a disulfide bridge, nor if it is on the surface as part of a dimerization motif. (And you're totally out of luck if you're hoping to predict something really weird like this: a heptasulfane bridge connecting two monomers in SOD1.) That said, there are tools for secondary structure prediction (and limited tertiary-structure guessing); see the ExPASy link below. TenOfAllTrades(talk) 19:10, 6 March 2010 (UTC)[reply]
For a curated database of protein structures, you might try SCOP ([10]) or CATH ([11]) — though it sounds like you're most interested in inter-protein binding...? Perhaps someone else here has suggestions for databases of protein-protein interaction motifs and post-translational assembly of protein complexes...? I might recommend looking through the list of tools linked from ExPASy here to see if there's anything that does what you're looking for. TenOfAllTrades(talk) 19:10, 6 March 2010 (UTC)[reply]

To answer the basic premise of the question, "how can you find out if a feature found in one instance is common in all instances", testing testing and yet more testing while recording the results of all the tests and building up a database of information which can be queried for correlations. —Preceding unsigned comment added by Bartender, bloodwine! (talkcontribs) 22:21, 6 March 2010 (UTC)[reply]

The answer will depend on how you classify your features, if you use a fine grained classification there will be only a few things in it, whereas broad schemes will include many members. If you stick with features named in literature or Wikipedia you can expect that there will be many members. Graeme Bartlett (talk) 04:17, 7 March 2010 (UTC)[reply]

perfect circle

Is there is no such thing as a perfect circle in nature? —Preceding unsigned comment added by RiseWave (talkcontribs) 19:18, 6 March 2010 (UTC)[reply]

The word 'perfect' is a Value judgment.--Aspro (talk) 19:37, 6 March 2010 (UTC)[reply]
No, a perfect circle is defined as a figure which has all points on its edge the same distance from its center. It's not based on a value judgment. --70.250.214.164 (talk) 20:23, 6 March 2010 (UTC)[reply]
Measured from the centre with what ? A knotted string, ears of barely corn, ... ect.--Aspro (talk) 20:59, 6 March 2010 (UTC)[reply]
Maybe the question should be better phrased '“have human engineers matched or exceeded the Accuracy and precision of circles that occur in nature? “ '(As for my example of 'perfection': I think octopus rings are 'perfect' circles of deliciousness unmatched (as yet) by any processed food manufacturer -but that is just my own value judgement based on my own pallet).--Aspro (talk) 21:45, 6 March 2010 (UTC)[reply]
The geometric circle is an abstraction in the geometry introduced in the books of Euclid's Elements. It does not exist as a physical object. Buckminster Fuller observes in Synergetics that it is unrealizable.
...We find local spherical systems of Universe are definite rather than infinite as presupposed by the calculus's erroneous assumption of 360-degreeness of surface plane azimuth around every point on a sphere. All spheres consist of a high-frequency constellation of event points, all of which are approximately equidistant from one central event point. All the points in the surface of a sphere may be interconnected. Most economically interconnected, they will subdivide the surface of the sphere into an omnitriangulated spherical web matrix...--Synergetics 224.07 Cuddlyable3 (talk) 00:11, 7 March 2010 (UTC)[reply]
The problem here is the word "perfect" - absolute perfection is impossible in the real world because the world is ultimately made of lumpy molecules with fuzzy edges and all sorts of wiggling with temperature and they have this annoying habit of moving when you try to measure their positions accurately. So there is no such thing as a perfect anything - in nature or otherwise. We talk about the perfect symmetry of snowflakes (they aren't even close to perfectly symmetrical) or that light moves in straight lines (well, not if there is anything with mass causing gravitation anywhere within an infinite distance). However, if you're prepared to live with some very tiny imperfections, nature can come pretty close. Drop a small rock in a still pond and the ripples will move outwards in almost perfect circles. SteveBaker (talk) 00:17, 7 March 2010 (UTC)[reply]
Right, in sufficiently deep-water pond the ripple from a small rock form a near-perfect circle. Also, halos around sun or moon are usually near-perfect circles. A rainbow also has a shape of a circular arc (not a complete circle, though). In still air the soap bubbles are near-perfect spheres. "Near-perfect" really means that the size of the deviations -- stochastic or systematic -- from the perfect circular shape are much smaller than the circle radius. Some of it is indeed due to the "fuzziness" of edges, to thermal motion, to medium anisotropy, to spontaneous symmetry breaking in a general sense, and so on. --Dr Dima (talk) 04:34, 7 March 2010 (UTC)[reply]
Is the sun a perfect circle within the "noise smaller than radius" train of thought? Aaadddaaammm (talk) 09:42, 7 March 2010 (UTC)[reply]
The best freehand drawer of circles (video). Cuddlyable3 (talk) 17:53, 7 March 2010 (UTC)[reply]

Convergent evolution

Do species via convergent evolution have similar DNA because they evolved to be the same, even though they have different ancestors? —Preceding unsigned comment added by Firesidedragon (talkcontribs) 21:25, 6 March 2010 (UTC)[reply]

No. There's lots of ways that DNA could work to produce the same results. A superficial similarity in physical appearance does not mean any genetic connection. --Jayron32 22:09, 6 March 2010 (UTC)[reply]
A similar phenotype can be expressed by different genotypes. The farther apart two species are, the more likely convergent evolution does not result in the same physical features. For example, a bat's wing is very different from a bird's. Imagine Reason (talk) 22:43, 6 March 2010 (UTC)[reply]
And hummingbirds wings are way different from bees, even though both can hover in a flower. StuRat (talk) 05:14, 7 March 2010 (UTC)[reply]
Yea, convergent evolution usually refers to phenotypes, but in principle there's no problem with convergent DNA sequences. Aaadddaaammm (talk) 09:44, 7 March 2010 (UTC)[reply]

Appropriate Technology Film

Hi

I saw an excellent film about 1987 produced I think by the BBC and a United Nations agency looking at the use of appropriate technology in a number of different areas: housing, birth control (Spain), using simple medical workers in a Far Eastern country etc.

I have been unable to trace its name and production details. I would like to get hold of it and find out if there has been a follow up.

If you can help with the these details it would be much appreciated. If not you may be able to refer me to someone at the BBC who may be able to assist.

Thanks —Preceding unsigned comment added by Michael in Dublin (talkcontribs) 23:32, 6 March 2010 (UTC)[reply]

I found a list of videos about appropriate technologies here. You may find a clip from the film you saw. Cuddlyable3 (talk) 23:47, 6 March 2010 (UTC)[reply]
You might find it by using the advanced search of the BFI website [12] --80.176.225.249 (talk) 23:51, 6 March 2010 (UTC)[reply]

March 7

Saturn and Jupiter

If the mass of Jupiter and Saturn were combined would there be enough mass, density and gravity to result in fusion? 71.100.11.118 (talk) 01:41, 7 March 2010 (UTC)[reply]

No. See Brown dwarf - the theoretical minimum size of a star in which fusion occurs is about 75 times the mass of Jupiter, and the smallest known star is AB Doradus C with a mass 93 times that of Jupiter. Saturn's mass is only about 0.3 times that of Jupiter, so adding them together wouldn't make a significant difference. Tevildo (talk) 01:56, 7 March 2010 (UTC)[reply]
At 75 times the mass of Jupiter, fusion occurs. At more than 13 times the mass of Jupiter, a star becomes a brown dwarf and produces heat and light by the star shrinking under its own weight. --The High Fin Sperm Whale 03:03, 7 March 2010 (UTC)[reply]
According to our article, "brown dwarfs heavier than 13 MJ do fuse deuterium and those above ~65 MJ also fuse lithium." Both isotopes/elements however would be present in far smaller proportions than the hydrogen that begins to be fused from 75 MJ upwards, so what fusion does occur is thought to be unable to prevent the brown dwarf from cooling quite quickly (by stellar standards) below fusion-sustaining temperatures. 87.81.230.195 (talk) 06:21, 7 March 2010 (UTC)[reply]

is Jupiter made of the right stuff to be a star, if it were bigger? If so, then is the only difference between a planet like Jupiter and a Sun that the former is too widdle to fuse? Instead of planets then why don't we call them little not-stars? 82.113.121.94 (talk) 11:49, 7 March 2010 (UTC)[reply]

You find the reason why we call them planets in the lead and the first section of the article Planet. DVdm (talk) 11:56, 7 March 2010 (UTC)[reply]
And yes, Jupiter is made of the right material - hydrogen and helium. --The High Fin Sperm Whale 03:32, 8 March 2010 (UTC)[reply]

classifying reaction?

What kind of chemical reaction is Cu+2H2SO4-->CuSO4+2H2+SO2

thanks in advance...

174.112.38.185 (talk) 02:46, 7 March 2010 (UTC)[reply]

see Redox, hydrogen production, solution, corrosion, pollution. Graeme Bartlett (talk) 04:08, 7 March 2010 (UTC)[reply]
I would be surprised if this reaction could really happen. 93.132.150.52 (talk) 09:30, 7 March 2010 (UTC)[reply]
The caption of one of the images on the Copper(II) sulfate page says it can be made by electrolysis. It wouldn't surprise me as it seems, at a glance, to be strongly entropically favourable, with a solid metal and six aqueous ions forming two aqueous ions and three equivalents of gas, which would then bubble off, slamming the equilibrium in favour of the products. Brammers (talk) 10:12, 7 March 2010 (UTC)[reply]
The reaction is a magical way to hide 2 atoms of oxygen. Cuddlyable3 (talk) 17:40, 7 March 2010 (UTC)[reply]
This sort of thing can actually happen when you heat copper in sulfuric acid, you may not get hydrogen produced however, getting water produced at the end instead. Same thing happens with mercury (don't try this at home). Graeme Bartlett (talk) 22:12, 7 March 2010 (UTC)[reply]
Is SO4 a molecule that could exist? If the last molecule were SO4 rather than SO2, then Cuddlyable3's objection would be satisfied. Otherwise if O2 were evolved it could balance. (I know something of electricity, less of chemistry).(Gee, I have some sulfuric acid and copper in the basement. Should I try the experiment? Naah!).Edison (talk) 23:33, 7 March 2010 (UTC)[reply]
It called a reaction that don't work :-). Copper is the "wrong" side of hydrogen in the electrochemical series. Most metals will react to give off hydrogen, copper is the odd one out. Copper only dissolves with an oxidant - nitric acid will do nicely.  Ronhjones  (Talk) 23:47, 7 March 2010 (UTC)[reply]
Pure sulfuric acid though can oxidise copper, I checked it out in an Inorganic Chemistry text book, in the sulfur dioxide section, it is one way to make SO2. Graeme Bartlett (talk) 00:33, 8 March 2010 (UTC)[reply]

star birth, life,and death equation and graph

is there an equation and graph that shows each of the basic conditions, i.e., gravity, diameter, volume density rate of fusion of a star over time for each sequence (big bang to cloud of hydrogen through the collapse of an iron core to the formation of a neutron star and eventually a black hole and beyond) over time? 71.100.11.118 (talk) 04:29, 7 March 2010 (UTC)[reply]

No. StuRat (talk) 05:04, 7 March 2010 (UTC)[reply]
How about, instead, thinking about saying "Not that I know of", which would be more accurate; and then realizing that it's better to not post than just to post either "no" or "not that I know of"? Comet Tuttle (talk) 07:30, 7 March 2010 (UTC)[reply]
How about keeping your sanctimonious pedantry to yourself? (Oh, I forgot -- there's no WP:KYSPTY rule, so you're perfectly legit.) 63.17.52.111 (talk) 11:02, 7 March 2010 (UTC)[reply]
Do you really advocate the unreferenced answer "no" for a reasonable refdesk question? That doesn't improve the refdesk at all. We are a reference desk, and we're supposed to supply references, not lazy rejections. Comet Tuttle (talk) 15:56, 7 March 2010 (UTC)[reply]
I said no because that's the correct answer. Many equations and graphs and charts and diagrams are needed to show all of that, it can't possibly all be put into one. And how exactly am I going to find a source proving that something doesn't exist ? That's just silly. If you think I'm wrong, let's see some sources supporting your position. StuRat (talk) 16:01, 7 March 2010 (UTC)[reply]
I'm not asserting that you're wrong; I'm asserting that an unreferenced "No" is an unacceptable answer on the refdesk. If you had typed the above paragraph as your original answer, I would not have complained at all. Comet Tuttle (talk) 16:55, 7 March 2010 (UTC)[reply]
It's still an unreferenced "No". Are you saying that it's now acceptable because I explained why the answer is no, or because I explained that it's impossible to provide a source for something which doesn't exist ? StuRat (talk) 20:36, 7 March 2010 (UTC)[reply]
Absolutely that would be a better answer. "No - because you can't fit all that stuff on one sheet of paper" would have been a useful answer. Particularly if you could explain why not. Just "No" is a very low-information-content response. So yes - your first answer was awful - with the clarification, it's borderline useful. The best answer would have been "No, you can't fit all of that on one sheet of paper - but here, here, here and here are the separate graphs and charts that will give you the information you need." As you will note below, the answer "Have you read out Stellar evolution article?" was judged by the OP to be useful. SteveBaker (talk) 23:02, 7 March 2010 (UTC)[reply]
They said it contained some useful info, but that it didn't answer the Q. The answer to the Q was "No". We can explain why that's the answer, but I doubt if you will find any links that say why you can't fit all that info on one page, which is what was actually asked. StuRat (talk) 00:08, 8 March 2010 (UTC)[reply]
I agree that making such a chart would be very difficult - but if you just say "No" then that might have been because the chart simply doesn't exist because nobody could be bothered to make one - or it might be because the information isn't available - or it might be because the sheet of paper would have to be too big - or (the truth) it might be because the resulting chart would be an unreadable mess. But the reason that there isn't such a chart is really very important. The purpose here is to help our questioner - not just to provide a literal boolean answer to the question. Your "No" response was true but 100% useless since one's first feeling on reading that is "How the hell could this guy possibly know that such a chart doesn't exists? He hasn't read every astronomy book in the world!" - so you aren't conveying the information needed for the answer to be remotely believable. Sure you can't find a reference for that - but there were ways you could have said something like: "There are X kinds of star and Y different parameters to be graphed for each star type so there would be X x Y lines on that graph and it would need Y columns of numbers on the vertical axis and you'd need to draw all of those curves with X x Y colors or line styles." Assuming that X and Y are reasonably large numbers - it would be clear why such a diagram would either be impossible, or really cluttered or actually unreadable. SteveBaker (talk) 04:55, 8 March 2010 (UTC)[reply]
While I agree that going beyond simply answering the question (as I did) to explain in depth how that answer was derived might be helpful, I don't agree that I should either do that or not answer. There's no requirement that each individual response be comprehensive. Thus, one person can answer and another can elaborate. I've often done the elaboration part myself. StuRat (talk) 10:26, 8 March 2010 (UTC) [reply]
How about for each sequence, including non-wiki articles? 71.100.11.118 (talk) 05:15, 7 March 2010 (UTC)[reply]
Have you read our Stellar evolution article? Comet Tuttle (talk) 07:30, 7 March 2010 (UTC)[reply]
Thanks for the ref. That article has some good stuff in it. What I had in mind though was a chart where core gravity, diameter, volume, density, rate of fusion, etc. (all of the basic measurements were plotted over life cycle and the equations that correlate rate of fusion with star diameter, etc. 71.100.11.118 (talk) 12:21, 7 March 2010 (UTC)[reply]

Life expectancy

In an interview I was watching, a man claimed that in the cave man era, if infant mortality were ignored, the average life expectancy of people would be roughly 70 years. I'm a little skeptical, is there any evidence that this is true? —Preceding unsigned comment added by 173.179.59.66 (talk) 09:19, 7 March 2010 (UTC)[reply]

The move away from nomadic hunter gather to neolithic settlements appears to have drastically increased the disease burden; which got worse as villages became towns and towns grew into cites. Things did not improve until the introduction of modern sewerage systems. There is little hard physical evidence to support the idea that palaeolithic people had longer life spans but there are good augments to suggest this: Grandmothering, menopause, and the evolution of human life histories--Aspro (talk) 11:10, 7 March 2010 (UTC)[reply]
The fact that human females are able to live for so long after they are no longer able to bear children suggests that there is evolutionary benefit to living beyond ~40 years that is nothing to do with reproductive ability. People who outlived their reproductive usefulness would consume food and other resources that the rest of the tribe needed - and that would impose evolutionary pressure for people to die at an age just beyond that at which their children became mature. People with the "die at 40" gene would have more successful children - and that gene would outperform the "die at 80" gene that would result in twice as much food consumption by a group of humans that would limit the number of surviving children into future generations.
But if that were true - then we wouldn't live much longer than 40 years...and we obviously do. Most other animals die of old age not long after they cease to be fertile. No amount of good health care, nutrition, comfortable living, etc will allow a dog to live more than about 15 years - or a mouse beyond about 3 years. But both are fertile until almost the end. So for humans to be so different, there must have been other benefits of long life (eg the ability to bear knowledge across multiple generations - or that 40 years of experience make for a better hunter than 20 years - so that the older people could produce more benefit than they 'cost').
We haven't really had time to evolve that long-life ability recently - so it was probably around in palaeolithic times - which probably means that at least some people lived to these kinds of ages. What's tough is to know the average age back then. Did just 1% live to 70 or did most people who survived childhood live that long? SteveBaker (talk) 22:52, 7 March 2010 (UTC)[reply]
The long childhood of humans meant that parents need to stay around for a much longer time after their last kid is born, if they expected it to survive. StuRat (talk) 01:47, 8 March 2010 (UTC)[reply]
Fascinating stuff, thanks for the detailed answer. By the way, is there any particular reason why diseases were more prevalent in sedentary societies? And how then were agriculture-based societies able to out-develop their nomadic counterparts? —Preceding unsigned comment added by 173.179.59.66 (talk) 01:12, 8 March 2010 (UTC)[reply]
Sedentary societies usually involve higher population densities and close contact with animals. Both of those factors contribute to increased disease. Agriculture allows people to produce more food than hunting and gathering (in some environments, anyway), which allows some people not to work on getting food. That frees them up for developing other crafts and for politics (which allows better coordination of large groups). --Tango (talk) 01:37, 8 March 2010 (UTC)[reply]

Temperature of outer space

If space is a vacuum, how can it be a temperature be associated with it? 173.179.59.66 (talk) 09:22, 7 March 2010 (UTC)[reply]

There is not usually a temperature associated with the vacuum of space. If someone does refer to the temperature of space, they usually mean the temperature of the cosmic background radiation or of the very low-density interplanetary medium or interstellar medium.83.134.173.114 (talk) 10:20, 7 March 2010 (UTC)[reply]

So the vacuum of space is temperatureless? —Preceding unsigned comment added by ApplePie456 (talkcontribs) 11:39, 7 March 2010 (UTC)[reply]
No, vacuum can still have radiation which wiil have a temperature. That would be the cosmic background radiation mentioned above.Dauto (talk) 14:14, 7 March 2010 (UTC)[reply]
The vacuum of space apparently does not hve a "temperature" other than the extremely low temperature of the cosmic background radiation referred to above. But an object in space will have a temperature, as would be measured by a thermometer in it or on its surface. A spacecraft in orbit will get very hot on the side facing the sun, depending on its absorption of the solar radiation, which amounts to something over 1 kilowatt per square meter. A white spacecraft would absorb less energy than a black one. See Albedo. The same is true for an object on the surface of the Moon where the sun is shining. Conversely, a spacecraft (or the side of a spececraft) which is in the dark will radiate off heat and can get extremely cold. NASA astronauts referred to the rotation of a spacecraft to even out the temperature "barbecue mode." Coolant can be circulated through tubes to even out the temperature. In the ill fated Apollo 13 moon mission, the spacecraft had to power down and it got very cold inside while they were travelling back to Earth after passing around the Moon. If the outside of the craft had been painted to absorb more solar energy, it would have been warmer. Edison (talk) 14:21, 7 March 2010 (UTC)[reply]
The key thing to remember about our definition of temperature is that it requires matter (unless you're talking about the background radiation mentioned above). You can only refer to conventional temperature when there is matter to study the kinetic energy of, so any floating material in the 'vacuum' of space (I use the term lightly, as there's actually quite a bit of matter floating around) will have a temperature, but the empty parts won't (except for the cosmic background radiation, see above). Regards, --—Cyclonenim | Chat  15:00, 7 March 2010 (UTC)[reply]
You say you use the term vacuum "lightly" to refer to outer space. I have read that space has a hard vacuum, equal to or better than any that can be achieved with the best technology on Earth. Maybe our vacuum pump technology has improved since I was last concerned with the quality of obtainable vacuum. Any vacuum tube would supposedly work fine there. Do you have a reference that supports your contempt for the quality of the vacuum seen in space, compared to what a vacuum pump can achieve on Earth? Edison (talk) 22:58, 7 March 2010 (UTC)[reply]
There is no reason to see the radiation temperature as any different from the temperature of matter and since radiation permeates space it is always possible to define the temperature of vacuum and by the third law of thermodynamics it is always different from zero. Dauto (talk) 20:29, 7 March 2010 (UTC)[reply]
Right. Cycloneim, this is important: Temperature per se has nothing whatsoever to do with kinetic energy per se. Temperature is defined by the relationship between entropy and internal energy. The only time you can use kinetic energy as an accurate proxy for temperature is when you're dealing with a monatomic medium, so that none of the energy goes into vibrational modes. --Trovatore (talk) 20:34, 7 March 2010 (UTC)[reply]
Actually I take back the "nothing whatsoever" — that was too strong. Certainly kinetic energy will increase when temperature does, and that in itself is some relationship.
The point I wanted to make is that many people seem to think that temperature is conceptually a matter of kinetic energy. And that's completely wrong. --Trovatore (talk) 20:46, 7 March 2010 (UTC)[reply]
Then I apologise, I'm no physicist but closer to a biologist so I suppose my definitions are very limited to molecular structures. Temperature is indeed a measure of kinetic energy in particles though, isn't it? If not, someone needs to go fix the article temperature if that's not it's primary definition, because that's the very first sentence in the lead. Regards, --—Cyclonenim | Chat  22:10, 7 March 2010 (UTC)[reply]
To the extent that the particles are like billiard balls, whose internal structure can be ignored, and which don't interact except via elastic collision, you can treat it that way. But molecules you specifically can't. Molecules have vibrational and rotational modes. The temperature gives you a unit of energy that is distributed equally among the modes, not equally among the particles. --Trovatore (talk) 00:26, 8 March 2010 (UTC)[reply]


Another way to see that the energy formulation (even when modes are considered) is inadequate is to note that some systems have negative temperature. (You can't have a whole object at negative temperature, but you can have, say, its system of magnetic spins at negative temperature.) But this certainly doesn't mean negative energy! Actually, it's higher energy than a positive-energy state of the same system. --Trovatore (talk) 00:38, 8 March 2010 (UTC)[reply]
The problem here is that there are at least two subtly different meanings for this word "temperature". Classically, it's a measurement of the speed that molecules are jiggling around in a liquid, solid or gas - and in that sense, a vacuum really doesn't have a temperature. However, you can talk about the kinetic energy of those molecules as "heat energy" which can be related to a temperature - and when you do that, you can speak meaningfully of "temperature" as a more general measure of the energy present. In that interpretation, the photons and other radiation within the vacuum "counts" in that energy measurement - so now you can talk about the temperature of a vacuum. The distinction is another one of these "what does this word mean" problems rather than some physical thing. SteveBaker (talk) 22:30, 7 March 2010 (UTC)[reply]
No, the "speed" version is really just wrong. See above. --Trovatore (talk) 00:40, 8 March 2010 (UTC)[reply]

OP here. So to make sure I understood, the temperature of outerspace near the earth would be quite high, because of the contribution of the sun's radiation, and this would be true regardless of whether there's actually any matter present. Meanwhile, far from the solar system (or any star for that matter), space is very cold, with a temperature associated with the cosmic microwave radiation. Is this correct? —Preceding unsigned comment added by 173.179.59.66 (talk) 01:30, 8 March 2010 (UTC)[reply]

Well, no, it's not that simple. "Outerspace near the Earth" is not in thermal equilibrium, so it doesn't have a single well-defined temperature. What it has, you might say, is multiple temperatures, for different subsystems that are in some sort of equilibrium. For example, the solar wind is very hot indeed (much hotter than the radiation temperature of the Sun) -- millions of degrees if I recall correctly, which I may not. The reason you don't get fried by it instantly is that it has very low heat capacity and thermal conductivity, so the interplanetary medium right next to your spacecraft quickly delivers its negligible heat and cools off, whereas the stuff just a little farther away can't get its heat to you efficiently at all, so it doesn't bother you.
On the other hand the cosmic microwave background is also there, and it's at 2.7 K (if I remember the number right), as always. --Trovatore (talk) 08:24, 8 March 2010 (UTC)[reply]

mind waves

is there any thing like mind waves? can a person just receive those mind waves? what type of energy are they really sound waves? —Preceding unsigned comment added by Myownid420 (talkcontribs) 09:50, 7 March 2010 (UTC)[reply]

You might want to read Electroencephalography. 93.132.150.52 (talk) 11:56, 7 March 2010 (UTC)[reply]
By recording cortical evoked potentials, it has been possible to read the mind to a limited extent. The Pentagon theough DARPA funded such experiments in the 1970's. See [13]. The thing being collected is very weak electrical potentials on the scalp which result from electrochemical activity in various brain regions and neural pathways. It is a very indirect measure, like determining when there are breaks in a televised sports program by noting the variations in water pressure due to toilet flushing. Edison (talk) 14:29, 7 March 2010 (UTC)[reply]
There is electrical activity in the brain - and where there is variation of electrical potentials, there are (inevitably) radio waves. However, the voltages are incredibly low and so are the frequencies - so those waves are incredibly, insanely weak - vastly too weak to make it past the skull - let alone any distance beyond that. Also, disentangling the results of a few billion neurons that are firing (seemingly) randomly is utterly impossible. The only way we have to pick this up is with sensors attached close to the scalp in the form of some kind of helmet - and even then, the signal strengths involved are on the lower edge of detectability. Because of the "firing all at once" problem, only the simplest information is obtainable in this way. We can pick up simple overall, slow speed activity this way (See: Electroencephalography).
We should VERY clearly state, however, that we can't "receive" those waves in our brains. That's absolutely and utterly impossible. Firstly we have no radio signal reception sensors in our heads (if we did, we'd be totally deafened by the insanely high powered TV, Radio, Cellphone, etc signals that are shooting through us all the time!). Secondly, the signals involved are so incredibly weak that it would be impossible - even in principle to detect them at any distance whatever. Thirdly, there is no way to extract information from the cacophany of activity within those billions of neurons without having some spatial information. So telepathy without intrusive electronics is physically quite impossible.
Having said that, an exceedingly crude form of telepathy has been implemented by Kevin Warwick (his article is worth reading). He has about 100 electrodes in an implant attached to the "median nerve" of one of his arms. A computer can measure nerve impulses that he sends that way...and he has demonstrated moving a robot arm using his mind alone. In a subsequent experiment, they implanted another set of sensors into his wife and they can now communicate by mentally activating their arm muscles and causing some kind of tingly sensation in the other person. It would be possible to learn to communicate (eg using Morse Code) by thought alone...but that's an exceedingly long way from simply "reading thoughts". This is more like a rather clunky wireless morse code transmitter that you have implanted in your body. However, "brain waves" aren't involved here - it's a conscious effort to transmit a 'tingle' and the resulting feeling is just like a tingle on the skin - not a mental image or speech or anything like that which might just "pop up" in your head.
SteveBaker (talk) 22:24, 7 March 2010 (UTC)[reply]

Corrosion of copper

Does corrosion of copper involve esterification? —Preceding unsigned comment added by Kencankan (talkcontribs) 09:58, 7 March 2010 (UTC)[reply]

You might want to read Organometallic chemistry. 93.132.150.52 (talk) 11:56, 7 March 2010 (UTC)[reply]

Cow pats

Can methan be extracted from cow pats? Why doesn't the farmers collected the cow pats and instead of putting them on rose plants as fertilizer use them for methan production? —Preceding unsigned comment added by Firendofdarkness (talkcontribs) 10:52, 7 March 2010 (UTC)[reply]

Yes, see :Anaerobic digestion. Collecting cow pats by hand however is labour intensive. It might be more economical to burn them as fuel. --Aspro (talk) 11:24, 7 March 2010 (UTC)[reply]
Not generally "cow pats" as found on grassy ground where they are grazing - but the 'slurry' that comes from concrete areas where cows live and are milked, etc is increasingly used by farmers. They can scrape it up and pump the semi-liquid mix of cow poop and pee into methane-producing "digesters" that produce power for lighting and heating cow sheds. SteveBaker (talk) 22:03, 7 March 2010 (UTC)[reply]

pus-like sinus stuff

What is the strong-smelling, pill-shaped puslike substance found in the sinuses? I know, that whole thing sounds horrible. --Chris (クリス • フィッチュ) (talk) 13:01, 7 March 2010 (UTC)[reply]

We cannot give medical advice. Have you read the article Mucus and the truly disturbing article Dried nasal mucus? Edison (talk) 14:23, 7 March 2010 (UTC)[reply]
How can pus be pill-shaped? Pus is a viscous liquid of sorts. DRosenbach (Talk | Contribs) 17:06, 7 March 2010 (UTC)[reply]
It can be pill-shaped if it's enclosed in an abscess, perhaps? Either way, go to your medical practitioner to check it out. Regards, --—Cyclonenim | Chat  17:15, 7 March 2010 (UTC)[reply]
How is it "found in" the sinuses? Cat scan? Surgery? Autopsy? Endoscopic exam? Or do you mean expelled by nose-blowing? Again, we will not provide diagnosis or medical device, but can discuss theoretical questions and refer a questioner to appropriate references such as Sinusitis, Rhinitis, Paranasal sinuses, Pus or Neti pot, dependindg on how we acting as reference librarians understand the question. Edison (talk) 23:08, 7 March 2010 (UTC)[reply]

Pineapple

I want to grow a pineapple plant. Can I just cut the top off an existing pineapple and plant it? Will it grow into a pineapple bush? —Preceding unsigned comment added by Hawclawlaw (talkcontribs) 14:45, 7 March 2010 (UTC)[reply]

I think pineapple seeds are in the lower portion of the fruit, in the interior of the fruit. I think they're pretty awkward to find in a pineapple you might have bought from the supermarket though as they're often removed. Regards, --—Cyclonenim | Chat  14:56, 7 March 2010 (UTC)[reply]
Actually, the original poster has it right. However, it's only going to grow in tropical (and maybe subtropical) regions. StuRat (talk) 15:51, 7 March 2010 (UTC)[reply]
It will grow in many places, but it will only fruit in tropical climates. --Tango (talk) 20:19, 7 March 2010 (UTC)[reply]
A Google search threw this up for me, which seems to be what you're after. I can't tell you how accurate the site is, but it seems ok to me. Brammers (talk) 15:19, 7 March 2010 (UTC)[reply]
Here's another interesting series of questions and answers about growing them. Scroll down to mid page for some good photos.Bielle (talk) 16:45, 7 March 2010 (UTC)[reply]
They'll grow anywhere as a house plant, too. I've always wanted to grow one, have tried a couple of times but has never worked. Good luck! Aaadddaaammm (talk) 17:13, 7 March 2010 (UTC)[reply]
Cut the top off with 1 inch of the fruit, plant in a very sandy soil mix, water sparingly. I've done in even in the UK - won't fruit, and won't survive any hint of frost.  Ronhjones  (Talk) 23:50, 7 March 2010 (UTC)[reply]
I seem to recall that, if you want them to fruit, you need the soil to be warm as well as the air: I saw a really interesting programme a while ago(in the UK), about how rich estates grew pineapples in England before they could be imported. Anyone know what that programme would have been? Anyway, it seemed to involve undersoil heating, because using a heated greenhouse wasn't enough. You could always try... 86.177.121.239 (talk) 00:03, 8 March 2010 (UTC)[reply]

Oldest survival animal species?

I would ask the oldest surviving species in general, but I assume someone would come back with cyanobacteria or something similar. I'm looking for the oldest surviving species in what today we consider to be in one of the major branches of animalia (e.g. mammals, fish, birds etc)? Regards, --—Cyclonenim | Chat  14:54, 7 March 2010 (UTC)[reply]

Sorry to respond with a question, but to give a reliable answer I think we'd have to agree on the definition of Species, and how we would apply that to the available evidence. We cannot attempt interbreeding, nor can we sequence enough DNA from tissue recovered from an organism that lived (say) 100 million years ago and compare in the manner that might be required for certainty today. It's quite difficult to assign species-level taxonomy using fossils from many millions of years ago. Do you have a well-founded proposal for the definition to be used, before people start proposing candidate species? -- Scray (talk) 15:12, 7 March 2010 (UTC)[reply]
Sorry, what I meant was are there any species around now that we know (from say, as you mentioned, fossil records) existed a long time ago, and which is the oldest? Clearly eukaryotic creatures are likely to have changed at least a bit in hundreds of millions of years, compared to say cyanobacteria which are remarkably stable. Regards, --—Cyclonenim | Chat  17:13, 7 March 2010 (UTC)[reply]
I still don't think I've explained the issue clearly enough. For example, if we were to kill a Tree Swallow, retain only the bones, compress them a bit, and fossilize them, would we be able to confidently differentiate that fossil from a Golden Swallow? I only choose this as an example, to illustrate how specific a species designation is. Back to your clarification, then, how can we "know" that any fossil corresponds to an extant species? Fossils provide only some of the distinguishing characteristics. Your question is great, because it challenges us to think about what a species is. -- Scray (talk) 17:23, 7 March 2010 (UTC)[reply]
So because our current definition of a species is two organism which can reproduce to create fertile offspring, that becomes irrelevant when looking at ancient animals because we have no method of studying whether their DNA is compatible? Not being an paleontologist I cannot answer you question, but presumably there are differences in bone anatomy between any two species, but I have no citation for that. If there isn't any difference physically, then clearly we would not be able to ascertain which is which. So is this search for the oldest known living animal pointless? I don't think so, since paleontologists clearly do know that, say, horseshoe crabs have been around for approximately 450 million years since the fossil record shows that the horseshoe crabs existing then are at least stupidly similar to the anatomy of those that are alive now. So, going on that method, which is the oldest surviving 'species'? Regards, --—Cyclonenim | Chat  19:36, 7 March 2010 (UTC)[reply]
This is the Science reference desk, and I'm just pointing out that getting a good scientific answer depends on defining the question. You've got some pretty interesting answers already, but to weigh them (for the superlative "oldest" you seek) the question would need to be defined. I'm not criticizing you - from the start I thought that this raised an interesting challenge. -- Scray (talk) 19:49, 7 March 2010 (UTC)[reply]
Sorry if you thought I was having a go, it wasn't supposed to come across that way. I think I just underestimated how complicated the question could be. Regards, --—Cyclonenim | Chat  22:02, 7 March 2010 (UTC)[reply]
If we go by animals that we would "recognize" as being similar to a modern species, how about the millipede, which goes back some 428 million years ? See Millipede#Evolution. StuRat (talk) 15:44, 7 March 2010 (UTC)[reply]
The brachiopod genus Lingula comes pretty close, having been around for more than 500 million years, not sure about a species though. Mikenorton (talk) 16:51, 7 March 2010 (UTC)[reply]
Theres Crocodile at 200 million years--Jac16888Talk 17:32, 7 March 2010 (UTC)[reply]
Take a look at the Crocodile article - that's a Family designation; there are 2 sub-families, 5 major genera within one of those, and about a dozen species for the most common genus. The OP asked about species - are we talking about those now, genera, families, or just living things with bones that look kind of similar to some fossil? -- Scray (talk) 17:58, 7 March 2010 (UTC)[reply]
Like millipedes, the oldest known scorpion fossil dates to about 430 million years ago. A beautiful creature, that. – ClockworkSoul 17:59, 7 March 2010 (UTC)[reply]
Tardigrade —Preceding unsigned comment added by TrialicWave (talkcontribs) 18:59, 7 March 2010
If you're after animal species that are anatomically unchanged for several million years, then the living fossil article has several examples. 62.56.68.180 (talk) 01:20, 8 March 2010 (UTC)[reply]

Extra Life expectancy for the health-concious

Are there any figures available for the extra life-expectancy of people who eat healthy foods, stay slim, exercise, do not smoke and seldom drink, compared with the average? Thanks 89.242.102.148 (talk) 16:14, 7 March 2010 (UTC)[reply]

Yes. Basically the standard life expectancy models can be extended for this, by looking at specific countries. For example, in Africa or India it's unlikely that people will be drinking and smoking heavily while consuming vast amounts of fast food, whereas in the Western world those forms of consumption are very common.
That sounds horribly imprecise. I'd expect health care access and capacity in the US, European nations, Africa, and India vary widely, among other things. I think a better study would be to talk to each region and ask individuals how much they pay attention to what they eat, how physically adept they are, and track people over their lifetime keeping this in mind. Mac Davis (talk) 19:21, 7 March 2010 (UTC)[reply]

You'll be interest in things such as Life Tables and in general Actuarial science. I don't know the stats but as MacDavis alludes to location of the individual is a huge contributory factor. I'm sure someone will be able to link some stats but not got time myself. ny156uk (talk) 23:04, 7 March 2010 (UTC)[reply]

Telescope

What is the difference between reflecting and refracting telescope? -- Extra999 (talk) 23:00, 7 March 2010 (UTC)[reply]

The optical telescope article explains this, but basically, a reflecting telescope uses mirrors, and a refracting telescope uses lenses. CS Miller (talk) 17:41, 7 March 2010 (UTC)[reply]
Or in simpler and more obvious terms; a reflecting telescope reflects light, while a refracting telescope refracts light to produce an image. —Preceding unsigned comment added by TrialicWave (talkcontribs) 18:46, 7 March 2010 (UTC)[reply]
Not just any old mirror, but one ground into a parabolic shape. For a number of reasons, the largest "visual" telescopes in the world are reflectors. Radio telescopes such as the Very Large Array also use "reflection" to gather signals. ←Baseball Bugs What's up, Doc? carrots20:25, 7 March 2010 (UTC)[reply]

Intron question

Do introns contain DNA from other species? —Preceding unsigned comment added by Celidia (talkcontribs) 18:22, 7 March 2010 (UTC)[reply]

Depending on what you mean by "DNA from other species", they can. It's not at all clear where introns originated, but some have speculated that (at least some) introns are derived from transposons (mobile DNA elements). Certain transposons (especially some retrotransposons) are thought to be derived from viruses (like retroviruses) which infected a cell, and then lost the ability to construct infectious viral particles). Viruses which insert and remove themselves from the host chromosomal DNA have been known be "inefficient" and take a chunk of host DNA with them when they leave. - The scheme goes like this: a virus excises itself from the host DNA, taking a chunk with it. It then infects a different species (which, depending on the virus, is a relatively common occurrence), inserting the other species DNA into the genome. This inserted sequence, over a very long period of time, eventually loses it's infectivity, leaving the chunk of foreign DNA in the cell. However, while this scheme sounds plausible to me, I'm not aware of any specific examples of it. - You may also be interested in the topic of horizontal gene transfer, which is the general mechanism by which DNA of one species gets transferred to another. -- 174.21.235.250 (talk) 18:55, 7 March 2010 (UTC)[reply]
And, in case you don't already know, our functional DNA also "contains the DNA of other species". We share over 98% of our DNA with chimps, for example. StuRat (talk) 20:24, 7 March 2010 (UTC)[reply]

Carbon planets

Resolved

Our article carbon planet says that the protoplanetary disc is carbon-rich and oxygen-poor. Why does it need to be oxygen-poor? Or, I suppose: What would happen if it was rich in both carbon and oxygen? Vimescarrot (talk) 18:41, 7 March 2010 (UTC)[reply]

Presumably if oxygen was present, it would react with the carbon, forming carbon dioxide. As this is a gas, it wouldn't settle on the surface to form a layer. CS Miller (talk) —Preceding undated comment added 18:49, 7 March 2010 (UTC).[reply]
And if by chance it did settle on the surface, it would cause rapid global warming because carbon dioxide is a greenhouse gas —Preceding unsigned comment added by TrialicWave (talkcontribs) 18:50, 7 March 2010 (UTC)[reply]

Thanks. I suppose what I really meant was, if it was oxygen-rich, why couldn't it still be a carbon planet? Why did Marc Kuchner think they need to exclude oxygen for it to be a carbon planet? Vimescarrot (talk) 19:37, 7 March 2010 (UTC)[reply]

If it contained both lots of carbon and oxygen, like Earth, wouldn't it develop in a similar manner ? I think the point is that carbon only develops into diamonds and such in the absence of oxygen, which is why diamonds are rare, here on Earth, where such oxygen is plentiful. StuRat (talk) 20:19, 7 March 2010 (UTC)[reply]
Okay...I think that answers it. Thanks. Vimescarrot (talk) 09:04, 8 March 2010 (UTC)[reply]
You're welcome, and I've now marked this question as resolved. StuRat (talk) 10:18, 8 March 2010 (UTC)[reply]

Brain calories

Does your brain burn more calories when you're thinking hard? When does it burn more calories (stress?) and less calories (sleep?), and has this been measured? Mac Davis (talk) 19:23, 7 March 2010 (UTC)[reply]

A rather brief and basic view of the brain's energy consumption during thought is given here. Let me know if it doesn't address your question. Regards, --—Cyclonenim | Chat  19:40, 7 March 2010 (UTC)[reply]
The answer is that it burns more calories when active but not a lot more. Many popular accounts give incorrect information about this; here is a recent authoritative discussion. Looie496 (talk) 20:32, 7 March 2010 (UTC)[reply]
(Original research/anecdote) Diabetics have reported greater blood sugar drop, a measure of metabolic rate, while they are asleep and having intensely visually detailed dreams with intricate plotting, compared to sleep without that extra brain work. Of course low blood sugar might have conversely caused the vivid dream images. Edison (talk) 23:24, 7 March 2010 (UTC)[reply]

Single source for printed label nutrition data and all or most UPC/PLC codes

Is there an existing single source for food label nutrition data and UPC/PLC code besides this start-up site or better still a site where a cross reference to the codes used for the extensive USDA food nutrition database at http://www.nutrition.gov/ (where the site is down right now) and the UPC/PLC codes can be found? 71.100.11.118 (talk) 19:26, 7 March 2010 (UTC)[reply]

In-text citation and reference for textbooks and websites in scientific journals

I searched for it but couldn't find instructions on the format for in-text and reference citations for textbooks and websites. I looked at different research papers to follow how they did it but it's hard to find one that cited textbooks or websites. Most papers only cite other research papers. —Preceding unsigned comment added by 70.68.120.162 (talk) 19:33, 7 March 2010 (UTC)[reply]

Here's one for citing websites: [14]. Franamax (talk) 19:48, 7 March 2010 (UTC)[reply]
And this should cover textbooks just fine, looking at the examples, several of them could be textbooks. Franamax (talk) 19:51, 7 March 2010 (UTC)[reply]
Generally, biological journals use either the (Authorname, Year) format for inline citations or a number. There can be some variation from journal to journal. I suggest reading the specific journal's "Instructions to Authors" or similarly named page, usually linked on the journal's home page (e.g. Here are the ones from Cell and Developement both found under "For Authors" tabs on the main page). It will tell authors which style to use and may have Endnote stylesheets available for download (which can also be used by free reference managers, like Zotero - you need to use v. 2.0 to import Endnote styles.). If you are going to be writing articles for publication I strongly recommend using a reference manager. Changing inline citation formatting is a 1 or 2 click operation! -- Flyguy649 talk 20:05, 7 March 2010 (UTC)[reply]
You could sign up for a free trial of RefWorks, import the book/site references you want, and see the product (you select the journal format). 30 days should be long enough to get a quick project done, or form a clear idea. Alternatively, have a look at our article on reference management software and try one of the open source products. Doing this by hand is so 20th century! -- Scray (talk) 20:39, 7 March 2010 (UTC)[reply]

Cyborgs

How close is science to having cyborgs similar to those in films and tv shows; ie a human whos organs are both organic and machine? —Preceding unsigned comment added by Heartofsand (talkcontribs) 21:23, 7 March 2010 (UTC)[reply]

Very far away indeed from what you are probably imagining. But you have to define your terms. There are people walking around with artificial hip joints who are (technically) part man, part machine today. People have had false teeth for hundreds of years. Wooden "peg" legs go back even further. What percentage of man has to become machine before you attach the label "Cyborg"? SteveBaker (talk) 21:50, 7 March 2010 (UTC)[reply]
Well, they did say "organs", by which I assume they mean to exclude bones and teeth. An artificial heart should pass that test, and we've had those, but they never really seemed to catch on. A pacemaker might qualify, and perhaps a cochlear implant and an insulin pump. There's also a type of implant for the blind. See artificial organ for more info. StuRat (talk) 22:04, 7 March 2010 (UTC)[reply]
Bones and teeth are collections of different tissues serving a common purpose, so they are organs. The first sentence of bone seems to agree with me. --Tango (talk) 01:41, 8 March 2010 (UTC)[reply]
Yes, but that's clearly not the definition the OP is using, and I don't think anyone would consider somebody with false teeth to be a cyborg. StuRat (talk) 09:01, 8 March 2010 (UTC)[reply]
Exactly. So we can say that there are already plenty of "Cyborgs" in our society today - and have been for a long time. The problem is that this clearly isn't what Heartofsand is asking. "Similar to films and TV shows" is clearly something dramatically more than we're talking. So let's try to pick a couple of TV/Movie "Cyborgs". I guess at one extreme, we have something like "The Terminator" - which is pretty much all machine - but with organic skin. At the other, perhaps something like "The Million Dollar Man" - who is a regular human with a bunch of machine parts added. Clearly, if we can replace one hip joint, we could replace both of them - and probably a bunch of other joints too. But we can't replace all of the bones because bones are where things like blood cells are made and you'd die pretty quickly if we replaced them all. But there are certainly artificial hearts and all sorts of things like artificial limbs of varying kinds. But making someone who can run at 80mph is really out of the realms of possibility. Our implant technologies are almost all inferior to the original body parts. I can't think of anything we implant that doesn't need to be replaced after 20 to 30 years.
The Terminator is a different problem. We might be able to grow skin and fat and figure out how to wrap them around a machine - but we can't build fully active, intelligent robots - we don't have the motor and battery technology - we don't have the AI capability. So forget the 'cyborg' angle - we can't build intelligent robots - so the whole wrapping them with skin thing is irrelevent.
SteveBaker (talk) 04:33, 8 March 2010 (UTC)[reply]
The main problem with building a cyborg is that you cannot easily mate the human nervous system with a machine "body"; and the human brain -- or at least the human mind, whatever part of the brain it is confined to -- is the key to being a proper cyborg. Now, cyborgs in sci-fi come in several flavors. "Terminator" - style cyborgs are not technically cyborgs, as they have no human parts to begin with; they are androids. So are the replicants in the "Blade Runner" and the type A7Mx in "Yokohama Kaidashi Kikō". Those are many tens of years away; maybe hundreds. "Ghost in the Shell" - style cyborgs are cyborgs with more and more human parts replaced by cybernetic devices, dealing directly with the Ship of Theseus issue of at what point a human is no longer human; YMMV. But this is also tens of years in the future. "Hacking the mind" may be further away yet, as the human brain does not internally use a deterministic code of any sort the way modern computers do (although motor commands to the muscles and sensory signals from the skin are pretty deterministic, or at least stereotypical). Taking a human head and spine (or head and torso) and mating them to a machine body, Darth Vader style, may be not that far away. Fairly advanced artificial limbs are available at present. Artificial hearts, lungs, and kidneys are also available, but not yet the easily portable ones. Artificial liver - not as far as I know, but maybe in a decade or two. Human liver can regenerate anyway, if the damage is not too severe. We'll see how fast this is going. The progress seems to have slowed down quite a bit now that we don't have the Cold War anymore, but hopefully it will pick up the pace again; maybe when cyborg building becomes commercially appealing. It may also be of interest to you that we actually have List of fictional cyborgs and articles on Cyborgs in general and Cyborgs in fiction in particular. --Dr Dima (talk) 04:24, 8 March 2010 (UTC)[reply]

F-ATPase

How is that pronounced? "ai, ti, pi, ase"? "ai, ti, pase"? "Adenosine Triphosphatase"? Thanks, 21:25, 7 March 2010 (UTC) —Preceding unsigned comment added by 89.139.173.179 (talk)

It's pronounced /ɑː(ɹ)s/
Whilst true, I suspect the OP wants a simpler answer. I've always said "ai-ti-pi, ase" and "adenosine triphosphatase" Regards, --—Cyclonenim | Chat  22:00, 7 March 2010 (UTC)[reply]
Concur; it's always referred to as F--A-T-P-ase (the letters + -ase suffix). That's roughly Template:Pron-en. -- Flyguy649 talk 22:10, 7 March 2010 (UTC)[reply]

measurement for light

What is the measurement for light? I've heard people using candlepower, is that an official term? What constitutes one unit of candlepower? —Preceding unsigned comment added by Venture4356 (talkcontribs) 22:45, 7 March 2010 (UTC)[reply]

If you mean brightness of light, that's called luminosity. Note that there's a difference between the total amount of light given off by an object and the amount which falls on any particular area. The unit candlepower is now largely replaced by the candela. There's also albedo, which is a measure of the reflectivity of a surface, and the frequency/wavelength (color) of light is often measured in THz or nM. StuRat (talk) 22:53, 7 March 2010 (UTC)[reply]
There are many confusingly similar ways to specify the brightness or light output of a light source. While in the 19th century they referred to "candlepower" now they specify a light bulb output in "lumens" A candlepower was the brightness of a certain kind of "standard" candle, burning at a certain rate, but in practice the observed brightness varied widely between standard candles and in the same candle over time. One candlepower = 12.57 or 4 pi lumen per [15]. But the conversion is misleading: it assumes uniform output in all directions. Candlepower is the light emitted in a particular direction, so a bulb in a reflector would have more candlepower in the direction where the reflector concentrates it. Lumens integrate the light emitted in all directions,per [16]. A light source with a lens or reflector might have lots of candlepower in a very small angle, but not all that many lumens of total light output. The SI unit of luminous intensity is the Candela, selected to be about equal to the old "candle." Edison (talk) 01:51, 8 March 2010 (UTC)[reply]

Disulfide bond connecting two adjacent helices not connected by a loop/turn

How common are disulfide bonds that connect two adjacent helices which are not connected by a loop/turn? —Preceding unsigned comment added by 70.68.120.162 (talk) 23:18, 7 March 2010 (UTC)[reply]

Not sure regarding the loops and turns bit, but disulphide bridges are pretty common. Regards, --—Cyclonenim | Chat  09:31, 8 March 2010 (UTC)[reply]

Talking parrots

Why is it that some species of parrot/cockatoo are better at mimicing human speech than others? Is there an anatomical or inteliigence reason? Something like an African Grey Parrot or an Amazon parrot is a very clear talker and sounds a bit like a human child speaking, wheras other species can talk - but their voices are basically just birdy squawks, tweets and growls shaped into words and syllables, or approximations of words and syllables.

Also - why is it that the Mynah is such a good mimic? It's even better than parrots, from what I've heard, despite not being a very intelligent creature. --95.148.104.245 (talk) 23:51, 7 March 2010 (UTC)[reply]

It's not just intelligence that matters. An animal that can talk does need to be able to remember what it hears, but not all animals that are intelligent can talk (take Chimpanzees for example. The biggest factor of whether or not an animal can talk are the organs that make the noise and how they are arranged (see Speech production), so a human can never bark and a dog can never talk. --The High Fin Sperm Whale 23:58, 7 March 2010 (UTC)[reply]
I should point out that the term "talk" is being used here to mean "mimic human speech". No animal actually talks, meaning using human speech fully (although they do communicate). StuRat (talk) 00:01, 8 March 2010 (UTC)[reply]
OK - StuRat - you're going to have to stop guessing and saying that things aren't true without some evidence. We have articles Alex (parrot) and N'kisi that certainly show at least a strong possiblity that African Grey parrots can learn to use language "for real". "Fully" is a tough standard to meet - quite a few humans that I've met don't use the language "fully". But I've seen plenty of documentaries about Alex and he had me convinced. Several of the great apes have been trained to use sign language - complete with grammar, inventive use of signed speech, etc. It's OK to be wrong - but it's not OK to be persistently, annoyingly and definitely wrong. So how about you slow down your answers and actually do some checking and referencing before you jump to a negative conclusion. SteveBaker (talk) 04:13, 8 March 2010 (UTC)[reply]
It's a bit rude to just accuse me of guessing. I've read several scientific journal articles and seen several documentaries of animal intelligence and specifically speech, and my answer was based on their conclusions. The sign language is an impressive form of communication (particularly in the case of Koko), but that's not speech, which our article defines as "vocalized human communication". They go on to define it as using a large number of words, typically over 10000. No animal uses anywhere near that many human words. Read our article and then tell me how, in any way, Alex or any other animal meets those definitions. Also, the question was about animal species capable of speech, which I take it means that most of the species is so capable, not just a rare exception. StuRat (talk) 08:33, 8 March 2010 (UTC)[reply]

Chicken butt

Hello all. When I was little, my grandfather used to warn us that the qiao qiao (the back part of a chicken that sticks up) of a chicken would make us sick if we ate it, reasoning that the chicken stores its toxins there. I came across such a "qiao qiao" and that made me think of this again. I was wondering: Is there any scientific merit to this assertion? 68.76.146.111 (talk) 23:53, 7 March 2010 (UTC)[reply]

No. StuRat (talk) 23:58, 7 March 2010 (UTC)[reply]
Now, no doubt others will complain that I need to add detail, so here goes:
1) I doubt if we have an article on that part of the chicken. I don't know it's name, so don't know how to find it, if we do. And, even if we do have such an article, I don't believe it will address the issue of toxicity in any way.
2) Since chicken is a common food item, we would know if any part of it is toxic.
3) The tail of a chicken is there to hold tail feathers, not toxins. StuRat (talk) 00:10, 8 March 2010 (UTC)[reply]
Of course there is an article, see Pygostyle, although it does not have the food properties, it is very fatty. Graeme Bartlett (talk) 00:39, 8 March 2010 (UTC)[reply]
My mother, who introduced me to the terms Parson's nose and Pope's nose, said that it was bitter and unpleasant. This book and our article suggest many people disagree with her. 86.177.121.239 (talk) 00:58, 8 March 2010 (UTC)[reply]
I added a redirect to fix the redlink on Parson's Nose. StuRat (talk) 01:10, 8 March 2010 (UTC)[reply]

Perhaps she was getting confused with the cloaca, which is where the eggs, feaces and urine leave the bird. Probably not the cleanest part of it. I think this is normally removed with the rest of the digestive systems when the bird is butchered. 62.56.68.180 (talk) 01:16, 8 March 2010 (UTC)[reply]

The issue of toxins deserves address, as in ordinary language it usually represents an imaginary concept from the century before last. In its strict sense a toxin is a biologically produced substance that functions as a poison to other organisms. One of the mechanisms of disease theories of 19th century medicine was that a mechanism of illness could be the failure to eliminate endogenously produced toxins, leading to "autointoxication". Other 19th century mechanisms of disease included subluxation, miasma, masturbation, and evil spirits. There are two major organs that eliminate dangerous compounds produced endogenously or ingested (the liver and kidneys), and hence the only real diseases produced by autointoxication are the effects of liver or kidney failure. The other major diseases of toxins result from exogenous toxins such as botulism or mycotoxins. We can acquire bacteria (not toxins) from food that has been kept in unsafe conditions. We can acquire bacteria and viruses from feces of other humans, and occasionally from the fecal bacteria of other animals like salmonella from chicken or E. coli from beef. Uneducated people conflate cultural categories of "clean" and "unclean" food with food-borne illnesses. Scientists moved on from the old autointoxication theory in the early 20th century, but quacks and con men continue to find it very profitable to sell people ways to eliminate these imaginary toxins. alteripse (talk) 01:44, 8 March 2010 (UTC)[reply]

March 8

Scientific journals

I went investigating today to see about subscribing to a meteorological journal. Upon seeing the prices, I quickly reconsidered the wisdom of that idea. Am I interpreting it correct that these prices are up to around $110 an issue?! If so, what on earth causes such massive prices for an academic journal of all things?! Ks0stm (TCG) 01:21, 8 March 2010 (UTC)[reply]

The usual problem is very low circulation, meaning they have to charge each subscriber a lot. A lack of advertising can also be a factor. StuRat (talk) 01:40, 8 March 2010 (UTC)[reply]
Price is determined more by what people are willing to pay than by cost. Cost sets a lower limit on price, but that is all. People (well, institutions, more often) are willing to pay that much for the journal, so the journal charges that much. Presumably, the publishers have determined that charging less wouldn't result in sufficiently more sales to get a higher total revenue. --Tango (talk) 01:44, 8 March 2010 (UTC)[reply]
Except not really, because most academic journals are actually sold to libraries and institutions in subscription bundles with various other journals from the same publisher. The cost of the bundle is generally much less than the sum of the face value prices on the individual components, so the institution generally ends up paying much less per work than the apparent price anyway. Dragons flight (talk) 03:47, 8 March 2010 (UTC)[reply]

The principal purchasers are institutional libraries, as they have priced themselves out of the individual market. alteripse (talk) 01:59, 8 March 2010 (UTC)[reply]

If you are interested in reading papers from a meteorological journal without having to subscribe, you should look into universities in your area. I see you're from Kansas. I don't exactly know how things work in Kansas, but in Nebraska, since UNL is a state school, the library offers library cards to Nebraska residents [17], and also has a high school students program [18]. Perhaps the universities in Kansas have similar policies. —Bkell (talk) 04:59, 8 March 2010 (UTC)[reply]

Traditionally, academic journals provided important services - organization of peer-review, proof-reading, typesetting, printing, and dissemination. Due to the low circulation, prices had to be high - and, in fact, library subscriptions are often much more expensive than individual subscriptions. Within the last 20 years or so, this role of publishers has gone down. Authors are now expected to submit camera-ready manuscripts in many fields, and dissemination is increasingly electronic. Peer review is cheaper and easier too, thanks to electronic communication. As a result, scientific publishing is undergoing massive changes, and its hard to predict what will happen over the next 10-20 years. --Stephan Schulz (talk) 10:37, 8 March 2010 (UTC)[reply]

Colour of glass

How is colour added to glass? I especially want to know for 'blue'. -- Extra999 (talk) 07:52, 8 March 2010 (UTC)[reply]

Colour is added to glass by adding certain metal salts to the molten glass (see Glass colouring and colour marking). Blue is made with Cobalt salts, I believe. --The High Fin Sperm Whale 02:51, 8 March 2010 (UTC)[reply]
Substances Used in the Making of Coloured Glass may be your friend. Cobalt seems to be the key to blue glass. --Tagishsimon (talk) 02:48, 8 March 2010 (UTC)[reply]

Thanks. -- Extra999 (talk) 18:00, 8 March 2010 (UTC)[reply]

The Lions

Hello, I live in Langley, BC, and I was wondering if you can see The Lions from here. I did see one mountain that looked a bit like them, but it was tiny and off in the distance. So can you see them from Langley? --The High Fin Sperm Whale 02:40, 8 March 2010 (UTC)[reply]

Assuming clear air. the old-fashioned pre-metric rule for how far away the horizon is when you are at h feet above sea level is: distance (miles) = sqrt(h x 7/4). So if you stood on top of the taller of the two peaks (5,400 feet) - then the horizon would be about 97 miles away. So you should be able to see the peak from sea-level locations within 97 miles. Google maps says that the center of Langley is about 45 miles away. So the curvature of the earth certainly isn't an obstacle. Whether there are intervening obstacles...I'm not sure. Whether the atmosphere is clear enough...I don't know. But if I had to guess, I'd say you'd be able to see the peaks on a clear day. SteveBaker (talk) 03:52, 8 March 2010 (UTC)[reply]
That's a neat formula. A back-of-the-envelope calculation (starting with the formula Steve provided) yields a metric approximation: distance (km) = sqrt(h[meters] x 15). This metric version is gives answers that are 0.5% greater (about a km for most peaks) than the formula SB gave above. In this specific case, 5400 ft is about 1646 m, yielding a visibility estimate of 157 km, or 97.6 miles (pretty close). The error (relative to the original formula) drops to 1.2% if you use 14.9 instead of 15 in the square root - but I am not sure that's worth it -- Scray (talk) 12:25, 8 March 2010 (UTC)[reply]

bombardment of hydrogen isotopes with neutrons

What happens when a plasma made up of all the different isotopes of hydrogen is bombarded with neutrons? 71.100.11.118 (talk) 03:32, 8 March 2010 (UTC)[reply]

Then it would become heavier isotopes, first deuterium then radioactive tritium. --The High Fin Sperm Whale 03:34, 8 March 2010 (UTC)[reply]
...and thereafter (in the presence of all three isotopes)? 71.100.11.118 (talk) 03:51, 8 March 2010 (UTC)[reply]
I suppose that if tritium was bombarded with neutrons, a neutron would decay into a proton and electron, forming helium. --The High Fin Sperm Whale 03:57, 8 March 2010 (UTC)[reply]
Is there any evidence for this say at higher densities and lower temperatures as close to absolute zero as possible and under maximum pressure or in high temperature, high pressure plasma? 71.100.11.118 (talk) 04:14, 8 March 2010 (UTC)[reply]
It is interesting that Hydrogen-4 and Hydrogen-5 and Hydrogen-6 will not be produced, since they actually decay back to tritium in sub attosecond timescale by giving off neutrons. (actually in a zeptosecond which apparently is well below the measurable time scale). Graeme Bartlett (talk) 06:50, 8 March 2010 (UTC)[reply]

Can persistent high blood pressure be someone's fault?

(1) Is it possible for someone (annoying and clingy and argumentative and stubborn, but never really cruel - just basically someone immature who doesn't play well with others) to drive someone's blood pressure up so high that they would be in medical danger and possibly even suffer long-term damage after being stabilized? (2) Or is it more likely that the hypertensive crisis would cause the person with high blood pressure to perceive the immature individual as more of a problem than s/he actually is? (3) And if the latter is the case, is it likely that the animosity the person with high blood pressure felt would pass when the blood pressure returned to a normal range? —Preceding unsigned comment added by 71.108.171.138 (talk) 03:50, 8 March 2010 (UTC)[reply]

There is a tested and proven correlation between stress and hypertension. The statement "stress causes hypertension" is not the same as "stress is correlated with hypertension." Therefore, you are making two jumps: annoying person causes stress and stress causes hypertension. As such, it is unlikely to find many references that justify that double-claim. -- kainaw 03:54, 8 March 2010 (UTC)[reply]

(4) Does that mean it's unlikely that one person's faults could singlehandedly drive another person into hypertensive crisis? (5) And more importantly, could high blood pressure cause a person to think or behave irrationally, particularly to prompt feelings of anger or paranoia? —Preceding unsigned comment added by 71.108.171.138 (talk) 03:58, 8 March 2010 (UTC)[reply]

It's hard to prove a negative, but all evidence suggests the answers are 1 No, 2 No, 3 No, 4 Yes, and 5 No respectively. alteripse (talk) 04:21, 8 March 2010 (UTC)[reply]

Hmmmmm . . . if I follow your answers correctly, you're saying that high blood pressure can prompt or exaggerate angry feelings (which is all I meant by "irrational thinking," by the way . . . such as blaming someone for something that really isn't his or her fault, and no clear-headed person would truly think it was . . . but ira est furor brevis)?

No. Ira sit furor brevis, sed non hypertensio. I went back and numbered your questions so you can match up the answers correctly. Hypertension is dangerous because it is asymptomatic (silent). The fluctuation with emotional upset results from adrenaline, and adrenaline elevations from emotions may be perceptible because of other adrenaline effects, but tend to be too brief to represent a long term health risk. There is a less well understood epidemiologic relationship between chronic life stress and hypertension but the mediating factors are not known and may include other things associated with poverty rather more than simply prolonged emotional stress. alteripse (talk) 11:42, 8 March 2010 (UTC)[reply]

menstrual period information

menstrual period information —Preceding unsigned comment added by Lorir7 (talkcontribs) 03:55, 8 March 2010 (UTC)[reply]

Asking or offering? You know, I'm sure Wikipedia has an article on this. At least one. —Preceding unsigned comment added by 71.108.171.138 (talk) 03:59, 8 March 2010 (UTC)[reply]

I suggest you start at Menstruation, and go from there. Dolphin51 (talk) 04:00, 8 March 2010 (UTC)[reply]
See below a question "menstrual period correlation with Metonic and other lunar cycles". Cuddlyable3 (talk) 11:40, 8 March 2010 (UTC)[reply]

Orbit

The moon orbits around the Earth and the Earth orbits around the Sun and the Sun orbits around the center of the Milky Way. What does the Milky Way orbit around? 71.100.11.118 (talk) 04:32, 8 March 2010 (UTC)[reply]

I don't believe the Milky Way follows a periodic orbit. They usually only exist when there are just two bodies of significant mass to consider (eg. when considering the Earth and the Sun, you can ignore the Moon and other planets as being too small or too far away to do much, or when considering the Earth and Moon, the Sun and everything else is too far away to worry about). For the Milky Way, that doesn't really work. The Local Group contains a large number of galaxies, several of which are too large or close to ignore. Over a short period of time, the Milky Way will act as though it is orbiting the centre of mass of the Local Group, which is somewhere roughly in the middle of the Milky Way and the Andromeda Galaxy (which, between them, make up most of the mass of the Group). Over longer time periods, it is much more complicated. --Tango (talk) 04:49, 8 March 2010 (UTC)[reply]
...humm. Thinking in terms of center of mass and the fact the that whole Universe is suppose to be related or connected by gravity what center of mass would the center of mass of the local group be attacted to? 71.100.11.118 (talk) 08:46, 8 March 2010 (UTC)[reply]
There is no center of mass for whole universe. See Cosmological Principle. manya (talk) 09:16, 8 March 2010 (UTC)[reply]
But nearby us, Great Attractor plays some role. - manya (talk) 09:24, 8 March 2010 (UTC)[reply]

Why is hunger so mild?

Is there a biological reason for why starvation is not intensely painful? It seems to be one of the most painless ways to die naturally, excluding instantaneous deaths, and is nothing like having cancer or dehydrating. --99.237.234.104 (talk) 04:48, 8 March 2010 (UTC)[reply]

Well, I've never had cancer or dehydrated, so I can't compare them . . . and I've never starved either for that matter. But I've skipped enough meals to wonder what on earth makes you think that starvation isn't intensely painful. That clawing sensation of your empty stomach cramping up begging for food? Ouch. Really, I think I'd prefer dehydration . . . I've never heard what it feels like, beyond that you'd be really really thirsty, which I imagine would be terribly unpleasant but much less painful than starvation. To try to take your question seriously, though, starvation is less acute a condition than dehydration. When you need water, you need it within a day or two at most to keep functioning. In the wild, animals often have to go for many days, longer at times, without food, and they need to be able to focus on other things and function in spite of the discomfort in order to keep up the search for food and other necessities and survive. In time of dehydration water is almost always going to be the first priority, but in starving times there are likely going to be many intervening priorities, as it may be quite some time before food can be found. So it wouldn't do to have hunger be as keen a drive. But in the final stages, of course it's intensely painful. And it's not terribly pleasant up to that point either . . . especially since it's drawn out over several weeks.


One major symptom of dehydration is extreme headaches - not pleasant at all. It is far quicker than starvation, though (about 3 days, rather than 3 months). --Tango (talk) 04:56, 8 March 2010 (UTC)[reply]
Dying of starvation is not at all pleasant and takes a very long time. The reason hunger is usually a very mild sensation is because you rarely get very hungry. Most people eat several times a day but they could go months without food, so clearly they are eating when hunger is still at a very early stage. See Starvation#Signs and symptoms for a description of some of the things that happen to your body when you don't eat. One part that jumps out at me is "All movements become painful due to muscle atrophy". --Tango (talk) 04:56, 8 March 2010 (UTC)[reply]

Ah, headaches. True enough. I've wondered often enough when I've downed a couple of Tylenol with a glass of water if it wasn't the water that relieved the headache at least as much as the medicine.

I've heard that for about the first three days of not having anything to eat, it is painful because your stomach starts to shrink and cramp, thus causing pain. Then, for the next 40 days, it stops hurting and your body starts to digest fat reserves. After that, it starts to hurt again, because your body starts digesting the muscles. Your eyes are one of the first to go. --The High Fin Sperm Whale 05:13, 8 March 2010 (UTC)[reply]
I have fasted for seven days at a stretch, water only, many times. It's not painful. Climbing stairs gets more tiring after 5 days or so. The hunger seems more psychologically strong in the first 3 days, then it gets weaker. Your sense of smell gets quite acute - it becomes disgusting to be near a fast food place or anywhere where there has been poison used recently. At 7 days, there are no problems with sight. —Preceding unsigned comment added by Marklawrence17 (talkcontribs) 08:06, 8 March 2010 (UTC)[reply]

Eyes aren't muscles. And look at that link . . . there's a lot more to the pain of starvation than the initial stomach cramps. —Preceding unsigned comment added by 71.108.171.138 (talk) 05:16, 8 March 2010 (UTC)[reply]

The eyes do have & need muscles however and if these go your vision will almost definitely go, see Extraocular muscles, Muscles of orbit, Ciliary muscle, Iris dilator muscle & Iris sphincter muscle for example. Whether there are really the first to go, I don't know but it's probably your vision will be strongly negatively affected if they start to atrophy whereas movement might be incredibly painful but possible even after your limb muscles have partially atrophied. Since headaches came up, I'm surprised no one mentioned hunger or fasting can cause headaches as well I presume due to mild hypoglycemia. However when it comes to real starvation, I don't know if this will persist since it seems likely when your body adapts it shouldn't be a problem. Nil Einne (talk) 06:43, 8 March 2010 (UTC)[reply]

An interesting website

I'm no sure if this is the right desk to ask this on, but I'm having trouble finding a website that i once visited. It was someone's personal website and he/she discussed topics including what a Neanderthal language might have been like, including a translation of genesis into his hypothetical neanderthal, and a list of scientific inaccuracies in Star Trek. There was also a fictional account of how civilization in the future was destroyed by people wanting to learn new words constantly to fit in with the latest dialect, so they overused neuroplasticisers and gave themselves brain damage. Does anyone know of anything like this? 74.14.109.146 (talk) 06:49, 8 March 2010 (UTC)[reply]

perhaps http://adamjamesnall.blogspot.com/ ? —Preceding unsigned comment added by Graeme Bartlett (talkcontribs) 07:07, 8 March 2010 (UTC)[reply]
You're thinking of http://www.xibalba.demon.co.uk/jbr/index.html. Coincidentally I just discovered this page two days ago, at the end of a chain of link-following starting with a random Wikipedia image which turned out to be one of the Sammarinese euro coins, which led me to San Marino, which led to Akademio Internacia de la Sciencoj San Marino, which got me curious about Esperanto, so I did a Google search for learn esperanto, and the fourth result was Learn Not to Speak Esperanto. (Now why in the world can I remember all of that?) —Bkell (talk) 07:21, 8 March 2010 (UTC)[reply]
Ah, yes, it's that one. As I believe is customary, I hereby grant you one (1) internet. 74.14.109.146 (talk) 08:12, 8 March 2010 (UTC)[reply]
Yay! I have an Internet! Bkell (talk) 08:29, 8 March 2010 (UTC)[reply]

menstrual period correlation with Metonic and other lunar cycles

How does the human female menstrual period compare with all of the various lunar cycle and with which one does it correlate best? 71.100.11.118 (talk) 08:43, 8 March 2010 (UTC)[reply]

THe Wikipedia article Menstrual cycle notes that the menstrual cycle varies from woman to woman and the average is 28 days. The moon takes 27.32 days to revolve around the Earth; the period between full moons is 29.53 days. Opinions differ on whether moonlight influences menstruation (see Lunaception) or the similarity of cysles is a coincidence. See also the article on the McClintock effect supposed to synchronise the menstrual cycles of women who live together.Cuddlyable3 (talk) 11:24, 8 March 2010 (UTC)[reply]

scientists

Do all scientists keep notes? What are some famous scientists who never kept any notes and worked in a very haphazard way? —Preceding unsigned comment added by Milticcason (talkcontribs) 09:07, 8 March 2010 (UTC)[reply]

I would think that every scientist would want to keep notes, if they could. Perhaps there were some illiterate scientists (particularly in societies that hadn't yet developed a written language). Also, some might not want to take notes as that might provide proof that they were engaged in an illegal activity. Those studying anatomy, for example, often illegally obtained bodies for dissection. StuRat (talk) 09:21, 8 March 2010 (UTC)[reply]
To your first question, the short answer is "no". Just as there are different learners out there (some learn from lectures, some from reading, some from copious note-taking, etc) there are different researchers - and I've seen the full range of laboratory records, from none to compulsive. As indirect evidence, I cite the plethora of guidelines like this that admonish lab personnel to document what they do - because these would be superfluous if people weren't "haphazard". Certainly, there may be some who keep no records to hide malfeasance, but there are others who just don't keep anything we'd recognize as useful records (except when they publish - but that's not "notes"). -- Scray (talk) 11:55, 8 March 2010 (UTC)[reply]

gravity versus electromagnetic forces

What is the formula that shows the point of balance between the electromagnetic repelling force of orbiting electrons and the pull of gravity between two atoms of hydrogen such that it can be applied to multiple atoms of hydrogen right up to the size of a star? Also, what is the formula for the density of hydrogen atoms due to gravity and the diameter of the whole? 71.100.11.118 (talk) 09:53, 8 March 2010 (UTC)[reply]

The high temperatures in a star mean that most of the hydrogen atoms are ionised - their electrons are stripped away, and the free electrons and protons form a plasma. The main force opposing gravitational collapse in a main sequence star, in which fusion is still active, is not electromagnetic repulsion - it is the thermal pressure arising from the energy released in the fusion reactions, which appears as kinetic energy in electrons and protons plus radiation pressure from photons. When a star has insufficient fuel to continue fusion, other forces such as electron degeneracy pressure become important. See our article on stellar evolution for more details. Gandalf61 (talk) 10:30, 8 March 2010 (UTC)[reply]

Lorentz transformations

If the velocities between two reference frames were such that the relative motion were not parallel to one of the axes, what would the lorentz transformations look like? —Preceding unsigned comment added by 173.179.59.66 (talk) 10:32, 8 March 2010 (UTC)[reply]

about thrusters used in the spacecrafts

hey.. please someone let me know about the working of the thrusters in the vaccum(space) in the absence of any medium which would provide a reaction to the craft's linear motion.. e-mail address deleted —Preceding unsigned comment added by 116.73.242.109 (talk) 11:23, 8 March 2010 (UTC)[reply]

See the article about the Rocket engine which obtains thrust in accordance with Newton's third law that says "For every action force there is an equal, but opposite, reaction force". It needs no external material to form its jet and therefore it can drive a spacecraft. See the article Spacecraft propulsion. Cuddlyable3 (talk) 11:32, 8 March 2010 (UTC)[reply]

Happiness

I was watching the simpsons and there was a chart which showed that less intelligent people are happier. Is there any evidence to support this claim? Any studies? —Preceding unsigned comment added by Doorelore (talkcontribs) 11:56, 8 March 2010 (UTC)[reply]

If you search Pubmed there are plenty of studies which correlate intelligence with suicide rates and similar ideas, which may be relevant. Unfortunately, I don't have access to these. Perhaps someone else will :) Regards, --—Cyclonenim | Chat  12:17, 8 March 2010 (UTC)[reply]

Artificial diamond

How is artificial diamond made? -- Extra999 (talk) 18:10, 8 March 2010 (UTC)[reply]

What is meant by decreasing the "vacuum"???

What is meant by decreasing the vacuum?? Is it increasing the vacuum pressure hence decreasing the quality of it?? Or decreasing the emptiness???

  1. ^ Insert footnote text here